SlideShare une entreprise Scribd logo
1  sur  90
Télécharger pour lire hors ligne
To download more new and updated pediatrics’ books,
Journals and guidelines you can visit my medical BlogSpot and
4shared on the following links
http://drsamed77paediatricbooks.blogspot.com/
http://www.4shared.com/u/N86Q7qyj/drsamed77mrcpch1.html
For contact
mailto:dr.samed77@yahoo.com
mailto:dr.samed77mrcpch1@gmail.com
mailto:dr.samed77@windowslive.com
Development 1
Development
Theme : Stature
A. Achondroplasia
B. Anorexia nervosa
C. Constitutional delay
D. Cornelia-de-Lange syndrome
E. Familial short stature
F. Growth hormone deficiency
G. Primordial dwarfism
H. Psychosocial poor growth
I. Prader-Willi syndrome
J. Russell-silver dwarfism
Select the most appropriate diagnosis from the above options to explain the following
presentations:
1) A child has short stature, a triangular face
and facial and limb length asymmetry.
J. Russell-silver dwarfism
Note:
describes a child with Russell –silver syndrome. A condition of short stature, hemi-hypertrophy and the
child often has triangular facies, frontal bossing and clinodactyly of the 5th finger.
2) A child has short stature and is noted to
have a large head with a prominent
forehead.
A. Achondroplasia
Note:
describes a child with achondroplasia. Short stature, rhizomelia (short limbs especially the proximal part).
Affected individuals usually have a large head with hydrocephalus being a complication.
3) A baby is born on the 50th centile for
height and weight. By the age of 2 her
length has fallen to the 2nd centile and
weight remains on the 50th. She has a
rounded face with small hands and feet.
F. Growth hormone deficiency
Note:
describes a growth hormone deficient child. Early recognition and initiation of treatment is required to
reach satisfactory adult height.
Theme : Stature
A. Achondroplasia
B. Anorexia nervosa
C. Constitutional delay
D. Cornelia-de-Lange syndrome
E. Familial short stature
Pediatric extended match questions Dr samed alsalmi
Development 2
F. Growth hormone deficiency
G. Primordial dwarfism
H. Psychosocial poor growth
I. Prader-Willi syndrome
J. Russell-silver dwarfism
Select the most appropriate diagnosis from the above options to explain the following
presentations:
1) A 9 year old boy statemented for special
educational needs is investigated for short stature.
He has a weight over the 91st centile. He is noted
to have small hands and feet and hypogonadism.
I. Prader-Willi syndrome
Note:
describes a child with Prader-Willi syndrome. A condition characterised by obesity after the first year of
life. Hypotonia, hypogonadism and learning difficulties due to a deletion of chromosome 15 (the deletion
occurring in the paternal gene, if the deletion of chromosome 15 in the child was inherited from the
mother the child would have the phenotype of Angelman's syndrome).
2) A 15 year old girl in foster care is referred to
outpatients because of delayed puberty. On
examination her weight is on the 2nd centile and
her height is on the 25th centile. She has poor
dentition and her temperature is measured at
35°C.
B. Anorexia nervosa
Note:
describes an adolescent girl with anorexia nervosa. A condition due to fear of becoming obese or
disturbed body image. There is usually a tendency to deny hunger, excessive dieting, laxative abuse or
excessive physical activity. The diagnosis is classified as weight of 15% below that expected for age and
height and is associated with physical characteristics including amenorrhoea, bradycardia, hypothermia
and electrolyte disturbances.
3) A 7 year old girl presents with short stature with
weight being on the 9th centile and height less
than 0.4th centile. Examination is normal as well as
all investigations including a bone age, which is
compatible with her chronological age. Her mother
is 5”1 (134cm), her father is 5”5 (143cm).
E. Familial short stature
Note:
describes a child with familial short stature. From the measurements of the parents the child's height is
compatible with this diagnosis. It is expected that most children will reach a height within 8cms of the
mid-parental height centile.
Pediatric extended match questions Dr samed alsalmi
Cardiology 1
Cardiology
Theme : Emergency medicine
A. Adenosine
B. Adrenaline
C. Atropine
D. DC shock
E. Dobutamine
F. Diving reflex
G. Endotracheal intubation
H. Intraosseous line
I. IV Morphine
J. Naloxone
Select the most appropriate emergency treatment for the following children:
1) A 4-year-old child is rescued from a house fire. She is admitted
tachypnoeic and tacchycardic. She has soot in her nostrils. G. Endotracheal intubation
Note:
Item 1 relates to burns. Most deaths following house fires occur secondary to smoke inhalation. Amongst
the indicators of inhaled smoke injury is deposits around the mouth and nose. Oedema follows thermal
injury and therefore any suspicion of airway compromise should result in endotracheal intubation.
2) A 3 month old baby is admitted with a history of poor feeding. On
arrival he has a pulse rate of 220 beats per minute. F. Diving reflex
Note:
Supraventricular tachycardia is the diagnosis in Item 3. Vagal stimulation is the treatment of choice and
the diving reflex is the simple procedure elicited by submerging the baby's face in to ice or placing an ice
bag over the face. The diving reflex increases vagal tone, slows AV conduction interrupting the
tachycardia.
3) A 13 year old boy is admitted with meningococcaemia. He is in
shock. Peripheral cannulation is difficult. H. Intraosseous line
Note:
Item 2 relates to a child in shock. In many life-threatening conditions venous cannulation is difficult. It is
important to obtain vascular access very quickly and therefore intraosseous infusion is recommended.
Theme : Syndromes associated with congenital heart disease
Pediatric extended match questions Dr samed alsalmi
Cardiology 2
A. Charcot-Marie- Tooth syndrome
B. Down syndrome
C. Fragile X syndrome
D. Marfan's syndrome
E. Noonan syndrome
F. Sturge-Weber syndrome
G. Tetrology of Fallot
H. Tourette syndrome
I. Turner's syndrome
J. William's syndrome
For each of the cardiac lesions described below, choose the most commonly associated
syndrome from the above list of options. Each option may be used once or not at all.
1) Supravalvular aortic stenosis J. William's syndrome
Note:
William's syndrome 7 is characterized by short stature, characteristic facies, supravalvular aortic
stenosis, mild to moderate learning difficulties and transient neonatal hypercalcemia.
2) Coarctation of Aorta I. Turner's syndrome
Note:
Turner's syndrome 7 This is characterized by 45, X genotype, ovarian dysgenesis leading to infertility,
short stature, webbing of the neck, wide carrying angles and wide spaced nipples. However they have
normal intellectual development.
3) Dilatation of aortic root/aortic regurgitation D. Marfan's syndrome
Note:
Marfan's syndrome 7 This is an autosomal dominant disorder. The clinical features are tall stature,
arachno-dactyly, high arched plate and increase in length of the lower segment of the body compared to
the upper segment. The cardiac manifestations include dilated aortic root, aortic incompetence, mitral
valve prolapse and mitral incompetence.
4) Cardiac cushion defects (leading to ASD, VSD) B. Down syndrome
Note:
Down syndrome 7 Features include characteristic facies, hypotonia, severe learning difficulties and
small stature. About 40% of patients have cardiac anomalies mainly endocardial cushion defects leading
to ASD and VSD.
5) Infundibular pulmonary stenosis G. Tetrology of Fallot
Note:
Tetrology of Fallot This is a cyanotic heart disease and the cardinal features include:
1. Infundibular pulmonary stenosis
2. VSD
3. Right ventricular hypertrophy
4. Over-riding of the aorta
Comments:
Charcot-Marie-Tooth syndrome 7 Autosomal dominant peroneal muscular dystrophy.
Fragile X syndrome 7 Moderate learning difficulty, macrocephaly, characteristic facies (long face, large
ears, prominent mandible and forehead)
Noonan syndrome7 Facies, mild learning difficulties, short webbed neck, short stature and congenital
heart disease (pulmonary valvular stenosis, ASD, left ventricular hypertrophy)
Sturge-Weber syndrome 7 Haemangiomas in the distribution of trigeminal nerve and in the brain.
Sometimes can have intractable epilepsy.
Tourette syndrome 7 Tics, compulsive utterances of obscene words (coprolalia)
Theme : Emergency treatments
Pediatric extended match questions Dr samed alsalmi
Cardiology 3
A. Adenosine
B. Adrenaline
C. Atropine
D. DC shock
E. Dobutamine
F. Diving reflex
G. Endotracheal intubation
H. Intraosseous line
I. IV Morphine
J. Naloxone
Select the most appropriate emergency treatment for the following children:
1) A 5 year old boy is brought to the hospital with 15% scalds to
his chest.
I. IV Morphine
Note:
Item 1 relates to a child with serious burns. Children who have been burnt are in severe pain and
therefore IV Morphine is the analgesic of choice.
2) A 14-year-old girl with a history of previous overdoses is
admitted to A&E apnoeic and unconsciousness. ECG shows
ventricular fibrillation. CPR is commenced.
D. DC shock
Note:
Item 2 describes a child in ventricular fibrillation. This is uncommon in childhood although may occur
as a result of tricyclic antidepressant overdose and hypothermia. If the arrest is witnessed a
precordial thump is carried out otherwise electrical de-fibrillation at 2 joules per kilogram.
3) A term baby is born in poor condition. Apgar scores 3 at 1
minute and 5 at 5 minutes. CPR is commenced. At 10 minutes he
remains bradycardic.
B. Adrenaline
Note:
Item 3 describes a baby born in poor condition. A bradycardia in an unstable newborn requires
oxygenation, ventilation and cardiac compressions. IV adrenaline is administered as Atropine is
ineffective in this age group.
Theme : Genetics - cardiac abnormalities in genetic disorders
Pediatric extended match questions Dr samed alsalmi
Cardiology 4
A. Angelman's syndrome
B. Beckwith-Wiedemann syndrome
C. Congenital Rubella syndrome
D. Down syndrome
E. Foetal alcohol syndrome
F. Glycogen storage disease
G. Marfan's syndrome
H. Noonan's syndrome
I. Turner's syndrome
J. Williams syndrome
Match each of the following cardiovascular abnormalities to the single most likely associated
genetic disorder.
1) Dilation of the aorta with aneurysms. G. Marfan's syndrome
Note:
In Marfan's syndrome dilatation of the ascending aorta is often seen with or without aneurysms. Less
commonly the thoracic abdominal aorta or pulmonary arteries are affected with secondary aortic
regurgitation and mitral valve prolapse.
Theme : Congenital cardiac defects
A. Angelman's syndrome
B. Beckwith-Wiedemann syndrome
2) Supra-valvular aortic stenosis. J. Williams syndrome
Note:
In Williams syndrome supra-valvular aortic stenosis is the most common cardiac lesion. Septal defects
also occur as well as peripheral branch pulmonary artery stenosis.
3) Pulmonary stenosis. H. Noonan's syndrome
Note:
In Noonan's syndrome pulmonary valve stenosis due to a dysplastic or thick valve is seen often
associated with left ventricular hypertrophy. Branch stenosis of the pulmonary artery also is found in
Noonan's syndrome.
Pediatric extended match questions Dr samed alsalmi
Cardiology 5
C. Congenital Rubella syndrome
D. Down syndrome
E. Foetal alcohol syndrome
F. Glycogen storage disease
G. Marfan's syndrome
H. Noonan's syndrome
I. Turner's syndrome
J. Williams syndrome
Match each of the following cardiovascular abnormalities to the single most likely associated
disorder.
1) Endocardial cushion defect. D. Down syndrome
Note:
In Down syndrome approximately 40% of children have a congenital heart disease. The most common
being endocardial cushion defects although VSDs, ASDs and PDA also occur.
2) Coarctation of the aorta. I. Turner's syndrome
Note:
In Turner's syndrome cardiac defects are common. 30% include bicuspid aortic valves with the second
most common heart defect being coarctation of the aorta. Aortic stenosis, mitral valve prolapse and
hypertension are also found.
3) Septal defects. E. Foetal alcohol syndrome
Note:
In foetal alcohol syndrome individuals have poor growth, developmental delay and usually characteristic
facial features including microcephaly and a short smooth philtrum. The most common cardiac lesion in
these children are septal defects primarily ventricular septal defects.
Theme : BREATHLESSNESS
A. Asthma
B. Hyperventilation
C. Tuberculosis
D. Cystic Fibrosis
E. Pneumocystis carinii
Pediatric extended match questions Dr samed alsalmi
Cardiology 6
F. Ventricular septal defect
G. Gastroesophageal reflux
H. Atrial septal defect
I. Bronchiolitis
J. Mitral stenosis
For each of these patients with breathlessness, select the most likely diagnosis
1) A thirteen-year-old girl who has intermittent episodes of
breathlessness, which tend to occur in crowded shops. She feels
the need to take deep breaths and then breathes very quickly,
complaining of pins & needles around her mouth and in her
hands. Her chest is clear and her blood gases show a normal
pO2 and low pCO2.
B. Hyperventilation
Note:
This description is one of anxiety. The blood gas picture is one of hyperventilation.
2) A three-year-old boy who presents with worsening cough and
breathlessness of 3 weeks’ duration. His mother was an
intravenous drug abuser. He has always been prone to infections.
When he was 2 years old he had chicken pox for 4 weeks. On
examination, he has an emaciated appearance, his weight is
below the 0.4th centile, he has a temperature of 37.6°C and he
has generalized crepitations on auscultation of his chest. A blood
count shows severe lymphopenia.
E. Pneumocystis carinii
Note:
This has resulted from congenitally acquired HIV. PCP has an insidious onset and often there are no
chest signs in children. Lymphopenia is consistent. Treatment is with septrin or nebulised pentamidine as
second line.
3) A 4-month-old baby has not gained much weight since birth
and only takes small milk feeds, as he appears to become
breathless on feeding. He is tachypnoeic, sweaty and has a
tachycardia. His liver is enlarged and he has a harsh grade 2-
pansystolic murmur at the left lower sternal edge.
F. Ventricular septal defect
Note:
Poor feeding is a symptom of heart failure in babies. A VSD has a pan systolic murmur and if large may
have a lower grade as there is less resistance to flow. Hepatomegaly is an early sign of heart failure in
infants.
Theme : CONGENITAL HEART DISEASE
A. Ostium secundum atrial septal defect
B. Ventricular septal defect
C. Transposition of the great arteries
D. Total anomalous pulmonary venous drainage
E. Atrioventricular septal defect
F. Patent ductus arteriosus
G. Pulmonary valve stenosis
Pediatric extended match questions Dr samed alsalmi
Cardiology 7
H. Coarctation of the aorta
I. Tetralogy of Fallot
J. Hypoplastic left heart syndrome
Which of the above is the most likely diagnosis in the following cases.
1) A 13-year-old girl is referred for evaluation of her short stature. She
is prepubertal. On auscultation she has an ejection systolic murmur in
the second and third left intercostals spaces radiating to the back, but is
a symptomatic.
G. Pulmonary valve stenosis
Note:
The murmur describes pulmonary stenosis, which could also be a left peripheral pulmonary stenosis. She
is short and has delayed puberty and coupled with the cardiac findings would suggest Noonan’s
syndrome.
2) A 7-week-old infant presents with breathlessness on feeding and
failure to thrive. On examination his femoral pulses are difficult to feel
but present. Chest radiograph shows cardiomegaly and increased
vascular markings.
H. Coarctation of the aorta
Note:
Absent or weak femoral pulses suggest coarctation. Remember association with Turner’s syndrome.
3) An infant is seen for his 6-week check and found to have a loud
ejection systolic murmur in the third left intercostal space and a single
second heart sound on examination. There is no obvious cyanosis but
a suggestion of mild desaturation. On the chest X ray there is a
concavity on the left heart border and decreased pulmonary vascular
markings.
I. Tetralogy of Fallot
Note:
Tetralogy of Fallot may present later than in the neonatal period. The ejection systolic murmur is from the
infundibular stenosis. The desaturation results from the right to left shunt across the VSD.
Theme : Congenital heart disease
A. Atrioventricular septal defect
B. Coarctation of the aorta
C. Hypoplastic left heart syndrome
D. Ostium secundum atrial septal defect
E. Patent ductus arteriosus
F. Pulmonary valve stenosis
G. Tetralogy of Fallot
H. Total anomalous pulmonary venous drainage
I. Transposition of the great arteries
Pediatric extended match questions Dr samed alsalmi
Cardiology 8
J. Ventricular septal defect
Which is the most likely diagnosis in the following cases?
1) An infant is found profoundly cyanosed and lethargic in his cot on
day 2. On auscultation there is a soft systolic murmur heard
inconsistently at the left sternal edge and a single second sound.
The chest X ray shows a narrow upper mediastinum, hypertrophied
right ventricle and increased pulmonary vascular markings. The
ECG shows a normal neonatal pattern.
I. Transposition of the great
arteries
Note:
Cyanosis on the second day is suggestive of a duct-dependent lesion. The rest of the answer describes
TGA
2) A 3-week-old premature infant born at 27 weeks gestation
remains ventilated following surfactant deficient respiratory distress
syndrome. On auscultation of his chest a systolic murmur is heard
at the left sternal edge and pulses are very easy to feel. There is
pulmonary plethora on chest X ray.
E. Patent ductus arteriosus
Note:
PDA is a relatively common problem in premature babies. The left to right shunt results in excess blood
flow through the lungs and frequently oxygen dependency and difficulty in weaning from the ventilator. A
loud systolic murmur radiating to the back with easily palpable pulses are typical.
3) A 7-year-old boy is examined for a chest infection. An incidental
finding of a short systolic murmur with fixed splitting of the second
heart sound is detected. His blood pressure is normal and all pulses
are normal.
D. Ostium secundum atrial septal
defect
Note:
Ostium primum defects are unlikely to present incidentally but rather with heart failure or pulmonary
hypertension. Fixed splitting is typical of ostium secundum defects
Pediatric extended match questions Dr samed alsalmi
Endocrine
Theme : Endocrine
A. Addison's disease
B. Congenital adrenal hyperplasia
C. Cushing's syndrome
D. Diabetes insipidus
E. Diabetes mellitus
F. Hyperthyroidism
G. Hypoparathyroidism
H. Hypothyroidism
I. Primary aldosteronism
J. Septo-optic dysplasia
Select the most likely diagnosis from the list above that would explain the presentations of the
following patients:
1) An 11 year old boy who was previously fit
and well presents to the GP with obesity and
purple abdominal striae. His blood pressure
is 140/90.
C. Cushing's syndrome
2) A breast-fed baby presents with poor
feeding. He has a hoarse voice and coarse
facial features. Examination of the skull
reveals wide sutures and a large anterior
fontanelle.
J. Septo-optic dysplasia
H. Hypothyroidism
3) A 7 year old girl with moderate learning
difficulties presents with dry skin, alopecia
and mucocutaneous candidiasis.
H. Hypothyroidism
G. Hypoparathyroidism
Comments:
Cushing's syndrome is a disorder due to high circulating cortisol levels either exogenous or endogenesis
in origin. Clinical features include arrested growth, trunk obesity, hirsuitism, muscle wasting and mood
changes. Purple striae are often seen. Laboratory investigations may reveal polycythaemia, abnormal
glucose tolerance and high cortisol levels.
2-Congenital hypothyroidism is relatively common. Affected babies may have coarse facies, dry skin and
a hoarse cry. Other features include hypotonia, umbilical hernia and constipation and prolonged jaundice.
Blood tests reveal low T4 levels and a high TSH. In some infants brain development may be irreversibly
damaged before birth however early detection may avoid permanent neurology sequelae.
3-Hypoparathyroidism – in the neonatal period transient hypoparathyroidism occurs resulting in
hypocalcaemia, possibly convulsions or apnoeic episodes. Hypoparathyroidism is rare. Features include
headaches, vomiting, photophobia, cataracts, poor dentition and chronic diarrhoea. Investigations will
1/
X
X
Pediatric extended match questions Dr samed alsalmi
confirm low calcium and high phosphate levels.
Theme : Endocrine
A. Addison's disease
B. Congenital adrenal hyperplasia
C. Cushing's syndrome
D. Diabetes insipidus
E. Diabetes mellitus
F. Hyperthyroidism
G. Hypoparathyroidism
H. Hypothyroidism
I. Primary aldosteronism
J. Septo-optic dysplasia
Select the most likely diagnosis from the list above that would explain the presentations of the following
patients:
1) A 14 year old girl presents with weight loss.
She denies any dieting or use of laxatives. She
also complains of polydipsia and has recently
started wetting the bed.
E. Diabetes mellitus
2) A 6 year old boy presents with fatigue and
weight loss. He complains of abdominal pain
and disturbed bowel habit. He is found to have
a low blood sugar.
A. Addison's disease
3) A 6 month old baby with histiocytosis
presents with vomiting weight loss and signs of
dehydration.
D. Diabetes insipidus
Comments:
1-Diabetes mellitus results from insulin deficiency. Symptoms are characteristic and early features being
polyuria, nocturnal enuresis, thirst, lethargy, weight loss and anorexia. Late symptoms include vomiting,
abdominal pain and shock.
2-Addison's disease or adrenocortical failure has an auto-immune basis and may occur in association
with other endocrine problems such as diabetes, hypoparathyroidism and thyroiditis. Clinical features
include weakness, weight loss and increased pigmentation. Children may also have hypotension and
hypoglycaemia.
3-Diabetes insipidus may be idiopathic or as a result of tumours, histiocytosis etc. It is due to a lack of
anti-diuretic hormone and results in polyuria, polydipsia hypothermia, weight loss, constipation and poor
growth. Treatment requires DDAVP.
Pediatric extended match questions Dr samed alsalmi
Hematology 1
Hematology
Theme : Haematological Disorders
A. Aplastic anaemia
B. Beta Thalassaemia Major
C. Elliptocytosis
D. Glucose 6 Phosphate dehydrogenase deficiency
E. Haemochromatosis
F. Haemophilia A
G. Immune thrombocytopenia
H. Polycythaemia
I. Pyruvate Kinase deficiency
J. Sickle Cell anaemia
Select the most appropriate haematological diagnosis from the above
list that best explains the following scenarios.
1) May be a consequence of parvovirus infection. A. Aplastic anaemia
2) May present with a characteristic skin
pigmentation and is secondary to multiple
transfusions.
E. Haemochromatosis
3) Occurs as a result of a congenital defect of the
red cell membrane.
C. Elliptocytosis
Comments:
Aplastic anaemia or bone marrow aplasia results in anaemia, neutropenia and
thrombocytopenia. It may be congenital for example Fanconi's anaemia or
acquired for example as a result of infection with for example Parvovirus
infectio or due to drugs such as Chloramphenical, Sulphonamides or
Chemotherapy. Haemochromatosis occurs when the structure or function of
organs is deranged because of excessive Iron storage. Patients may exhibit
bronzed skin and may develop Hepatitis or diabetes. Treatment requires
chelation. Elliptocytosis is similar to spherocytosis in that the red blood cells
exhibit a defect in the membrane. In Elliptocytosis the red blood cells are oval
and the condition is benign. Haemolysis occurs with some forms of the
disease.
Theme : Malignancy
Pediatric extended match questions Dr samed alsalmi
Hematology 2
A. Acute leukaemia
B. Ewing's Osteosarcoma
C. Lymphoma
D. Melanoma
E. Neuroblastoma
F. Retinoblastoma
G. Sarcoma
H. Thyroid cancer
I. Testicular seminoma
J. Wilm's tumour
Select the most appropriate chocie from thee above list for the following
scenarios.
1) A condition which affects children under
the age of 3, associated with a genetic locus
on Chromosome 13.
F. Retinoblastoma
2) A tumour known that is recognised to
spontaneously regress.
E. Neuroblastoma
3) A jaundiced baby on the postnatal ward is
described as having blueberry muffin spots
on the skin.
A. Acute leukaemia
Comments:
Retinoblastoma is a tumour in the posterior part of the retina. It is associated
with a deletion of the long arm of chromosome 13. The average age of
diagnosis is 8 months for bilateral involvement ands 24 months for unilateral
involvement. It may present with leucocoria, visual loss or a squint. A
Neuroblastoma may arise anywhere where neural crest cells migrate. The
median age of diagnosis is approximately 2. In babies less then 1 year of age
or in the early stages of tumour development spontaneous regression has
been known to occur. Acute lymphoblastic leukaemia is the commonest
malignancy of childhood. Initial symptoms may be non-specific for example
lethargy and irritability. Ultimately there is bone marrow failure which presents
with pallor, thrombocytopenia and neutropenia. Diagnosis is made by seeing
blasts on a peripheral smear There may be anaemia, thrombocytopenia and
the white cell count may be raised or low. Babies born with congenital
leukaemia have lesions over the skin which are described as being similar to
blueberry muffins spots
Theme : Haematological Disorders
A. Aplastic anaemia
Pediatric extended match questions Dr samed alsalmi
Hematology 3
B. Beta Thalassaemia Major
C. Elliptocytosis
D. Glucose 6 Phosphate dehydrogenase deficiency
E. Haemochromatosis
F. Haemophilia A
G. Immune thrombocytopenia
H. Polycythaemia
I. Pyruvate Kinase deficiency
J. Sickle Cell anaemia
Select the most appropriate haematological diagnosis from the above
list that best explains the following scenarios.
1) May be exacerbated by exposure to
anti-malarial therapy
D. Glucose 6 Phosphate
dehydrogenase deficiency
2) Characterised by the overgrowth of
bones of the face.
B. Beta Thalassaemia Major
3) May be an associated feature of
congenital cyanotic heart disease.
H. Polycythaemia
Comments:
Glucose-6-phosphate dehydrogenase deficiency is an episodic haemolytic
anaemia which can be exacerbated by certain drugs, in particular anti-malarial
therapy. There is a racial difference in prevalence. Laboratory findings include
low levels of G6PD and Heinz bodies present in the red blood cells.
Thalassaemia is a group of hypochromic anaemia's due to abnormalities of
globin chain synthesis. Regular blood transfusions are necessary and clinical
features include compensatory hypertrophy of erythropoietic tissue in
medullary and extra medullary spaces. For example liver, spleen and the
marrow in the face which results in characteristic facies. Polycythaemia exists
when red blood cell count, Haemoglobin and haematocrit exceed the upper
limits of normal and may co-exist with chronic oxygen desaturation states, for
example cyanotic cardiovascular and pulmonary disease.
Theme : Bleeding disorders
A. Antithrombin III deficiency
B. Congenital afibrinogenaemia
Pediatric extended match questions Dr samed alsalmi
Hematology 4
C. Haemophilia A
D. Haemophilia B
E. Haemophilia C
F. Idiopathic thrombocytopenic purpura
G. Protein C deficiency
H. Protein S deficiency
I. Vitamin K deficiency
J. Von Willebrands disease
Choose the most appropriate diagnosis from the above list that best
describes the following coagulation abnormalities.
1) An autosomal dominant disorder,
characterised by thrombotic events.
A. Antithrombin III deficiency
Note:
Antithrombin 3 deficiency prolonged in occlusion of blood vessels with platelet plugs
resulting in thrombotic events.
2) Prolonged partial thromboplastin
time, normal Prothrombin time.
factor 8 deficiency
C. Haemophilia A
Note:
Haemophilia A results due to low factor 8 levels. The severity of the illness depends
on the level of activity and patients often present with haemarthrosis.
3) An autosomal dominant disorder
associated with a prolonged
bleeding time.
J. Von Willebrands disease
Note:
describes Von Willebrand's disease an autosomal dominant condition which results in
prolonged bleeding due to low Von Willebrand factor, which is needed for platelet
aggregation. There is a prolonged bleeding time and treatment with FFP is sometimes
required.
Theme : Bleeding disorders
A. Antithrombin III deficiency
B. Congenital afibrinogenaemia
C. Haemophilia A
D. Haemophilia B
Pediatric extended match questions Dr samed alsalmi
Hematology 5
E. Haemophilia C
F. Idiopathic thrombocytopenic purpura
G. Protein C deficiency
H. Protein S deficiency
I. Vitamin K deficiency
J. Von Willebrands disease
Choose the most appropriate diagnosis from the above list that best
describes the following coagulation abnormalities.
1) Normal Prothrombin time.
Elevated thromboplastin time,
factor 9 deficiency.
D. Haemophilia B
Note:
Haemophilia B also known as Christmas disease is an X linked recessive
disorder. It is as a result of low Factor 9 levels.
2) Prolonged Prothrombin time
and partial thromboplastin time,
normal bleeding time.
I. Vitamin K deficiency
Note:
Item 2 describes Vitamin K deficiency which is necessary for carboxylation of
factors 2, 7 ,9 and 10.
3) Bruising and petechiae over
lower extremities with low
platelets.
F. Idiopathic thrombocytopenic purpura
Note:
fits the diagnosis of idiopathic thrombocytopenic purpura. A condition
associated with mucocutaneous bleeding and petechiae often secondary to
viral infections and the platelet count is less than 20 x 10/9. Treatment options
include steroids and gammaglobulin.
Theme : Malignancy
A. Acute leukaemia
B. Ewing's Osteosarcoma
C. Lymphoma
D. Melanoma
E. Neuroblastoma
Pediatric extended match questions Dr samed alsalmi
Hematology 6
F. Retinoblastoma
G. Sarcoma
H. Thyroid cancer
I. Testicular seminoma
J. Wilm's tumour
Select the most appropriate choice from the above list for the following
scenarios
1) A 3 year old with pallor, bruising
and hepatosplenomegaly.
A. Acute leukaemia
2) A 16 year old with shortness of
breath and night sweats and a
cervical lymphnode.
C. Lymphoma
3) A 4 year old known to have a
deletion of Chromosome 11 presents
with an abdominal mass.
J. Wilm's tumour
Comments:
Acute lymphoblastic leukaemia is the commonest malignancy of childhood.
Initial symptoms may be non-specific for example lethargy and irritability.
Ultimately there is bone marrow failure which presents with pallor,
thrombocytopenia and neutropenia. Diagnosis is made by seeing blasts on a
peripheral smear There may be anaemia, thrombocytopenia and the white cell
count may be raised or low. Babies born with congenital leukaemia have
lesions over the skin which are described as being similar to blueberry muffins
spots. Lymphoma is divided in to Hodgkin's and Non-Hodgkin's lymphoma. It
arises in lymph nodes and enlarged nodes may be firm and non-tender.
Mediastinal involvement may cause a chronic cough or bronchial / tracheal
compression. Symptoms include night sweats, persistent fever and weight
loss . Wilm's tumour is a solitary growth in a part of the kidney. It is associated
with deletions of chromosome 11 (the probable location of the tumour
suppression gene). It may present with an abdominal mass or haematuria. It
is also associated with genital urinary anomalies, aniridia, hemihypertrophy
and Beckwith-Wiedemann syndrome.
Theme : Diagnosis of neonatal jaundice.
A. ABO incompatibilty
B. Breast milk jaundice
C. Cephalhematoma
D. Drug allergy
E. Extrahepatic biliary atresia
F. Galactosaemia
G. G6PD Deficiency
H. Neonatal sepsis
I. Physiological jaundice
J. RH Incompatibility
Pediatric extended match questions Dr samed alsalmi
Hematology 7
For each of these jaundiced babies below choose from the list above the single most
likely diagnosis. Each option may be chosen more than once or not at all.
1) A mother has blood group A Rhesus positive. Her
baby is blood group B Rhesus negative.
A. ABO incompatibilty
Note:
This mother would produce antibodies to the baby's B blood group and hence haemolysis with
jaundice occurs.
2) A neonate is severely jaundiced with reducing
substances noted on urine dipstick.
F. Galactosaemia
Note:
A typical picture of galactossaemia. In the newborn period, infants present with an acute
encephalopathy. In untreated patients, there is severe liver disease, mental retardation,
epilepsy and choreoathetosis.
3) A 4 day old baby is well but has a tinge of jaundice. I. Physiological jaundice
Note:
Common in the first week of life.
4) A breast fed 3 week old baby has mild jaundice but
is gaining weight satisfactorily.
B. Breast milk jaundice
Note:
Common from the 2nd to 5th days of life.
5) A 12 day old baby is jaundiced with pale stools. E. Extrahepatic biliary
atresia
Note:
Biliary atresia. is a progressive inflammatory process that begins very soon after birth. On
average, there is one case of biliary atresia out of every 15,000 live births. Females are
affected slightly more often than males. In the United States, approximately 300 new cases are
diagnosed each year.
Pediatric extended match questions Dr samed alsalmi
Infectious Diseases
2) Contra-indications to Vaccines
A. BCG
B. Diphtheria Tetanus Polio
C. Hepatitis B
D. Haemophilus Influenzae B
E. Influenza
F. Measles Mumps Rubella
G. Polio
H. Rubella
I. Tetanus
J. Tuberculin
Which of the vaccines listed above would be contra-indicated in the following scenarios:
1) DIARRHOEA
A. Coeliac disease
B. Crohns disease
C. Ulcerative colitis
D. Cows milk protein intolerance
E. Toddlers diarrhoea
F. Cystic fibrosis
G. Lactose intolerance
H. Irritable bowel syndrome
I. Abdominal migraine
J. Haemolytic uraemic syndrome
For the following children presenting with diarrhoea which is the most likely diagnosis:
1) An 11 month infant presents with chronic diarrhoea and failure to thrive since breast feeding was
discontinued at 9 months. He has been treated by his GP with antibiotics on two occasions for chest
infections. Initial investigations reveal low serum potassium, chloride and metabolic alkalosis.
F. Cystic fibrosis
Note:
Failure to thrive, chest infections and diarrhoea (the latter results from failure of pancreatic exocrine
function) are indicative of CF. The abnormal biochemistry is caused by salt loss and may be described as
pseudo-bartters syndrome
2) A 15 year old boy presents with delayed puberty and short stature. He gives a history of intermittent
abdominal pain and diarrhoea. He recently presented to his family doctor with a painful red swelling on
his shin which has now resolved.
B. Crohns disease
Note:
Crohn’s disease may present in this way, the skin lesions are erythema nodosum which are associated
with inflammatory bowel disease.
3) A 8 month old infant has recurrent diarrhoea. Mother dates the start of symptoms from the an episode
of gastroenteritis. The diarrhoea is explosive in nature.
G. Lactose intolerance
Note:
Secondary lactose intolerance is a known sequelae of viral gastroenteritis. It is not prevented by
regrading milk following the episode of viral infection.
Pediatric extended match questions Dr samed alsalmi
1) Contra-indication if known to have a hypersensitive reaction to egg.
E. Influenza
Note:
relates to hypersensitivity to egg, which contra-indicates the Influenza vaccine as there is residual egg
protein present.
2) Contra-indication if known to have an allergy to gelatin.
F. Measles Mumps Rubella
Note:
refers to contra-indications to MMR, which include children with allergies to Gelatin, Neomycin or
kanamycin as well as children with untreated malignant disease or altered immunity. Those receiving
immuno-suppressive drugs or radiotherapy are also contraindicated. Children who have received another
live vaccine by injection within 3 weeks should not be given MMR vaccine. MMR should also not be given
within 3 months of receiving an Immunoglobulin injection.
3) Contra-indication if known to have an allergy to kanamycin.
F. Measles Mumps Rubella
Note:
refers to contra-indications to MMR, which include children with allergies to Gelatin, Neomycin or
kanamycin as well as children with untreated malignant disease or altered immunity. Those receiving
immuno-suppressive drugs or radiotherapy are also contraindicated. Children who have received another
live vaccine by injection within 3 weeks should not be given MMR vaccine. MMR should also not be given
within 3 months of receiving an Immunoglobulin injection.
3) Childhood chest infections
A. Chlamydia
B. Cytomegalovirus
C. Group B Streptococcus
D. Haemophilus Influenza
E. Mycoplasma pneumonia
F. Parainfluenza
G. Respiratory syncytial virus
H. Rhinovirus
I. Staphylococcus aureus
J. Ureaplasma
Select the most appropriate pathogen from the above list that would account for the following
presentations:
1) The commonest cause of pneumonia in the 2 month - 6month age group.
G. Respiratory syncytial virus
2) A 9 year old boy presents with a cough and fever. Chest x-ray demonstrates a reticular nodular
pattern with high adenopathy and a small pleural effusion.
E. Mycoplasma pneumonia
3) A 5 year old boy presents with painful red swellings of his shins. Examination of his throat reveals an
exudative tonsillitis.
Pediatric extended match questions Dr samed alsalmi
C. Group B Streptococcus
Comments:
The commonest cause of pneumonia in the 2month – 6 month age group is Respiratory syncytial virus,
which results in bronchiolitis and occurs in epidemics. Mycoplasma pneumonia mainly affects older
children (between 5 and 15 years) resulting in community acquired pneumonias. Tonsillitis can occur at
any age although is frequently seen in the 4-7 age group. The majority of episodes are caused by viruses
although beta haemolytic streptococcal infection is also responsible and has systemic reactions such as
erythema nodosum, rheumatic fever and glomerular nephritis.
4) Childhood Diarrhoea
A. Adenovirus
B. Cytomegalovirus
C. Epstein-Barr virus
D. E. Coli
E. Giardia Lamblia
F. Hepatitis A
G. Norwalk virus
H. Rotavirus
I. Salmonella species
J. Staphylococcus aureus
Select one option from the list above that is most suitable for the following patients
1) Accepted as the most common cause of infectious diarrhoea in children in the developed society.
H. Rotavirus
Note:
Rotavirus is the most common virus responsible for diarrhoea worldwide. It is a double stranded RNA
virus, especially prevalent in the winter months. It causes fever and watery diarrhoea.
2) Can follow ingestion of dust containing dried faecal material.
F. Hepatitis A
Note:
Hepatitis A is an RNA virus typically transmitted by the faecal-oral route.
3) Attaches the small intestinal border and releases extoxin.
D. E. Coli
Note:
E-coli and enteropathogen is one of the bacterial causes of diarrhoea. Many of its effects are a
consequence of an exotoxin. It should be suspected if there is a history of travel or a history of ingestion
of poorly prepared food. Bacterial diarrhoea usually results in bloody diarrhoea.
Pediatric extended match questions Dr samed alsalmi
5) Urinary Tract Disease
A. Haemolytic uraemic syndrome
B. Henoch Schonlein purpura
C. Nephrotic syndrome
D. Polycystic kidneys
E. Pyelonephritis
F. Renal Calculi
G. Renal tubular acidosis
H. Renal vein thrombosis
I. Systemic lupus erythematosus
J. Wilm's tumour
Select the most appropriate diagnosis from the above list of options that would best explain the
following cases:
1) A 2 year old boy is admitted with a history of bloody diarrhoea, abdominal pain and puffiness around
his eyes. He is found to be slightly jaundiced. His urea is 12.6 and Creatinine 163 mmol / liter.
A. Haemolytic uraemic syndrome
Note:
describes a child with haemolytic uraemic syndrome, which is of unknown aetiology. Various agents have
been implicated including E.coli (0157) Salmonella, Shigella and viruses. It usually presents with a
prodromal symptoms of an upper respiratory tract infection or gastroenteritis and later develops in to
pallor, oliguria and hypertension. Laboratory investigations may reveal microangiopathic haemolytic
anaemia, thrombocytopenia and urinalysis may reveal proteinuria.
2) A 9 day old baby is admitted with a 2 day history of vomiting and diarrhoea. During the admission
haematuria is noted and he has had 2 convulsions. A mass is palpable in the left hypochondrium.
H. Renal vein thrombosis
Note:
describes a child with renal vein thrombosis. In neonates it is associated with dehydration, asphyxia,
shock or sepsis. Manifestations include sudden onset of haematuria and identification of an enlarged
kidney. If both kidneys are involved it may result in acute renal failure.
3) A 5 year old girl presents with puffiness around the eyes. Urinalysis confirms proteinuria with a trace of
blood.
C. Nephrotic syndrome
Note:
relates to a child with Nephrotic syndrome. This occurs when there is proteinuria resulting in
hypoalbuminaemia and oedema with an unknown aetiology. Peri-orbital or dependant oedema is usually
noted first. There may also be a history of abdominal pain, vomiting and diarrhoea. The child needs to be
monitored carefully as hypovolaemia and circulatory collapse. Steroids are the treatment of choice with
careful management of fluids.
6) NEONATAL JAUNDICE
A. Physiological jaundice
B. Biliary atresia
C. Hypothyroidism
D. Rhesus incompatibility
E. Congenital spherocytosis
F. Congenital cytomegalo virus infection
G. Galactosaemia
H. Glucose-6-phosphate dehydrogenase deficiency
I. Cystic fibrosis
J. Fructose intolerance
Pediatric extended match questions Dr samed alsalmi
For each of the following jaundiced babies, select the most likely cause:
1) A 13 day old infant who was noted to have an umbilical hernia and has very dry skin presents with
jaundice. She is a floppy baby. Her bilirubin is checked and is found to be elevated and mainly
unconjugated. The community midwife has been unable to gain access to the home for the last week.
C. Hypothyroidism
Note:
Umbilical hernia, dry skin, hypotonia and jaundice are features of congenital hypothyroidism.
2) A caucasian infant has required surgery at the regional neonatal unit for meconium ileus and has
developed jaundice. The serum conjugated bilirubin is 65 micromol/L. The diagnosis is eventually
confirmed from the neonatal screening tests.
I. Cystic fibrosis
Note:
Meconium ileus and jaundice may be presenting features of CF in the neonatal period. The screening
test is immunoreactive trypsin.
3) A 12 day old male baby has a conjugated and unconjugated hyperbilirubinaemia. He is breast fed and
has become more disinterested in feeds. He was investigated for sepsis and blood cultures
demonstrated E-coli septicaemia, urine cultures were clear. His clotting is deranged.
G. Galactosaemia
Note:
Galactosaemia typically presents around 2 weeks of age with jaundice. E coli septicaemia is a feature.
Treatment is by removing galactose from the diet. Cataracts are a later feature even if treatment
instituted early.
7) Viral infections
A. Measles
B. Rubella
C. Chicken Pox
D. Herpes Simplex
E. Mumps
F. Glandular fever
G. Pertussis
H. Polio myelitis
I. Hepatitis A
For each description below choose the single most likely viral infection from the list of options.
1) May result in Giant cell pneumonia.
A. Measles
Note:
Measles infection is uncommon due to world-wide immunisation. A child with measles usually has a
fever, upper respiratory tract symptoms and a morbilliform rash. Serious complications include a giant
cell pneumonia and encephalitis (SSPE).
2) May result in a Keratoconjuctivitis.
D. Herpes Simplex
Pediatric extended match questions Dr samed alsalmi
Note:
Herpes simplex infection may be transmitted verdantly to an infant from their mother's genital tract. It may
cause isolated skin lesions, a Keratoconjunctivitis or a paronychia. More seriously it is also responsible
for encephalitis.
3) Caused by an RNA virus with no known carrier state.
I. Hepatitis A
Note:
Hepatitis A, which is caused by an RNA virus, is usually transmitted by the oral route. It has an incubation
period of between 15-50 days and treatment is usually symptomatic only.
8) Urinary Tract Disease
A. Haemolytic uraemic syndrome
B. Henoch Schonlein purpura
C. Nephrotic syndrome
D. Polycystic kidneys
E. Pyelonephritis
F. Renal Calculi
G. Renal tubular acidosis
H. Renal vein thrombosis
I. Systemic lupus erythematosus
J. Wilm's tumour
Select the most appropriate diagnosis from the above list of options that would best explain the
following cases:
1) An 11 year old with a previous history of chronic glomerulonephritis presents with bruising and
epistaxis. A full blood count confirms a pancytopenia.
I. Systemic lupus erythematosus
Note:
describes a child with glomerulonephritis and bone marrow failure suggestive of a connective tissue
disease such as SLE.
2) A 14 year old boy with a history of recurrent urinary tract infections present with severe abdominal pain
radiating to his back. Dysuria and haematuria.
F. Renal Calculi
Note:
relates to Urolithiasis, Renal Calculi. Children present with abdominal pain, voiding abnormalities,
dysuria, haematuria may be present. Avoidance of dehydration is important. Treatment may require
lithotripsy.
3) A 1 year old girl with a 3 month history of vomiting is investigated for failure to thrive. She is found to
be mildly acidotic.
G. Renal tubular acidosis
Note:
Renal tubular acidosis is the answer to Item 3. It is a clinical state of systemic hyperchloraemic acidosis
resulting from impaired urinary acidification. Three types exist. Type 1 distal RTA, type 2 proximal RTA
and type 4 mineralocorticoid deficiency. Type 3 is thought to be a variant of type 1. Types 1, 2 and 4
each have several causes. Children with isolated forms of proximal or distal commonly present with
Pediatric extended match questions Dr samed alsalmi
growth failure; gastrointestinal symptoms are also common. Nephro-calcinosis and hypercalciuria may
complicate distal RTA and goals of treatment are to correct the acidosis and to maintain normal
Bicarbonate and Potassium levels.
9) Mucocutaneous infections and infestations
A. Enterobiasis vermicularis
B. Chlamydia
C. Candidiasis
D. Tinea pedis
E. Pityriasis rosea
F. Scabies
G. Ringworm
H. Lichen sclerosis
I. Napkin dermatitis
J. Lichen planus
For each of the following children who present with an itch, select the most appropriate diagnosis
from the list of options:
1) A child complains of an itchy rash over his arms. His brother and sister have similar symptoms.
F. Scabies
Note:
Scabies is caused by the mite Sarcoptes scabiei hominis. Transmission is through close body contact,
the adult mites lay their eggs in burrows in the skin and it results in an eczematous rash with parotitis
skin.
2) A baby has an excoriated perineal rash involving the flexures.
C. Candidiasis
Note:
Candidiasis is caused by Candida albicans (yeast). In babies it presents as a perineal rash which usually
affects the flexures. Satellite lesions may also be seen.
3) A boy has an itchy rash over the trunk with a solitary large oval lesion on the back.
E. Pityriasis rosea
Note:
Pityriasis rosea is a benign condition of the skin resulting in oval pink / brown scaly lesions over the trunk,
which are usually preceded by a herald patch (a solitary large lesion usually between 1 –10cm). No
treatment is required.
10) Childhood respiratory diseases
A. Asthma
B. Bronchiolitis
C. Croup
D. Cystic fibrosis
E. Diphtheria
F. Epiglottis
G. Pneumonia
H. Influenza
I. Retropharyngeal abscess
J. Whooping cough
For each patient below, choose the SINGLE most probable diagnosis from the above list of
Pediatric extended match questions Dr samed alsalmi
options. Each option may be used once, more than once or not at all.
1) A six month old baby presents with high fever, breathlessness, cough and feeding difficulties. Chest
examination reveals dull percussion note over the right base posteriorly with bronchial breath sounds on
auscultation.
G. Pneumonia
Note:
Pneumonia may occur at any age. Patients present with fever, tachypnoea, feeding difficulties and
cyanosis. Examination reveals bronchial breath sounds and crepitations. Chest X ray may show
consolidation. Common organisms are pneumococcus, haemophilus, staphylococcus, mycoplasma, TB
and viruses.
2) A 1 year old baby boy is wheezy, coughing, cyanosed and breathless with intercostal recession.
B. Bronchiolitis
Note:
Acute bronchiolitis is very common in infancy. In winter epidemics of respiratory syncytial virus infection
are the commonest cause. Wheeze, cough, fever and respiratory distress are common. Chest X ray
shows hyperinflation.
3) A 4 year old non immunized boy presents with bouts of coughing ending in vomiting. He has an
absolute lymphocytosis.
J. Whooping cough
Note:
Whooping cough is caused by Bordetella pertussis infection. Bouts of coughing ending in vomiting,
especially at night and after feeding suggest the diagnosis. The characteristic whoop, forced inspiration
through a closed glottis may or may not be present. Absolute lymphocytosis is common. Complications
include CNS haemorrhages, rectal prolapse and bronchiectasis.
4) A nine month old baby girl is upset and has stridor. Her voice is hoarse and has a barking cough. She
has a low grade fever.
C. Croup
Note:
Croup usually occurs in epidemics in autumn or spring. Causative viruses are Parainfluenza (types 1, 2,
or 3), respiratory syncytial viruses and measles virus. Onset is over a few days; stridor is harsh and
occurs only when child is upset. A barking cough, harsh voice and ability to swallow secretions are
typical.
5) A 2 year old boy is very unwell. His temperature is 39°C and he is unable to swallow his secretions.
F. Epiglottis
Note:
Acute epiglottis is due to Haemophilus influenza type B infection. It is characterised by sudden onset,
high fever, continuous stridor and drooling of secretions. Intravenous antibiotics, anaesthetic support
are usually indicated.
Pediatric extended match questions Dr samed alsalmi
11) Treatment of infectious disease
A. Acyclovir
B. Acyclovir plus Cefotaxime
C. Ampicillin plus Gentamicin
D. Benzylpenicillin
E. Ceftriaxone
F. Ciprofloxacin
G. Flucloxacillin
H. Netilmicin
I. Symptomatic treatment
J. Teicoplanin
For each case of infectious disease described below choose the single best treatment option
from the list.
1) A 4 year old with a 3 day history of vomiting and diarrhoea.
I. Symptomatic treatment
Note:
A child with a history of gastroenteritis is a very common childhood illness. Conservative treatment only is
required.
2) A 2 year old child with an immune deficiency develops Chicken Pox.
A. Acyclovir
Note:
A child who is immuno-deficient and therefore must be provided with Zoster Immune Globulin if exposed
to Chicken pox through contact. If the Chicken pox develops, treatment with Acyclovir is required.
3) A 7 year old child with widespread impetigo.
G. Flu-cloxacillin
Note:
Impetigo is a common staphylococcal skin manifestation, which is highly contagious. Most frequent sites
being nostrils and the peri-oral area. Flucloxacillin is the treatment of choice.
12) Exanthem
A. Staphylococcal scalded skin syndrome
B. Rubella
C. Measles
D. Kawasaki’s disease
E. Impetigo
F. Scarlatina
G. Infectious mononucleosis
H. Henoch – Schonlein purpura
I. Meningococcal infection
J. Stills disease (systemic onset juvenile chronic arthritis)
Match the following descriptions of rash with the illness for which they are the most typical
exanthem
1) A 5 year old boy has a bright, red, punctate, erythematous rash which blanches on pressure,
beginning in the axillae with some perioral pallor and relative facial sparing. The skin feels like
“sandpaper”. The rash fades and desquamates on the hands and feet. A thick white exudate develops on
the tongue which peels leaving a “strawberry tongue” with prominent papillae.
F. Scarlatina(scarlet fever)
Pediatric extended match questions Dr samed alsalmi
Note:
This description is typical of Scarlatina i.e. beta haemolytic streptococcal infection. The rash may be
confused with that of Kawasaki disease. However, desquamation occurs in late in Kawasaki’s disease.
2) A maculopapular rash develops in a child with sore throat and fever who has been treated with
ampicillin.
G. Infectious mononucleosis
Note:
This is a known effect of giving ampicillin during EBV infection.
3) A 3 year old child presents with high fever for 7 days, conjunctival injection, fissuring of the lips and
strawberry tongue, erythema followed by desquamation of the hands and feet, and a macular rash over
the trunk with cervical lymphadenopathy.
D. Kawasaki’s disease
Note:
There are major and minor features of Kawasaki disease. High fever and desquamation are typical.
13) Presentation of infectious disease in childhood
A. Chicken Pox
B. Herpes simplex
C. Infectious mononucleosis
D. Measles
E. Mumps
F. Mycoplasma
G. Pertussis
H. Rubella
I. Tuberculosis
For each presentation of infectious disease choose the single most likely diagnosis from the list
of options.
1) May be complicated by cerebellar ataxia.
A. Chicken Pox
Note:
Chicken pox is a common childhood illness caused by Varicella. Vesicles usually appear as crops over
the trunk. Viral cerebellitis is a complication.
2) May present with apnoeas during infancy.
G. Pertussis
Note:
Pertussis is caused by Bordetella pertussis. In infancy it can be dangerous often presenting with
apnoeas. Signs of an upper respiratory tract infection are also common with a paroxysmal cough
(whoop). Babies may also present with vomiting and cyanosis. A blood count will reveal a lymphocytosis.
Complications include broncho-pneumonia and bronchiectasis as sequelae is also associated though
uncommon.
3) Associated with a maculo-papular rash that typically starts on face and extends to rest of body
H. Rubella
Pediatric extended match questions Dr samed alsalmi
Note:
Rubella infection is uncommon now due to widespread MMR vaccination. Typically causes a mild illness
which may go undetected in 25-50% of cases. Typical symptoms are transient macular papular rash that
starts on the face and extends to body and a low grade fever. Complications include arthritis, encephalitis
and thrombocytopenia.
14) RASHES
A. Staphylococcal scalded skin syndrome
B. Rubella
C. Measles
D. Kawasaki’s disease
E. Impetigo
F. Scarlatina
G. Infectious mononucleosis
H. Henoch – Schonlein purpura
I. Meningococcal infection
J. Stills disease (systemic onset juvenile chronic arthritis)
Match the following descriptions of rash with the illness for which they are the most typical
exanthem.
1) A salmon-coloured, reticulate macular rash develops mainly over the extensor surfaces of the limbs in
a 5 year old boy with swinging temperature; hot, swollen, painful knees and left elbow and palpable
spleen. The ESR is 95. The blood count, C-reactive protein and chest X-ray are normal.
J. Stills disease (systemic onset juvenile chronic arthritis)
Note:
“Salmon-coloured” is the description used to describe the rash of Still’s disease. The distribution is not
that of HSP which covers typically the buttocks and limbs and is a purpuric rash.
2) A 12 year old boy develops petechiae and papules, some of which become purpuric over his buttocks
and legs, associated with painful swollen knees. There is microscopic haematuria on testing. The platelet
count is normal.
H. Henoch – Schonlein purpura
Note:
This is a description of HSP and the well recognised complication of HSP nephritis. A proportion of these
patients will develop progressive nephritis and end stage renal failure.
3) A 5 day old girl has a high temperature and is irritable. She has areas of desquamation over her finger
tips and in the axillae. Her carer notices that her skin blisters easily following minimal contact.
A. Staphylococcal scalded skin syndrome
Note:
SSSS results from infection with staphylococci with the exofoliative toxin A and B. These exotoxins cause
disruption to the epidermal layer by interfering with intercellular junctions. Mortality is up to 3% in
children. A positive Nikolsky sign (slippage of the superficial layer of the epithelium on gentle pressure) The
desquamation occurs concomitantly with the illness unlike Kawaskis disease and Kawasaki disease does
not occur in this age group. There may be a history of minimal skin trauma which provides a port of entry
for the organism.
Pediatric extended match questions Dr samed alsalmi
15) DIARRHOEA
A. Coeliac disease
B. Crohns disease
C. Ulcerative colitis
D. Cows milk protein intolerance
E. Toddlers diarrhoea
F. Cystic fibrosis
G. Lactose intolerance
H. Irritable bowel syndrome
I. Abdominal migraine
J. Haemolytic uraemic syndrome
For the following children presenting with diarrhoea which is the most likely diagnosis:
1) A 6 year old boy with Downs’s syndrome has between 3 and 4 loose stools a day. He is pale but
otherwise the examination is unremarkable. When plotting his growth on a chart specific for Downs
syndrome children it is clear that both height and weight have fallen across 2 centiles.
A. Coeliac disease
Note:
Coeliac disease is more occurs more frequently in Downs’s syndrome. To screen, anti-TTG antibodies
taken with serum IgA must be evaluated.
2) A 3 year old has intermittent diarrhoea with stools of varying consistency and sometimes undigested
food particles. His growth is satisfactory. His mother had tried to reduce his intake of diary produce but
dietary manipulation has had no impact on the symptoms.
E. Toddlers diarrhoea
Note:
Toddler diarrhoea (thought to be a normal variant) occurs mainly in preschool children and is evidenced
by undigested food observed in the faeces in a well child. Poor growth would indicate a likely pathological
cause.
3) A 4 month baby girl has severe atopic eczema and is regularly reviewed in the paediatric clinic. She
had now developed diarrhoea and her weight has been static in recent weeks. A full blood count shows a
slight eosinophilia.
D. Cows milk protein intolerance
Note:
CMPI is associated with eczema and eosinophilia. Cow’s milk should be excluded from the diet and
reintroduced between 1-2 years if tolerated. An alternative milk source such as soy milk can be used until
this is done.
Pediatric extended match questions Dr samed alsalmi
16) Childhood infections
A. Adenovirus
B. Cytomegalovirus
C. Epstein-Barr virus
D. Escherichia coli
E. Giardia lamblia
F. Haemophilus influenzae type B
G. Listeria monocytogenes
H. Mycobacterium tuberculosis
I. Mycoplasma pneumoniae
J. Neisseria meningitidis
K. Pseudomonas aeruginosa
L. Pneumocystis carinii
M. Respiratory syncytial virus
N. Rotavirus
O. Staphylococcus aureus
P. Staphylococcus epIdermidis
Q. Streptococcus agalactiae
R. Streptococcus pneumoniae
S. Streptococcus pyogenes
T. Toxoplasma gondii
All of the infectious diseases described below occur in children. For each one, select the most
likely underlying causative agent from the list of options
1) A 3 year old boy is referred to hospital with a two day history of lethargy, irritability and poor feeding.
On examination, he is pyrexial, drowsy and has a purpuric rash on his trunk and extremities. CSF
obtained from a lumbar puncture is cloudy and contains 540 white cells/mm
3
(90% polymorphs) and 5
red blood cells/mm
3
.
J. Neisseria meningitidis
2) A 6 year old girl presents with a one week history of febrile illness with sore throat and headache.
One day prior to hospital admission, the patient awoke with pain and swelling in the right ankle. On
examination, she has a warm swollen right ankle and a systolic heart murmur, consistent with mitral
regurgitation.
S. Streptococcus pyogenes
3) A new born infant is found to be lethargic and has a distended abdomen immediately after birth. On
examination, the infant is jaundiced and has hepatosplenomegaly. A cranial CT scan reveals
periventricular calcification.
B. Cytomegalovirus
4) A 7 year old male child is referred to hospital by the general practitioner with acute renal failure. The
child had bloody diarrhoea and a low grade fever a week ago; both resolved with rehydration.
D. Escherichia coli
5) A 4 month old female infant is brought to the hospital with severe respiratory distress. Five days
previously, she had a cough and rhinitis. On examination her temperature is 38.9
o
C, pulse 180/min and
the respiratory rate 80/min. She had subcostal retractions and nasal flaring. On auscultation, there are
rhonchi and wheezes all over her chest.
M. Respiratory syncytial virus
Comments:
1. The diagnosis of Meningococcal meningitis is clear. 2. Rheumatic fever results from immune-mediated
post Group A streptococcal infection, Streptococcus pyogenes being a common pathogen. 3. Congenital
CMV infection is associated with petechiae, choroidoretinitis, hepatosplenomegaly, intracerebral
calcification which may lead to CNS damage with long term sequaelae. 4. E. coli infection has resulted in
Haemolytic Uraemic Syndrome with renal failure. 5. Respiratory Syncytial Virus is the commonest cause
of lower respiratory tract infections in children worldwide, and is the leading cause of bronchiolitis and
pneumonia in children.
Pediatric extended match questions Dr samed alsalmi
17) Viral infections
A. Measles
B. Rubella
C. Chicken Pox
D. Herpes Simplex
E. Mumps
F. Glandular fever
G. Pertussis
H. Polio myelitis
I. Hepatitis A
For each description below choose the single most likely viral infection from the list of options
1) Caused by a gram negative pleomorphic bacillus
G. Pertussis
2) Is caused by a paramyxovirus.
E. Mumps
3) This infection is often followed by a transient immuno-deficiency.
F. Glandular fever
Comments:
Whooping cough is not uncommon in infancy. It typically presents with apnoeic episodes or cyanotic
episodes during infancy. In the older child upper respiratory tract infections and a paroxysmal cough with
a whoop is characteristic. Mumps is caused by a paramyxovirus. Glandular fever virus infects the B
lymphocytes which results in an immuno-deficiency which is usually self limiting.
18) Childhood Diarrhoea
A. Adenovirus
B. Cytomegalovirus
C. Epstein-Barr virus
D. E. Coli
E. Giardia Lamblia
F. Hepatitis A
G. Norwalk virus
H. Rotavirus
I. Salmonella species
J. Staphylococcus aureus
Select one option from the list above that is most suitable for the following patients
1) Causes diarrhoea by invading the brush border of the small intestine and causes vacuolation.
I. Salmonella species
Note:
Salmonella may contaminate foods improperly foods cooked or stored and invades the brush border of
the small intestine. Symptoms include nausea, vomiting and diarrhoea approximately between 1-8 hours
after ingestion.
2) Spread is due to personal contact as well as by contaminated water supply.
E. Giardia Lamblia
Note:
Pediatric extended match questions Dr samed alsalmi
Giardia Lamblia is a protozoa and may contaminate water or be transmitted by the faecal or oral route or
person to person contact. Children are often asymptomatic however diarrhoea and cramps and weight
loss may be features of the disease.
3) An organism which is not only associated with gastroenteritis but is also the most common cause of
osteomyelitis and arthritis in children.
J. Staphylococcus aureus
Note:
Staphylococcus aureus is a gram positive coccus and the most common cause of osteo-myelitis and
arthritis in children. It may contaminate skin resulting in impetigo, cellulitis, folliculitis and furunculosis. It
may cause pneumonias and may contaminate food resulting in enterotoxins being released in to the
intestinal tract.
19) Contra-indications to Vaccines
A. BCG
B. Diphtheria Tetanus Pertussis
C. Hepatitis B
D. Haemophilus Influenzae B
E. Influenza
F. Measles Mumps Rubella
G. Polio
H. Rubella
I. Tetanus
J. Tuberculin
Which of the vaccines listed above would be contra-indicated in the following scenarios:
1) Contra-indicated if known to be allergic to Neomycin.
F. Measles Mumps Rubella
Note:
refers to contra-indications to MMR, which include children with allergies to Gelatin, Neomycin or
kanamycin as well as children with untreated malignant disease or altered immunity. Those receiving
immuno-suppressive drugs or radiotherapy. Children who have received another live vaccine by injection
within 3 weeks and children should not be given MMR within 3 months of an Immunoglobulin injection.
2) Contra-indicated in HIV positive patients.
A. BCG
Note:
relates to vaccines in HIV infection. The department of health has advised that HIV positive subjects with
or with out symptoms should not receive BCG, yellow fever or typhoid vaccinations.
3) Contra-indicated in subjects with progressive neurological conditions.
B. Diphtheria Tetanus Pertussis
Note:
relates to DTP (and the pertussis component) which should not be given to subjects with any progressive
neurological disorder particularly epilepsy and immunisation should be delayed until the condition is
stable.
Pediatric extended match questions Dr samed alsalmi
20) CHILDHOOD RESPIRATORY DISEASES
A. Asthma
B. Bronchiolitis
C. Croup
D. Cystic fibrosis
E. Diphtheria
F. Epiglottis
G. Pneumonia
H. Influenza
I. Retropharyngeal abscess
J. Whooping cough
For each patient below, choose the SINGLE most probable diagnosis from the above list of
options. Each option may be used once, more than once or not at all.
1) A six month old baby presents with high fever, breathlessness, cough and feeding difficulties. Chest
examination reveals dull percussion note over the right base posteriorly with bronchial breath sounds on
auscultation.
G. Pneumonia
Note:
Pneumonia may occur at any age. Patients present with fever, tachypnoea, feeding difficulties and
cyanosis. Examination reveals bronchial breath sounds and crepitations. Chest X ray may show
consolidation. Common organisms are pneumococcus, haemophilus, staphylococcus, mycoplasma, TB
and viruses.
2) A 1 year old baby boy is wheezy, coughing, cyanosed and breathless with intercostal recession.
B. Bronchiolitis
Note:
Acute bronchiolitis is very common in infancy. In winter epidemics of respiratory syncytial virus infection
are the commonest cause. Wheeze, cough, fever and respiratory distress are common. Chest X ray
shows hyperinflation.
3) A 4 year old non immunized boy presents with bouts of coughing ending in vomiting. He has an
absolute lymphocytosis.
J. Whooping cough
Note:
Whooping cough is caused by Bordetella pertussis infection. bouts of coughing ending in vomiting,
especially at night and after feeding suggest the diagnosis. The characteristic whoop, forced inspiration
through a closed glottis may or may not be present. Absolute lymphocytosis is common. Complications
include CNS haemorrhages, rectal prolapse and bronchiectasis.
4) A nine month old baby girl is upset and has stridor. Her voice is hoarse and has a barking cough. She
has a low grade fever.
C. Croup
Note:
Croup usually occurs in epidemics in autumn or spring. Causative viruses are Parainfluenza (types 1, 2,
or 3), respiratory syncytial viruses and measles virus. Onset is over a few days; stridor is harsh and
occurs only when child is upset. A barking cough, harsh voice and ability to swallow secretions are
typical.
5) A 2 year old boy is very unwell. His temperature is 39oC and he is unable to swallow his secretions.
F. Epiglottis
Note:
Acute epiglottis is due to Haemophilus influenza type B infection. It is characterised by sudden onset,
high fever, and continuous stridor and drooling of secretions. Intravenous antibiotics, anaesthetic support
are usually indicated.
Pediatric extended match questions Dr samed alsalmi
21) CONGENITAL AND NEONATAL DEFECTS DUE TO MATERNAL INFECTIONS
A. AIDS
B. Cytomegalovirus (CMV)
C. Coxsackie group B
D. Hepatitis B
E. Herpes simplex
F. Listeriosis
G. Rubella
H. Syphilis
I. Toxoplasmosis
J. Varicella
For each patient below, choose the SINGLE most probable diagnosis from the above list of
options. Each option may be used once, more than once or not at all.
1) A newborn baby presents with rudimentary digits, limb hypoplasia and convulsions.
J. Varicella
Note:
Chicken pox infection within the first 20 weeks of pregnancy may result in the congenital varicella
syndrome. This is characterised by cerebral cortical and cerebellar hypoplasia, microcephaly,
convulsions, limb hypoplasia and rudimentary digits. Prevention is by administering varicella vaccine
even before pregnancy. Varicella immunoglobulin is administered to pregnant women who are exposed
to infection. Infection during pregnancy is treated with acylovir.
2) A six week old baby is confirmed to have cataracts, cardiac abnormalities, thrombocytopenia and
cerebral calcification.
G. Rubella
Note:
This baby has congenital rubella. It occurs in children of non immunised women. Symptoms are absent in
50% of mothers. The foetus is most vulnerable in the first 16 weeks of pregnancy. Cataracts are
associated with infections in weeks 8-9, deafness at 5-7 weeks and cardiac lesions from 5-10 weeks.
Diagnosis is based on rising antibody titres in blood taken 10 days apart and the presence of IGM
antibodies at 4-5 weeks from incubation period.
3) A pre term neonate has multi-organ disease with granulomas on his skin. His mother had a special
liking for soft cheese during her pregnancy.
F. Listeriosis
Note:
Maternal listeriosis is usually a mild infection but transplacental infection and premature labour may occur
in about 5% of cases. Avoidance of partially cooked meats, soft cheeses and unpasteurised milk should
avoid this infection. Treatment is with ampicillin and gentamicin. Neonatal infection is usually multi-organ
and granulomas may be found on the skin and the pharynx.
4) A two week old baby has microcephaly, seizures and chorioretinitis.
I. Toxoplasmosis
Note:
Maternal and foetal toxoplasma infection may be avoided by advising pregnant women to wear gloves
when gardening or handling cat litter and to thoroughly cook meat. Affected babies are treated with
pyrimethamine, sulphadiazine and folic acid.
5) A new born baby is very unwell with jaundice, hepatosplenomegaly and microcephaly.
B. Cytomegalovirus (CMV)
Note:
Maternal CMV infection is usually mild and asymptomatic. 5 in 1000 live births are affected, 5% will
develop cytomegalic inclusion disease. The foetus is most at risk in early pregnancy. There is no
effective prevention.
Pediatric extended match questions Dr samed alsalmi
22) Childhood chest infections
A. Chlamydia
B. Cytomegalovirus
C. Group B Streptococcus
D. Haemophilus Influenza
E. Mycoplasma pneumonia
F. Parainfluenza
G. Respiratory syncytial virus
H. Rhinovirus
I. Staphylococcus aureus
J. Ureaplasma
Select the most appropriate pathogen from the above list that would account for the following
presentations:
1) A 4 year old boy presents with a rapid history of high fever and drooling.
D. Haemophilus Influenza
2) A 4 year old girl presents with a 24 hour history of rhinitis, a barking cough and hoarseness.
F. Parainfluenza
3) The commonest cause of pneumonia in the neonate.
C. Group B Streptococcus
Comments:
Item 1 describes a child with acute epiglottitis. This is a rare infection mainly caused by Haemophilus
influenza. Airway obstruction can develop rapidly due to oedema around the epiglottis. Item 2 describe a
child with croup. Acute laryngotracheal bronchitis. It is almost exclusively viral in origin. Mainly
Parainfluenza. Streptococcus pneumonia is a common pathogen in the lung and the commonest cause
of pneumonia in the neonate.
23) Mucocutaneous infections and infestations
A. Enterobiasis vermicularis
B. Chlamydia
C. Candidiasis
D. Tinea pedis
E. Pityriasis rosea
F. Scabies
G. Ringworm
H. Lichen sclerosis
I. Napkin dermatitis
J. Lichen planus
For each of the following children who present with an itch, select the most appropriate diagnosis
from the list of options:
1) A child has itching in the vulva region mostly at night.
A. Enterobiasis vermicularis
Note:
Enterobiasis vermicularis is also known as threadworms. It is a common infestation in children and
present with nocturnal anal pruritus and a perianal irritation.
2) A child presents with annular regions over the trunk.
G. Ringworm
Pediatric extended match questions Dr samed alsalmi
Note:
Ringworm also known as tinea corporis presents with plaques of scaling eczema which are
characteristically itchy.
3) A diabetic child presents with a vulva rash.
C. Candidiasis
Note:
Candidiasis is caused by Candida albicans (yeast). In babies it presents as a perineal rash which usually
affects the flexures. Satellite lesions may also be seen.
24) Presentation of infectious disease in childhood
A. Chicken Pox
B. Herpes simplex
C. Infectious mononucleosis
D. Measles
E. Mumps
F. Mycoplasma
G. Pertussis
H. Rubella
I. Tuberculosis
For each presentation of infectious disease choose the single most likely diagnosis from the list
of options.
1) Commonly causes an acute gingivostomatitis.
B. Herpes simplex
Note:
Herpes simplex - the majority of children have benign manifestations of primary infection with Herpes
simplex, for example a gingival stomatitis. The virus is readily spread by direct contact especially to
damaged skin e.g. eczema.
2) Causing an acute parotitis
E. Mumps
Note:
Mumps infection is now uncommon due to the vaccination. It is caused by a paramyxovirus and usually
causes minimal symptoms. The most common manifestation being an acute parotitis although severe
infection with mumps may result in meningoencephalitis plus deafness.
3) Causing an exudative tonsillitis.
C. Infectious mononucleosis
Note:
Glandular fever is also called infectious mononucleosis. It is caused by Epstein-Barr virus and usually
presents with an exudative pharyngitis or tonsillitis and cervical lymphadenopathy. It may cause a
transient impairment of cellular and humeral immunity, which is usually self limiting.
Pediatric extended match questions Dr samed alsalmi
25) Childhood respiratory infections
A. Allergic Bronchopulmonary aspergillosis
B. Aspergilloma
C. Bordetella pertussis
D. Chlamydia
E. Coxsackie B
F. Klebsiella pneumonia
G. Mycoplasma pneumonia
H. Pneumocystis carinii
I. Pseudomonas aeruginosa
J. Tuberculosis
From the list above, select the most appropriate diagnosis for the following presentations:
1) A 5 year old boy with cystic fibrosis undergoes a routine chest x-ray which reveals an apical round
lesion on the left of his chest.
B. Aspergilloma
Note:
Aspergilloma is a fungus which may complicate conditions such as asthma and cystic fibrosis. Children
present with a cough and wheeze and there may be a positive skin test to aspergillus with an
eosinophilia and an elevated IgE. Chest x-ray may reveal an apical round mass.
2) A 5 year old girl with a high temperature presents with painful blisters on the palms and soles of her
feet
E. Coxsackie B
Note:
describes a child with hand foot and mouth, which is typically caused by Coxsackie B. Characteristically
individuals develop papules which then progress to vesicles. They are painful and pruritic and typically
affect acral areas. It is often associated with a high temperature.
3) A 14 year old boy presents with fever, anorexia and loss of weight of 3 months duration. Scattered
crepitations are heard over both lungs. Chest x-ray is abnormal with generalised mottling.
J. Tuberculosis
Note:
TB- Mycobacterium Tuberculosis is a primary infection which may occur in the lung, gut or skin. The local
infection spreads to the surrounding lymph nodes, which constitutes the primary complex in the lungs.
Progression of the primary complex may result in bronchopneumonia or bronchial obstruction secondary
to enlargement of the lymph nodes. Pleural effusions may also occur as well as cavitations. Primary TB
may spread to the blood stream resulting in TB. Examination may reveal hepatosplenomegaly and
fundoscopy may reveal choroid tubercles.
26) Treatment of infectious disease
A. Acyclovir
B. Acyclovir plus Cefotaxime
C. Ampicillin plus Gentamicin
D. Benzylpenicillin
E. Ceftriaxone
F. Ciprofloxacin
G. Flucloxacillin
H. Netilmicin
I. Symptomatic treatment
J. Teicoplanin
Pediatric extended match questions Dr samed alsalmi
For each case of infectious disease described below choose the single best treatment option
from the list.
1) A 7 year old child develops an exudative tonsillitis and lethargy, monospot is positive
I. Symptomatic treatment
Note:
Glandular fever where the treatment is symptomatic / conservative only.
2) A 1 year old child presents with neck stiffness and a purpuric rash.
D. Benzylpenicillin
Note:
A child with meningococcaemia presented to the surgery and Benzylpenicillin needs to be given urgently
prior to transfer to hospital.
3) A 4 year old child presents with encephalopathy. The EEG shows high amplitude, abnormal
waveforms. The MRI scan is also abnormal. A diagnosis of encephalitis is made.
B. Acyclovir plus Cefotaxime
Note:
A child with encephalitis. The majority of cases are secondary to viruses although toxic and metabolic
causes need to be considered. Of the viruses Herpes simplex is the most common agent causing
encephalitis. When encephalitis is suspected both antibiotics and Acyclovir must be commenced
immediately.
27) Cutaneous manifestations of infectious disease
A. Rheumatic fever
B. TB
C. Lyme's disease
D. Chicken pox
E. Histoplasmosis
F. Cat scratch disease
G. Measles
H. Parvovirus
I. Hepatitis B
J. Herpes simplex
For each cutaneous manifestation described below choose the single most likely associated
infectious disease.
1) Erythema infectiosum.
H. Parvovirus
Note:
Erythema infectiosum also known as Fifths disease caused by human parvovirus, often results in a low
grade temperature, slapped cheek appearance to the face and a reticular lacy rash to the arms.
2) Gianotti-Crosti.
I. Hepatitis B
Note:
Gianotti-Crosti is a syndrome of non-pruritic erythematous papules on the face, buttocks and
extremities. Characteristically related to Hepatitis B infection although other viruses for example EBV
may be associated.
3) Koplik spots.
G. Measles
Note:
Koplik spots are white coarse granules found on the buccal mucosa opposite the back molars and is
pathognomonic for measles.
Pediatric extended match questions Dr samed alsalmi
28) Congenital and neonatal defects due to maternal infection
A. AIDS
B. Coxsackie group B
C. Cytomegalovirus (CMV)
D. Hepatitis B
E. Rubella
F. Herpes simplex
G. Listeriosis
H. Syphilis
I. Varicella
J. Toxoplasmosis
For each patient below, choose the SINGLE most probable diagnosis from the above list of
options. Each option may be used once, more than once or not at all.
1) A newborn baby presents with rudimentary digits, limb hypoplasia and convulsions.
I. Varicella
2) A six week old baby is confirmed to have cataracts, cardiac abnormalities, thrombocytopenia and
cerebral calcification.
E. Rubella
3) A pre term neonate has multi-organ disease with granulomas on his skin. His mother had a special
liking for soft cheese during her pregnancy.
G. Listeriosis
4) A two week old baby has microcephaly, seizures and chorioretinitis.
J. Toxoplasmosis
5) A new born baby is very unwell with jaundice, hepatosplenomegaly and microcephaly.
C. Cytomegalovirus (CMV)
Comments:
Rudimentary digits and limb hypopkasia suggest congenital chicken pox infection. The presence of
cataracts, cardiac abnormalities, thrombocytopaenia and cerebral calcification suggest a diagnosis of
Congenital rubella. The presence of Choroido-retinitis strongly suggests a diagnosis of Toxoplasmosis.
The baby born with granulomas and whose mother has a penchant for soft cheeses suggests Listeria
Monocytogenes infection. Chorio-retinitis suggests The presence of Jaundice, HSM and microcephaly
suggests CMV infection.
29) CHILDHOOD VIRAL INFECTIONS
A. Adenovirus
B. Coxsackie
C. Cytomegalovirus
D. Epstein barr
E. Measles
F. Molluscum contagiosum
G. Mumps
H. Rotavirus
I. Rubella
J. Varicella
For each patient with the group of symptoms listed below, choose the SINGLE most probable
causative agent from the above list of options. Each option may be used once, more than once or
not at all.
Pediatric extended match questions Dr samed alsalmi
1) A two year old infant boy is admitted to hospital with vomiting, none bloody watery diarrhoea and is
dehydrated. It emerges other children from his play group have developed a similar illness.
H. Rotavirus
Note:
Rotavirus is the most common cause of severe viral gastroenteritis worldwide. Infection is via the faeco-
oral route and often occurs in children aged between six months to six years. This RNA virus replicates in
the intestinal mucosal cells damages transport mechanisms leading to salt and water depletion which
results in diarrhorea and vomiting. Diagnosis is made from clinical features and culture of virus from
stools and also by Polymerase chain reaction techniques. Treatment is mainly re-hydration and
correction of any electrolyte imbalance.
2) A two year old boy is mildly unwell. His mother has noticed vesicles in his mouth, palms and soles of
his feet.
B. Coxsackie
Note:
Coxsackie A16 virus is the cause of hand, foot and mouth disease characterized by fever, sore throat
and ulcerating vesicles in palms, orophaynx and on soles. Incubation period is 5-7 days and these heal
without crusting. Treatment is symptomatic.
3) A Three-year baby girl presents with a macular confluent rash which appeared initially behind the ears
and is spreading. Over the previous five days she has had a low grade fever, catarrh and conjunctivitis.
Her mother is vague about her immunization history.
E. Measles
Note:
Measles is caused by and RNA paramyxovirus and occurs worldwide. Outbreaks are common in areas
with high numbers of non immunized children. Infection is transmitted via respiratory droplets and
incubation period is 10-21 days. The prodromal stage fever conjunctivitis, runny nose and coughing lasts
for five days. Koplik’s spots are bright red lesions with a central white dot which appear on the buccal
mucosa. These are virtually diagnostic. The typical macular confluent rash appears on the face from day
3-5 and spreads to the rest of the body. Diagnosis is made from clinical features, viral culture from
lesions and a grater than 4-fold rise in antibody titres. Otitis media, pneumonia, meningitis and very rarely
several years after primary infection subacute sclerosing panencephalitis (SSPE).
4) A Ten year old girl develops an itchy rash on her body which is spreading to her body. She traveled to
see her grandmother who was unwell with a painful rash three weeks ago.
J. Varicella
Note:
Varicella (Chicken pox) is transmitted by respiratory droplets and contact with somebody with shingles.
Incubation period is 14-21 days and following a brief period of malaise, an itchy papulovesicular rash
appears appears on trunk and spreads to the head and the extremities. The rash evolves from papules to
vesicles, pustules and finally crusts. Antiviral therapy is reserved for systemic disease in the
immunocompromised.
5) The four month old baby daughter of an HIV positive mother is admitted to hospital with seizures. She
has neonatal jaundice and microcephaly.
C. Cytomegalovirus
Note:
Cytomegalovirus inclusion disease is the result of infection of the foetus. Many organs may be affected
and congenital abnormalities result. Microcephaly, seizures, neonatal jaundice, hepatosplenomegaly,
deafness and mental retardation are some of the features that may occur.
Pediatric extended match questions Dr samed alsalmi
30) Cutaneous manifestations of infectious disease
A. Rheumatic fever
B. TB
C. Lyme's disease
D. Chicken pox
E. Histoplasmosis
F. Cat scratch disease
G. Measles
H. Parvovirus
I. Hepatitis B
J. Herpes simplex
For each cutaneous manifestation described below choose the single most likely associated
infectious disease.
1) Erythema nodosum in presence of abnormal chest x-ray.
B. TB
Note:
Erythema nodosum may be associated with streptococcal reactions, rheumatic fever and Tuberculosis.
In the presence of an abnormal chest X-ray TB is the most likely answer.
2) Erythema marginatum
A. Rheumatic fever
Note:
Erythema marginatum is one of the five major criteria to make a diagnosis of rheumatic fever. It is a
pink rash with pale centres and a serpiginous margin, found on the trunk and proximal limbs.
3) Erythema chronicum migrans.
C. Lyme's disease
Note:
Erythema chronicum migrans found in Lyme's disease. A febrile illness caused by Borrelia burgdorferi
transmitted by bites of animal tic. The rash is characterised by red margins and central clearing.
Pediatric extended match questions Dr samed alsalmi
Neurology
Theme : Genetics - clinical abnormalities of limbs
A. Bloom's syndrome
B. Cockayne's syndrome
C. Down's syndrome
D. Ehlers Danlos syndrome
E. Hunter's syndrome
F. Prader-Willi syndrome
G. Rubinstein-Taybi syndrome
H. Russell Silver syndrome
I. Sotos' syndrome
J. Williams syndrome
In a child with a suspected genetic disorder and the following clinical abnormalities choose the
single most likely diagnosis from the list of options.
1) Asymmetry of limbs and clinodactyly
H. Russell Silver syndrome
Note:
In Russell-Silver syndrome skeletal features of this condition include short stature of pre-natal onset with
asymmetry, most commonly of the limbs. There is a short inward curving to the 5th finger (clinodactyly).
2) Sandal gap.
C. Down's syndrome
Note:
In Down's syndrome 'Sandal gap' is found. This is wide gap between the first and second toes. Plantar
creases between the first and second toes are usually deep and there is characteristic dermal ridge
pattern. Simian creases are found in approximately 45% of babies with Down's syndrome. This is a
single horizontal palmer crease. 85% of affected individuals have a distal positioning of the palmer axial
tri-radius.
3) Lax ligaments and joint hypermobility.
D. Ehlers Danlos syndrome
Note:
In Ehlers Danlos syndrome there is joint and skeletal hyper-extensibility with poor wound healing. Blood
vessels are fragile and this often manifests itself with recurrent bruising. Affected individuals tend to
have characteristic facial features including epicanthic folds, blue sclerae and a narrow mandible.
Comments:
Examination of the hand sand feet are important in the assessment of a dysmorphic infant.
Theme : Neurological investigation
Pediatric extended match questions Dr samed alsalmi
A. Angiography
B. CT head scan
C. ECG
D. EEG
E. Lumbar puncture
F. MRI scan
G. No investigation required
H. Skeletal survey
I. Skull x-ray
J. Tensilon test
Select the most appropriate investigation for the following cases
1) A 6 month old baby with microcephaly is
brought to A&E. His mother gives a history of the
baby flexing forwards whilst sat in her highchair.
D. EEG
2) A 6 year old boy with a previous history of
convulsions and abnormal EEG's presents with
tingling sensations in his right thumb which
precedes rapid jerking of his right arm.
B. CT head scan
3) A 2 year old boy with an otitis media is
brought to casualty. His father reports that the
child has had a fit. Temperature recorded is
38.8°C.
G. No investigation required
Comments:
Item 1 relates to a baby with infantile spasms. Brief symmetrical contractions of the neck, trunk and
extremities. It is related to pre-natal, peri-natal and post-natal causes for example hypoxic ischaemic
encephalopathy, congenital infections and metabolic disease. The baby requires an EEG and this will
show hypsarrhythmia, a chaotic high amplitude slow wave and spike abnormality. Item 2 describes a boy
with focal seizures, previous EEG's have been abnormal. A CT scan is indicated to exclude underlying
space occupying lesions. The treatment of choice would be Carbamazepine. Febrile convulsions are very
common they manifest as generalised tonic clonic seizures and occur with rapidly rising temperatures.
50% of children having had one febrile convulsion are likely to have a second. No investigations are
required.
Theme : Neurological investigation
A. Angiography
B. CT head scan
C. ECG
D. EEG
E. Lumbar puncture
F. MRI scan
G. No investigation required
Pediatric extended match questions Dr samed alsalmi
H. Skeletal survey
I. Skull x-ray
J. Tensilon test
Select the most appropriate investigation for the following cases
1) A 5 year old boy presents with screaming at
night usually between midnight and 2am. G. No investigation required
2) A 3 month old with a coryzal illness presents to
surgery for his first immunisations. He has
bruising to both cheeks and mother reports that it
is due to rolling off the settee.
H. Skeletal survey
3) A 3 year old boy is brought to hospital for
investigations with a history that he is
progressively getting more tired by the end of the
day and an inability to swallow.
J. Tensilon test
Comments:
Item 1 describes a child having night terrors. These are very common (approximately 3% of all children).
Typically a child between the ages of 5 and 7 will awake between the age of 12am and 2am screaming.
They appear frightened, they may have a tachycardia and dilated pupils. Children tend to fall back to
sleep with total amnesia of the event the following morning. Item 2 describes a baby having sustained
physical, non-accidental injury. This diagnosis must be suspected if medical findings are unexplained or
the history offered is inconsistent. Also if the child suggests that the injury has been caused by an adult.
Characteristic injuries include bruises as in Item 3 and further investigations including blood tests,
skeletal survey, plus or minus a CT head scan must be considered. The last question describes a child
with progressive weakness, hypotonia and an inability to suck and swallow. Myasthenia Gravis must be
considered, this is a defect in the neuromuscular junction with acetyl choline receptor antibodies. A
Tensilon test may show an increase in muscle strength.
Theme : Side effects of Anti-Convulsants
A. Carbamazepine
B. Clonazepam
C. Ethosuximide
D. Gabapentin
E. Lamotrigine
F. Phenobarbitone
G. Phenytoin
H. Sodium Valproate
I. Topiramate
J. Vigabatrin
Select one of the drugs from the list of options that corresponds with the listed side effect:
Pediatric extended match questions Dr samed alsalmi
1) Increased appetite resulting in weight gain.
H. Sodium Valproate
2) Visual field defects.
J. Vigabatrin
3) Is associated with hyperammonaemia.
H. Sodium Valproate
Comments:
Sodium Valproate has many side-effects including gastric irritation and nausea as well as
Hyperammonaemia (interferes with the urea cycle, reducing Carnitine concentrations and has been
associated with hyperammonaenic encephalopathy), increased appetite and weight gain and transient
hair loss.
Vigabatrin is used for treatment of partial epilepsy with or with our secondary generalisation, usually in
combination with other anti-epileptic drugs although can be used as monotherapy in the management of
infantile spasms. Side-effects include drowsiness and confusion and visual field defects.
Theme : Side effects of Anti-Convulsants
A. Carbamazepine
B. Clonazepam
C. Ethosuximide
D. Gabapentin
E. Lamotrigine
F. Phenobarbitone
G. Phenytoin
H. Sodium Valproate
I. Topiramate
J. Vigabatrin
Select one of the drugs from the list of options that corresponds with the listed side effect:
1) Needs to be withdrawn if a rash develops.
E. Lamotrigine
2) Coarse facial features, hirsutism and gingival
hypertrophy. G. Phenytoin
3) Anorexia leading to weight loss.
Pediatric extended match questions Dr samed alsalmi
I. Topiramate
Comments:
Lamotrigine is an anti-convulsant used for treatment of partial seizures as well as generalised seizures. It
is also used for myoclonic seizures, atypical absences and Lennox-Gastaut syndrome. Lamotrigine may
cause a serious skin rash in children and consideration must be given to withdrawal of the drug should
the rash develop.
Phenytoin is effective in the treatment of tonic-clonic seizures and partial seizures. It may cause facies,
acne, hirsutism and gingival hyperplasia.
Topiramate can be given as adjunctive treatment for partial seizures with or with secondary
generalisation. One of the main side-effects is anorexia leading to weight loss.
Pediatric extended match questions Dr samed alsalmi
Pediatrics 1
Pediatrics
1) Theme: ABDOMINAL PAIN
A. Peritonitis
B. Appendicitis
C. Constipation
D. Urinary tract infection
E. Mesenteric lymphadenitis
F. Migraine
G. Intussusception
H. Henoch-Schonlein vasculitis
I. Psychosomatic
J. Shigella dysentry
For the following patients with abdominal pain, choose the most likely diagnosis.
1) A previously well 10 month old baby presenting with
drawing up of her legs with crying and some bloody runny
stools.
G. Intussusception
Note:
This is the right age group for intussception. It is often preceded by an URTI and it is thought that
mesenteric lymphadenopathy may precipitate intussception. A red currant jelly-like stool is a late sign. In
the older child, the cause of intussception may be HSP or lymphoma.
2) A 4-year-old child, with poor eating habits, iron deficiency
anaemia and intermittent colicky abdominal pain.
C. Constipation
Note:
Constipation is common in children and should be managed aggressively to establish regular bowel habit
quickly.
3) A 2-year-old girl with nephrotic syndrome who has
developed a fever and abdominal pain.
A. Peritonitis
Note:
The causes of abdominal pain in nephrotic syndrome are peritonitis (usually pneumococcal), renal vein
thrombosis and hypovolaemia.
2) Theme: Skin lesions
A. Café au lait spots
B. Capillary haemangioma
C. Port-wine stain
D. Shagreen patch
E. Adenoma sebaceum
F. Cutis Marmorata
G. Herald's patch
H. Peri-oral pigmentation
I. Hypo pigmentation
J. Tuber
Select one option from the list above that is most suitable for the following patients
1) An 11 year old boy with learning difficulties has an acne type
rash over the nose
E. Adenoma sebaceum
Note:
relates to the condition known as Tuberous sclerosis. It is a condition where by hamartomatous lesions
become evident in early childhood. Seizures and learning difficulties are common and skin manifestations
include fibrous angiomatous lesions in the naso labial folds known as adenoma sebaceum. White macules
are seen over the trunk known as Ash leaf macules and Shagreen patches resemble goose like flesh.
Subungual fibromata are also common.
2) A 4 year old boy with a history of focal seizures and a birth
mark
C. Port-wine stain
Note:
relates to Sturge-Weber syndrome. The association and localization of aberrant vasculature in the facial
skin, eyes and meninges. Most commonly in the trigeminal nerve distribution.
3) An infant with Down's syndrome has mottled appearance to
the skin.
F. Cutis Marmorata
Note:
relates to Cutis Marmorata, a mottled, marbled type appearance to the skin. Common in Down's
syndrome as well as other Trisomies, hypothyroidism and Cornelia-de-Lange syndrome.
Pediatric extended match questions Dr samed alsalmi
Pediatrics 2
3) Theme: Poisoning
A. Ant psychotics
B. Benzodiazepines
C. Ethylene glycol
D. Methanol
E. Opiates
F. Organophosphates
G. Salicylates
H. Sympathomimetic
I. Tricyclic antidepressants
J. Volatile solvents
Select the most appropriate agent from the above list that would elicit the following toxic effects:
1) Coma, pinpoint pupils, hyperventilation E. Opiates
2) Hyper salivation, broncho-rrhoea, broncho- spasms,
perspiration, neuromuscular paralysis
F. Organophosphates
3) Tachypnoea, metabolic acidosis, Haematemesis, Reyes
Syndrome.
G. Salicylates
Comments:
A pinpoint pupil suggests : opiates (Heroin – Morphine – codeine). The hyper salivation, tachypnoea
and sweating suggest :Organophosphate poisoning. This drug inhibits ant-cholinesterase activity
promoting Ach effects. Treatment consists of anticholinergics < (Atropine dose i.v 0.05 mg/kg repeated
every 5-10 min as needed adverse effect tachycardia, dry mouth, blurred vision, and urinary retention).
Finally, the use of salicylates in children is well-recognized cause of Reyes syndrome, a condition
associated with fulminant hepatic failure. Suspected poisoning in children result in thousands of
attendances at A&E department each year. Various drugs cause specific signs when taken in overdose.
Careful examination of the child should follow A, B, C protocol.
4) Theme: Chemotherapy
Select the most appropriate chemotherapeutic agent from the above list that corresponds with
the following actions and side effects:
1) A drug disrupting microtubule leading to loss of deep tendon reflexes. J. Vincristine
2) A drug inhibiting initiation of DNA synthesis resulting in conjunctivitis
and cerebellar toxicity.
G. Cytosine arabinoside
3) A drug resulting in dose related lung damage. D. Bleomycin
Comments:
All cytotoxic agents have adverse effects. General side-effects include nausea,
vomiting and bone marrow suppression, alopecia and stomatitis. More specific side-
effects are listed in the questions above. Alkylating agents such as Cyclophosphamide
may cause a haemorrhagic cystitis. Asparaginase results in a dose related Pancreatitis.
The anthracyclines e.g. Doxorubicin and Daunorubicin are cardiotoxic, which is often
very difficult to detect. Monitoring with echocardiograms is advised. Vincristine is an
alkaloid agent and results in sensory motor neuropathy with long-term use. It also has
an affect on the autonomic system resulting in severe constipation and paralytic ileus.
It may also result in sensory changes with parathesis progressing to loss of tendon
reflexes. Bleomycin is the main drug resulting in lung damage and occurs in up to 10%
of patients. The damage is dose related.
Pediatric extended match questions Dr samed alsalmi
Pediatric emq
Pediatric emq
Pediatric emq
Pediatric emq
Pediatric emq
Pediatric emq
Pediatric emq
Pediatric emq
Pediatric emq
Pediatric emq
Pediatric emq
Pediatric emq
Pediatric emq
Pediatric emq
Pediatric emq
Pediatric emq
Pediatric emq
Pediatric emq
Pediatric emq
Pediatric emq
Pediatric emq
Pediatric emq
Pediatric emq
Pediatric emq
Pediatric emq
Pediatric emq
Pediatric emq
Pediatric emq
Pediatric emq
Pediatric emq
Pediatric emq
Pediatric emq
Pediatric emq
Pediatric emq
Pediatric emq
Pediatric emq
Pediatric emq
Pediatric emq

Contenu connexe

Tendances

Neonatology test
Neonatology testNeonatology test
Neonatology testVarsha Shah
 
Approach to a child with Hepatosplenomegaly
Approach to a child with HepatosplenomegalyApproach to a child with Hepatosplenomegaly
Approach to a child with HepatosplenomegalySunil Agrawal
 
DNB Pediatrics OSCE CME (Command Hospital, Pune)
DNB Pediatrics OSCE CME (Command Hospital, Pune)DNB Pediatrics OSCE CME (Command Hospital, Pune)
DNB Pediatrics OSCE CME (Command Hospital, Pune)Dr Padmesh Vadakepat
 
OSCE in Pediatrics (Wadia, Sept 2011)
OSCE in Pediatrics (Wadia, Sept 2011)OSCE in Pediatrics (Wadia, Sept 2011)
OSCE in Pediatrics (Wadia, Sept 2011)Dr Padmesh Vadakepat
 
Diagnosis and management of dengue in children (IAP Infectious Diseases Chapter)
Diagnosis and management of dengue in children (IAP Infectious Diseases Chapter)Diagnosis and management of dengue in children (IAP Infectious Diseases Chapter)
Diagnosis and management of dengue in children (IAP Infectious Diseases Chapter)Dr Padmesh Vadakepat
 
Basic approach on short stature in children
Basic approach on short stature in childrenBasic approach on short stature in children
Basic approach on short stature in childrenAzad Haleem
 
Hematemesis in children-Beyond Infancy
Hematemesis in children-Beyond InfancyHematemesis in children-Beyond Infancy
Hematemesis in children-Beyond Infancydivyaanair
 
approach to short stature
approach to short statureapproach to short stature
approach to short statureRatnakar Vallem
 
Approach to child with short stature
Approach to child with short statureApproach to child with short stature
Approach to child with short statureAbdulmoein AlAgha
 
Mock OSCE in Pediatrics Apr 2014 Part 1 qn ans
Mock OSCE in Pediatrics Apr 2014 Part 1 qn ansMock OSCE in Pediatrics Apr 2014 Part 1 qn ans
Mock OSCE in Pediatrics Apr 2014 Part 1 qn ansDr Padmesh Vadakepat
 
OSCE IN PEDIATRICS (March 4th 2008) SGRH
OSCE IN PEDIATRICS (March 4th 2008) SGRHOSCE IN PEDIATRICS (March 4th 2008) SGRH
OSCE IN PEDIATRICS (March 4th 2008) SGRHDr Padmesh Vadakepat
 
Practical pediatric quiz - Kaun Banega Winner
Practical pediatric quiz - Kaun Banega WinnerPractical pediatric quiz - Kaun Banega Winner
Practical pediatric quiz - Kaun Banega WinnerGaurav Gupta
 
Pediatric hypoglycemia
Pediatric hypoglycemiaPediatric hypoglycemia
Pediatric hypoglycemiaOsama Arafa
 
Pediatrics OSCE pictures
Pediatrics OSCE pictures Pediatrics OSCE pictures
Pediatrics OSCE pictures Shahd Al Ali
 
OSCE Pediatrics Observed Stations (Mock Exam Apr 2013)
OSCE Pediatrics Observed Stations (Mock Exam Apr 2013)OSCE Pediatrics Observed Stations (Mock Exam Apr 2013)
OSCE Pediatrics Observed Stations (Mock Exam Apr 2013)Dr Padmesh Vadakepat
 
Chronic liver disease in children 2021
Chronic liver disease in children 2021Chronic liver disease in children 2021
Chronic liver disease in children 2021Imran Iqbal
 

Tendances (20)

Neonatology test
Neonatology testNeonatology test
Neonatology test
 
Approach to a child with Hepatosplenomegaly
Approach to a child with HepatosplenomegalyApproach to a child with Hepatosplenomegaly
Approach to a child with Hepatosplenomegaly
 
DNB Pediatrics OSCE CME (Command Hospital, Pune)
DNB Pediatrics OSCE CME (Command Hospital, Pune)DNB Pediatrics OSCE CME (Command Hospital, Pune)
DNB Pediatrics OSCE CME (Command Hospital, Pune)
 
OSCE in Pediatrics (Wadia, Sept 2011)
OSCE in Pediatrics (Wadia, Sept 2011)OSCE in Pediatrics (Wadia, Sept 2011)
OSCE in Pediatrics (Wadia, Sept 2011)
 
Diagnosis and management of dengue in children (IAP Infectious Diseases Chapter)
Diagnosis and management of dengue in children (IAP Infectious Diseases Chapter)Diagnosis and management of dengue in children (IAP Infectious Diseases Chapter)
Diagnosis and management of dengue in children (IAP Infectious Diseases Chapter)
 
Basic approach on short stature in children
Basic approach on short stature in childrenBasic approach on short stature in children
Basic approach on short stature in children
 
Hematemesis in children-Beyond Infancy
Hematemesis in children-Beyond InfancyHematemesis in children-Beyond Infancy
Hematemesis in children-Beyond Infancy
 
approach to short stature
approach to short statureapproach to short stature
approach to short stature
 
RIPE 2012 Pediatrics OSCE
RIPE 2012 Pediatrics OSCERIPE 2012 Pediatrics OSCE
RIPE 2012 Pediatrics OSCE
 
Diabetes Insipidus in Children
Diabetes Insipidus in Children Diabetes Insipidus in Children
Diabetes Insipidus in Children
 
Approach to child with short stature
Approach to child with short statureApproach to child with short stature
Approach to child with short stature
 
Mock OSCE in Pediatrics Apr 2014 Part 1 qn ans
Mock OSCE in Pediatrics Apr 2014 Part 1 qn ansMock OSCE in Pediatrics Apr 2014 Part 1 qn ans
Mock OSCE in Pediatrics Apr 2014 Part 1 qn ans
 
OSCE IN PEDIATRICS (March 4th 2008) SGRH
OSCE IN PEDIATRICS (March 4th 2008) SGRHOSCE IN PEDIATRICS (March 4th 2008) SGRH
OSCE IN PEDIATRICS (March 4th 2008) SGRH
 
Practical pediatric quiz - Kaun Banega Winner
Practical pediatric quiz - Kaun Banega WinnerPractical pediatric quiz - Kaun Banega Winner
Practical pediatric quiz - Kaun Banega Winner
 
Pediatric hypoglycemia
Pediatric hypoglycemiaPediatric hypoglycemia
Pediatric hypoglycemia
 
Pediatrics OSCE pictures
Pediatrics OSCE pictures Pediatrics OSCE pictures
Pediatrics OSCE pictures
 
NEC in newborn
NEC in newbornNEC in newborn
NEC in newborn
 
OSCE Pediatrics Observed Stations (Mock Exam Apr 2013)
OSCE Pediatrics Observed Stations (Mock Exam Apr 2013)OSCE Pediatrics Observed Stations (Mock Exam Apr 2013)
OSCE Pediatrics Observed Stations (Mock Exam Apr 2013)
 
SHORT STATURE
SHORT STATURESHORT STATURE
SHORT STATURE
 
Chronic liver disease in children 2021
Chronic liver disease in children 2021Chronic liver disease in children 2021
Chronic liver disease in children 2021
 

Similaire à Pediatric emq

Pediatric endocrinology review - part 3
Pediatric endocrinology review -  part 3Pediatric endocrinology review -  part 3
Pediatric endocrinology review - part 3Abdulmoein AlAgha
 
Paediatric Emergencies
Paediatric EmergenciesPaediatric Emergencies
Paediatric EmergenciesKane Guthrie
 
Krok 2 - 2013 (Pediatrics)
Krok 2 - 2013 (Pediatrics)Krok 2 - 2013 (Pediatrics)
Krok 2 - 2013 (Pediatrics)Eneutron
 
Krok 2 - 2014 (Pediatrics)
Krok 2 - 2014 (Pediatrics)Krok 2 - 2014 (Pediatrics)
Krok 2 - 2014 (Pediatrics)Eneutron
 
16672475 23 Mock Codes 2
16672475 23  Mock  Codes 216672475 23  Mock  Codes 2
16672475 23 Mock Codes 2Surgery Center
 
Drs. Olson’s and Jackson’s CMC Pediatric X-Ray Mastery: May Cases
Drs. Olson’s and Jackson’s CMC Pediatric X-Ray Mastery: May CasesDrs. Olson’s and Jackson’s CMC Pediatric X-Ray Mastery: May Cases
Drs. Olson’s and Jackson’s CMC Pediatric X-Ray Mastery: May CasesSean M. Fox
 
Learning points emergency resuscitation
Learning points emergency resuscitationLearning points emergency resuscitation
Learning points emergency resuscitationAltaf Bhat
 
Seizures in children
Seizures in childrenSeizures in children
Seizures in childrenMuhamad Masri
 
Care of unconscious patient
Care of unconscious patient  Care of unconscious patient
Care of unconscious patient Slideshare User
 
Treating pediatric summertime emergencies
Treating pediatric summertime emergenciesTreating pediatric summertime emergencies
Treating pediatric summertime emergenciesfiremed2c
 
Cardiomyopathy & Homoeopathy: A Case Study
Cardiomyopathy & Homoeopathy: A Case StudyCardiomyopathy & Homoeopathy: A Case Study
Cardiomyopathy & Homoeopathy: A Case StudyGyandas Wadhwani
 
Premier Medillectuals - Mains
Premier Medillectuals - MainsPremier Medillectuals - Mains
Premier Medillectuals - MainsDhananjay Bansal
 
Drs. Olson’s and Jackson’s CMC Pediatric X-Ray Mastery: August Cases
Drs. Olson’s and Jackson’s CMC Pediatric X-Ray Mastery: August CasesDrs. Olson’s and Jackson’s CMC Pediatric X-Ray Mastery: August Cases
Drs. Olson’s and Jackson’s CMC Pediatric X-Ray Mastery: August CasesSean M. Fox
 
PRIMARY MANAGEMENT OF TRAUMA.pptx
PRIMARY MANAGEMENT OF TRAUMA.pptxPRIMARY MANAGEMENT OF TRAUMA.pptx
PRIMARY MANAGEMENT OF TRAUMA.pptxRAGHUNATHKARMAKER1
 

Similaire à Pediatric emq (20)

Emergency cases 1
Emergency cases 1Emergency cases 1
Emergency cases 1
 
Congenital heart-disease2787
Congenital heart-disease2787Congenital heart-disease2787
Congenital heart-disease2787
 
Pediatric endocrinology review - part 3
Pediatric endocrinology review -  part 3Pediatric endocrinology review -  part 3
Pediatric endocrinology review - part 3
 
Paediatric Emergencies
Paediatric EmergenciesPaediatric Emergencies
Paediatric Emergencies
 
Krok 2 - 2013 (Pediatrics)
Krok 2 - 2013 (Pediatrics)Krok 2 - 2013 (Pediatrics)
Krok 2 - 2013 (Pediatrics)
 
Krok 2 - 2014 (Pediatrics)
Krok 2 - 2014 (Pediatrics)Krok 2 - 2014 (Pediatrics)
Krok 2 - 2014 (Pediatrics)
 
Trauma lecture
Trauma lectureTrauma lecture
Trauma lecture
 
16672475 23 Mock Codes 2
16672475 23  Mock  Codes 216672475 23  Mock  Codes 2
16672475 23 Mock Codes 2
 
Drs. Olson’s and Jackson’s CMC Pediatric X-Ray Mastery: May Cases
Drs. Olson’s and Jackson’s CMC Pediatric X-Ray Mastery: May CasesDrs. Olson’s and Jackson’s CMC Pediatric X-Ray Mastery: May Cases
Drs. Olson’s and Jackson’s CMC Pediatric X-Ray Mastery: May Cases
 
Learning points emergency resuscitation
Learning points emergency resuscitationLearning points emergency resuscitation
Learning points emergency resuscitation
 
Seizures in children
Seizures in childrenSeizures in children
Seizures in children
 
Medicognitive Quiz
Medicognitive QuizMedicognitive Quiz
Medicognitive Quiz
 
Care of unconscious patient
Care of unconscious patient  Care of unconscious patient
Care of unconscious patient
 
Treating pediatric summertime emergencies
Treating pediatric summertime emergenciesTreating pediatric summertime emergencies
Treating pediatric summertime emergencies
 
Cardiomyopathy & Homoeopathy: A Case Study
Cardiomyopathy & Homoeopathy: A Case StudyCardiomyopathy & Homoeopathy: A Case Study
Cardiomyopathy & Homoeopathy: A Case Study
 
Premier Medillectuals - Mains
Premier Medillectuals - MainsPremier Medillectuals - Mains
Premier Medillectuals - Mains
 
Premier Medillectuals :- Mains
Premier Medillectuals :- MainsPremier Medillectuals :- Mains
Premier Medillectuals :- Mains
 
Drs. Olson’s and Jackson’s CMC Pediatric X-Ray Mastery: August Cases
Drs. Olson’s and Jackson’s CMC Pediatric X-Ray Mastery: August CasesDrs. Olson’s and Jackson’s CMC Pediatric X-Ray Mastery: August Cases
Drs. Olson’s and Jackson’s CMC Pediatric X-Ray Mastery: August Cases
 
PRIMARY MANAGEMENT OF TRAUMA.pptx
PRIMARY MANAGEMENT OF TRAUMA.pptxPRIMARY MANAGEMENT OF TRAUMA.pptx
PRIMARY MANAGEMENT OF TRAUMA.pptx
 
Junior Medillectuals, Synapse 2018
Junior Medillectuals, Synapse 2018Junior Medillectuals, Synapse 2018
Junior Medillectuals, Synapse 2018
 

Dernier

Ahmedabad Call Girls CG Road 🔝9907093804 Short 1500 💋 Night 6000
Ahmedabad Call Girls CG Road 🔝9907093804  Short 1500  💋 Night 6000Ahmedabad Call Girls CG Road 🔝9907093804  Short 1500  💋 Night 6000
Ahmedabad Call Girls CG Road 🔝9907093804 Short 1500 💋 Night 6000aliya bhat
 
Call Girls Electronic City Just Call 7001305949 Top Class Call Girl Service A...
Call Girls Electronic City Just Call 7001305949 Top Class Call Girl Service A...Call Girls Electronic City Just Call 7001305949 Top Class Call Girl Service A...
Call Girls Electronic City Just Call 7001305949 Top Class Call Girl Service A...narwatsonia7
 
VIP Call Girls Mumbai Arpita 9910780858 Independent Escort Service Mumbai
VIP Call Girls Mumbai Arpita 9910780858 Independent Escort Service MumbaiVIP Call Girls Mumbai Arpita 9910780858 Independent Escort Service Mumbai
VIP Call Girls Mumbai Arpita 9910780858 Independent Escort Service Mumbaisonalikaur4
 
Hemostasis Physiology and Clinical correlations by Dr Faiza.pdf
Hemostasis Physiology and Clinical correlations by Dr Faiza.pdfHemostasis Physiology and Clinical correlations by Dr Faiza.pdf
Hemostasis Physiology and Clinical correlations by Dr Faiza.pdfMedicoseAcademics
 
Housewife Call Girls Bangalore - Call 7001305949 Rs-3500 with A/C Room Cash o...
Housewife Call Girls Bangalore - Call 7001305949 Rs-3500 with A/C Room Cash o...Housewife Call Girls Bangalore - Call 7001305949 Rs-3500 with A/C Room Cash o...
Housewife Call Girls Bangalore - Call 7001305949 Rs-3500 with A/C Room Cash o...narwatsonia7
 
Bangalore Call Girls Marathahalli 📞 9907093804 High Profile Service 100% Safe
Bangalore Call Girls Marathahalli 📞 9907093804 High Profile Service 100% SafeBangalore Call Girls Marathahalli 📞 9907093804 High Profile Service 100% Safe
Bangalore Call Girls Marathahalli 📞 9907093804 High Profile Service 100% Safenarwatsonia7
 
call girls in Connaught Place DELHI 🔝 >༒9540349809 🔝 genuine Escort Service ...
call girls in Connaught Place  DELHI 🔝 >༒9540349809 🔝 genuine Escort Service ...call girls in Connaught Place  DELHI 🔝 >༒9540349809 🔝 genuine Escort Service ...
call girls in Connaught Place DELHI 🔝 >༒9540349809 🔝 genuine Escort Service ...saminamagar
 
Sonagachi Call Girls Services 9907093804 @24x7 High Class Babes Here Call Now
Sonagachi Call Girls Services 9907093804 @24x7 High Class Babes Here Call NowSonagachi Call Girls Services 9907093804 @24x7 High Class Babes Here Call Now
Sonagachi Call Girls Services 9907093804 @24x7 High Class Babes Here Call NowRiya Pathan
 
See the 2,456 pharmacies on the National E-Pharmacy Platform
See the 2,456 pharmacies on the National E-Pharmacy PlatformSee the 2,456 pharmacies on the National E-Pharmacy Platform
See the 2,456 pharmacies on the National E-Pharmacy PlatformKweku Zurek
 
College Call Girls Vyasarpadi Whatsapp 7001305949 Independent Escort Service
College Call Girls Vyasarpadi Whatsapp 7001305949 Independent Escort ServiceCollege Call Girls Vyasarpadi Whatsapp 7001305949 Independent Escort Service
College Call Girls Vyasarpadi Whatsapp 7001305949 Independent Escort ServiceNehru place Escorts
 
College Call Girls Pune Mira 9907093804 Short 1500 Night 6000 Best call girls...
College Call Girls Pune Mira 9907093804 Short 1500 Night 6000 Best call girls...College Call Girls Pune Mira 9907093804 Short 1500 Night 6000 Best call girls...
College Call Girls Pune Mira 9907093804 Short 1500 Night 6000 Best call girls...Miss joya
 
Aspirin presentation slides by Dr. Rewas Ali
Aspirin presentation slides by Dr. Rewas AliAspirin presentation slides by Dr. Rewas Ali
Aspirin presentation slides by Dr. Rewas AliRewAs ALI
 
Russian Call Girl Brookfield - 7001305949 Escorts Service 50% Off with Cash O...
Russian Call Girl Brookfield - 7001305949 Escorts Service 50% Off with Cash O...Russian Call Girl Brookfield - 7001305949 Escorts Service 50% Off with Cash O...
Russian Call Girl Brookfield - 7001305949 Escorts Service 50% Off with Cash O...narwatsonia7
 
VIP Call Girls Lucknow Nandini 7001305949 Independent Escort Service Lucknow
VIP Call Girls Lucknow Nandini 7001305949 Independent Escort Service LucknowVIP Call Girls Lucknow Nandini 7001305949 Independent Escort Service Lucknow
VIP Call Girls Lucknow Nandini 7001305949 Independent Escort Service Lucknownarwatsonia7
 
Call Girls Jayanagar Just Call 7001305949 Top Class Call Girl Service Available
Call Girls Jayanagar Just Call 7001305949 Top Class Call Girl Service AvailableCall Girls Jayanagar Just Call 7001305949 Top Class Call Girl Service Available
Call Girls Jayanagar Just Call 7001305949 Top Class Call Girl Service Availablenarwatsonia7
 
Call Girls Hsr Layout Just Call 7001305949 Top Class Call Girl Service Available
Call Girls Hsr Layout Just Call 7001305949 Top Class Call Girl Service AvailableCall Girls Hsr Layout Just Call 7001305949 Top Class Call Girl Service Available
Call Girls Hsr Layout Just Call 7001305949 Top Class Call Girl Service Availablenarwatsonia7
 
Book Call Girls in Yelahanka - For 7001305949 Cheap & Best with original Photos
Book Call Girls in Yelahanka - For 7001305949 Cheap & Best with original PhotosBook Call Girls in Yelahanka - For 7001305949 Cheap & Best with original Photos
Book Call Girls in Yelahanka - For 7001305949 Cheap & Best with original Photosnarwatsonia7
 
Call Girls Service Chennai Jiya 7001305949 Independent Escort Service Chennai
Call Girls Service Chennai Jiya 7001305949 Independent Escort Service ChennaiCall Girls Service Chennai Jiya 7001305949 Independent Escort Service Chennai
Call Girls Service Chennai Jiya 7001305949 Independent Escort Service ChennaiNehru place Escorts
 
Call Girls Jp Nagar Just Call 7001305949 Top Class Call Girl Service Available
Call Girls Jp Nagar Just Call 7001305949 Top Class Call Girl Service AvailableCall Girls Jp Nagar Just Call 7001305949 Top Class Call Girl Service Available
Call Girls Jp Nagar Just Call 7001305949 Top Class Call Girl Service Availablenarwatsonia7
 

Dernier (20)

Ahmedabad Call Girls CG Road 🔝9907093804 Short 1500 💋 Night 6000
Ahmedabad Call Girls CG Road 🔝9907093804  Short 1500  💋 Night 6000Ahmedabad Call Girls CG Road 🔝9907093804  Short 1500  💋 Night 6000
Ahmedabad Call Girls CG Road 🔝9907093804 Short 1500 💋 Night 6000
 
Call Girls Electronic City Just Call 7001305949 Top Class Call Girl Service A...
Call Girls Electronic City Just Call 7001305949 Top Class Call Girl Service A...Call Girls Electronic City Just Call 7001305949 Top Class Call Girl Service A...
Call Girls Electronic City Just Call 7001305949 Top Class Call Girl Service A...
 
sauth delhi call girls in Bhajanpura 🔝 9953056974 🔝 escort Service
sauth delhi call girls in Bhajanpura 🔝 9953056974 🔝 escort Servicesauth delhi call girls in Bhajanpura 🔝 9953056974 🔝 escort Service
sauth delhi call girls in Bhajanpura 🔝 9953056974 🔝 escort Service
 
VIP Call Girls Mumbai Arpita 9910780858 Independent Escort Service Mumbai
VIP Call Girls Mumbai Arpita 9910780858 Independent Escort Service MumbaiVIP Call Girls Mumbai Arpita 9910780858 Independent Escort Service Mumbai
VIP Call Girls Mumbai Arpita 9910780858 Independent Escort Service Mumbai
 
Hemostasis Physiology and Clinical correlations by Dr Faiza.pdf
Hemostasis Physiology and Clinical correlations by Dr Faiza.pdfHemostasis Physiology and Clinical correlations by Dr Faiza.pdf
Hemostasis Physiology and Clinical correlations by Dr Faiza.pdf
 
Housewife Call Girls Bangalore - Call 7001305949 Rs-3500 with A/C Room Cash o...
Housewife Call Girls Bangalore - Call 7001305949 Rs-3500 with A/C Room Cash o...Housewife Call Girls Bangalore - Call 7001305949 Rs-3500 with A/C Room Cash o...
Housewife Call Girls Bangalore - Call 7001305949 Rs-3500 with A/C Room Cash o...
 
Bangalore Call Girls Marathahalli 📞 9907093804 High Profile Service 100% Safe
Bangalore Call Girls Marathahalli 📞 9907093804 High Profile Service 100% SafeBangalore Call Girls Marathahalli 📞 9907093804 High Profile Service 100% Safe
Bangalore Call Girls Marathahalli 📞 9907093804 High Profile Service 100% Safe
 
call girls in Connaught Place DELHI 🔝 >༒9540349809 🔝 genuine Escort Service ...
call girls in Connaught Place  DELHI 🔝 >༒9540349809 🔝 genuine Escort Service ...call girls in Connaught Place  DELHI 🔝 >༒9540349809 🔝 genuine Escort Service ...
call girls in Connaught Place DELHI 🔝 >༒9540349809 🔝 genuine Escort Service ...
 
Sonagachi Call Girls Services 9907093804 @24x7 High Class Babes Here Call Now
Sonagachi Call Girls Services 9907093804 @24x7 High Class Babes Here Call NowSonagachi Call Girls Services 9907093804 @24x7 High Class Babes Here Call Now
Sonagachi Call Girls Services 9907093804 @24x7 High Class Babes Here Call Now
 
See the 2,456 pharmacies on the National E-Pharmacy Platform
See the 2,456 pharmacies on the National E-Pharmacy PlatformSee the 2,456 pharmacies on the National E-Pharmacy Platform
See the 2,456 pharmacies on the National E-Pharmacy Platform
 
College Call Girls Vyasarpadi Whatsapp 7001305949 Independent Escort Service
College Call Girls Vyasarpadi Whatsapp 7001305949 Independent Escort ServiceCollege Call Girls Vyasarpadi Whatsapp 7001305949 Independent Escort Service
College Call Girls Vyasarpadi Whatsapp 7001305949 Independent Escort Service
 
College Call Girls Pune Mira 9907093804 Short 1500 Night 6000 Best call girls...
College Call Girls Pune Mira 9907093804 Short 1500 Night 6000 Best call girls...College Call Girls Pune Mira 9907093804 Short 1500 Night 6000 Best call girls...
College Call Girls Pune Mira 9907093804 Short 1500 Night 6000 Best call girls...
 
Aspirin presentation slides by Dr. Rewas Ali
Aspirin presentation slides by Dr. Rewas AliAspirin presentation slides by Dr. Rewas Ali
Aspirin presentation slides by Dr. Rewas Ali
 
Russian Call Girl Brookfield - 7001305949 Escorts Service 50% Off with Cash O...
Russian Call Girl Brookfield - 7001305949 Escorts Service 50% Off with Cash O...Russian Call Girl Brookfield - 7001305949 Escorts Service 50% Off with Cash O...
Russian Call Girl Brookfield - 7001305949 Escorts Service 50% Off with Cash O...
 
VIP Call Girls Lucknow Nandini 7001305949 Independent Escort Service Lucknow
VIP Call Girls Lucknow Nandini 7001305949 Independent Escort Service LucknowVIP Call Girls Lucknow Nandini 7001305949 Independent Escort Service Lucknow
VIP Call Girls Lucknow Nandini 7001305949 Independent Escort Service Lucknow
 
Call Girls Jayanagar Just Call 7001305949 Top Class Call Girl Service Available
Call Girls Jayanagar Just Call 7001305949 Top Class Call Girl Service AvailableCall Girls Jayanagar Just Call 7001305949 Top Class Call Girl Service Available
Call Girls Jayanagar Just Call 7001305949 Top Class Call Girl Service Available
 
Call Girls Hsr Layout Just Call 7001305949 Top Class Call Girl Service Available
Call Girls Hsr Layout Just Call 7001305949 Top Class Call Girl Service AvailableCall Girls Hsr Layout Just Call 7001305949 Top Class Call Girl Service Available
Call Girls Hsr Layout Just Call 7001305949 Top Class Call Girl Service Available
 
Book Call Girls in Yelahanka - For 7001305949 Cheap & Best with original Photos
Book Call Girls in Yelahanka - For 7001305949 Cheap & Best with original PhotosBook Call Girls in Yelahanka - For 7001305949 Cheap & Best with original Photos
Book Call Girls in Yelahanka - For 7001305949 Cheap & Best with original Photos
 
Call Girls Service Chennai Jiya 7001305949 Independent Escort Service Chennai
Call Girls Service Chennai Jiya 7001305949 Independent Escort Service ChennaiCall Girls Service Chennai Jiya 7001305949 Independent Escort Service Chennai
Call Girls Service Chennai Jiya 7001305949 Independent Escort Service Chennai
 
Call Girls Jp Nagar Just Call 7001305949 Top Class Call Girl Service Available
Call Girls Jp Nagar Just Call 7001305949 Top Class Call Girl Service AvailableCall Girls Jp Nagar Just Call 7001305949 Top Class Call Girl Service Available
Call Girls Jp Nagar Just Call 7001305949 Top Class Call Girl Service Available
 

Pediatric emq

  • 1.
  • 2. To download more new and updated pediatrics’ books, Journals and guidelines you can visit my medical BlogSpot and 4shared on the following links http://drsamed77paediatricbooks.blogspot.com/ http://www.4shared.com/u/N86Q7qyj/drsamed77mrcpch1.html For contact mailto:dr.samed77@yahoo.com mailto:dr.samed77mrcpch1@gmail.com mailto:dr.samed77@windowslive.com
  • 3. Development 1 Development Theme : Stature A. Achondroplasia B. Anorexia nervosa C. Constitutional delay D. Cornelia-de-Lange syndrome E. Familial short stature F. Growth hormone deficiency G. Primordial dwarfism H. Psychosocial poor growth I. Prader-Willi syndrome J. Russell-silver dwarfism Select the most appropriate diagnosis from the above options to explain the following presentations: 1) A child has short stature, a triangular face and facial and limb length asymmetry. J. Russell-silver dwarfism Note: describes a child with Russell –silver syndrome. A condition of short stature, hemi-hypertrophy and the child often has triangular facies, frontal bossing and clinodactyly of the 5th finger. 2) A child has short stature and is noted to have a large head with a prominent forehead. A. Achondroplasia Note: describes a child with achondroplasia. Short stature, rhizomelia (short limbs especially the proximal part). Affected individuals usually have a large head with hydrocephalus being a complication. 3) A baby is born on the 50th centile for height and weight. By the age of 2 her length has fallen to the 2nd centile and weight remains on the 50th. She has a rounded face with small hands and feet. F. Growth hormone deficiency Note: describes a growth hormone deficient child. Early recognition and initiation of treatment is required to reach satisfactory adult height. Theme : Stature A. Achondroplasia B. Anorexia nervosa C. Constitutional delay D. Cornelia-de-Lange syndrome E. Familial short stature Pediatric extended match questions Dr samed alsalmi
  • 4. Development 2 F. Growth hormone deficiency G. Primordial dwarfism H. Psychosocial poor growth I. Prader-Willi syndrome J. Russell-silver dwarfism Select the most appropriate diagnosis from the above options to explain the following presentations: 1) A 9 year old boy statemented for special educational needs is investigated for short stature. He has a weight over the 91st centile. He is noted to have small hands and feet and hypogonadism. I. Prader-Willi syndrome Note: describes a child with Prader-Willi syndrome. A condition characterised by obesity after the first year of life. Hypotonia, hypogonadism and learning difficulties due to a deletion of chromosome 15 (the deletion occurring in the paternal gene, if the deletion of chromosome 15 in the child was inherited from the mother the child would have the phenotype of Angelman's syndrome). 2) A 15 year old girl in foster care is referred to outpatients because of delayed puberty. On examination her weight is on the 2nd centile and her height is on the 25th centile. She has poor dentition and her temperature is measured at 35°C. B. Anorexia nervosa Note: describes an adolescent girl with anorexia nervosa. A condition due to fear of becoming obese or disturbed body image. There is usually a tendency to deny hunger, excessive dieting, laxative abuse or excessive physical activity. The diagnosis is classified as weight of 15% below that expected for age and height and is associated with physical characteristics including amenorrhoea, bradycardia, hypothermia and electrolyte disturbances. 3) A 7 year old girl presents with short stature with weight being on the 9th centile and height less than 0.4th centile. Examination is normal as well as all investigations including a bone age, which is compatible with her chronological age. Her mother is 5”1 (134cm), her father is 5”5 (143cm). E. Familial short stature Note: describes a child with familial short stature. From the measurements of the parents the child's height is compatible with this diagnosis. It is expected that most children will reach a height within 8cms of the mid-parental height centile. Pediatric extended match questions Dr samed alsalmi
  • 5. Cardiology 1 Cardiology Theme : Emergency medicine A. Adenosine B. Adrenaline C. Atropine D. DC shock E. Dobutamine F. Diving reflex G. Endotracheal intubation H. Intraosseous line I. IV Morphine J. Naloxone Select the most appropriate emergency treatment for the following children: 1) A 4-year-old child is rescued from a house fire. She is admitted tachypnoeic and tacchycardic. She has soot in her nostrils. G. Endotracheal intubation Note: Item 1 relates to burns. Most deaths following house fires occur secondary to smoke inhalation. Amongst the indicators of inhaled smoke injury is deposits around the mouth and nose. Oedema follows thermal injury and therefore any suspicion of airway compromise should result in endotracheal intubation. 2) A 3 month old baby is admitted with a history of poor feeding. On arrival he has a pulse rate of 220 beats per minute. F. Diving reflex Note: Supraventricular tachycardia is the diagnosis in Item 3. Vagal stimulation is the treatment of choice and the diving reflex is the simple procedure elicited by submerging the baby's face in to ice or placing an ice bag over the face. The diving reflex increases vagal tone, slows AV conduction interrupting the tachycardia. 3) A 13 year old boy is admitted with meningococcaemia. He is in shock. Peripheral cannulation is difficult. H. Intraosseous line Note: Item 2 relates to a child in shock. In many life-threatening conditions venous cannulation is difficult. It is important to obtain vascular access very quickly and therefore intraosseous infusion is recommended. Theme : Syndromes associated with congenital heart disease Pediatric extended match questions Dr samed alsalmi
  • 6. Cardiology 2 A. Charcot-Marie- Tooth syndrome B. Down syndrome C. Fragile X syndrome D. Marfan's syndrome E. Noonan syndrome F. Sturge-Weber syndrome G. Tetrology of Fallot H. Tourette syndrome I. Turner's syndrome J. William's syndrome For each of the cardiac lesions described below, choose the most commonly associated syndrome from the above list of options. Each option may be used once or not at all. 1) Supravalvular aortic stenosis J. William's syndrome Note: William's syndrome 7 is characterized by short stature, characteristic facies, supravalvular aortic stenosis, mild to moderate learning difficulties and transient neonatal hypercalcemia. 2) Coarctation of Aorta I. Turner's syndrome Note: Turner's syndrome 7 This is characterized by 45, X genotype, ovarian dysgenesis leading to infertility, short stature, webbing of the neck, wide carrying angles and wide spaced nipples. However they have normal intellectual development. 3) Dilatation of aortic root/aortic regurgitation D. Marfan's syndrome Note: Marfan's syndrome 7 This is an autosomal dominant disorder. The clinical features are tall stature, arachno-dactyly, high arched plate and increase in length of the lower segment of the body compared to the upper segment. The cardiac manifestations include dilated aortic root, aortic incompetence, mitral valve prolapse and mitral incompetence. 4) Cardiac cushion defects (leading to ASD, VSD) B. Down syndrome Note: Down syndrome 7 Features include characteristic facies, hypotonia, severe learning difficulties and small stature. About 40% of patients have cardiac anomalies mainly endocardial cushion defects leading to ASD and VSD. 5) Infundibular pulmonary stenosis G. Tetrology of Fallot Note: Tetrology of Fallot This is a cyanotic heart disease and the cardinal features include: 1. Infundibular pulmonary stenosis 2. VSD 3. Right ventricular hypertrophy 4. Over-riding of the aorta Comments: Charcot-Marie-Tooth syndrome 7 Autosomal dominant peroneal muscular dystrophy. Fragile X syndrome 7 Moderate learning difficulty, macrocephaly, characteristic facies (long face, large ears, prominent mandible and forehead) Noonan syndrome7 Facies, mild learning difficulties, short webbed neck, short stature and congenital heart disease (pulmonary valvular stenosis, ASD, left ventricular hypertrophy) Sturge-Weber syndrome 7 Haemangiomas in the distribution of trigeminal nerve and in the brain. Sometimes can have intractable epilepsy. Tourette syndrome 7 Tics, compulsive utterances of obscene words (coprolalia) Theme : Emergency treatments Pediatric extended match questions Dr samed alsalmi
  • 7. Cardiology 3 A. Adenosine B. Adrenaline C. Atropine D. DC shock E. Dobutamine F. Diving reflex G. Endotracheal intubation H. Intraosseous line I. IV Morphine J. Naloxone Select the most appropriate emergency treatment for the following children: 1) A 5 year old boy is brought to the hospital with 15% scalds to his chest. I. IV Morphine Note: Item 1 relates to a child with serious burns. Children who have been burnt are in severe pain and therefore IV Morphine is the analgesic of choice. 2) A 14-year-old girl with a history of previous overdoses is admitted to A&E apnoeic and unconsciousness. ECG shows ventricular fibrillation. CPR is commenced. D. DC shock Note: Item 2 describes a child in ventricular fibrillation. This is uncommon in childhood although may occur as a result of tricyclic antidepressant overdose and hypothermia. If the arrest is witnessed a precordial thump is carried out otherwise electrical de-fibrillation at 2 joules per kilogram. 3) A term baby is born in poor condition. Apgar scores 3 at 1 minute and 5 at 5 minutes. CPR is commenced. At 10 minutes he remains bradycardic. B. Adrenaline Note: Item 3 describes a baby born in poor condition. A bradycardia in an unstable newborn requires oxygenation, ventilation and cardiac compressions. IV adrenaline is administered as Atropine is ineffective in this age group. Theme : Genetics - cardiac abnormalities in genetic disorders Pediatric extended match questions Dr samed alsalmi
  • 8. Cardiology 4 A. Angelman's syndrome B. Beckwith-Wiedemann syndrome C. Congenital Rubella syndrome D. Down syndrome E. Foetal alcohol syndrome F. Glycogen storage disease G. Marfan's syndrome H. Noonan's syndrome I. Turner's syndrome J. Williams syndrome Match each of the following cardiovascular abnormalities to the single most likely associated genetic disorder. 1) Dilation of the aorta with aneurysms. G. Marfan's syndrome Note: In Marfan's syndrome dilatation of the ascending aorta is often seen with or without aneurysms. Less commonly the thoracic abdominal aorta or pulmonary arteries are affected with secondary aortic regurgitation and mitral valve prolapse. Theme : Congenital cardiac defects A. Angelman's syndrome B. Beckwith-Wiedemann syndrome 2) Supra-valvular aortic stenosis. J. Williams syndrome Note: In Williams syndrome supra-valvular aortic stenosis is the most common cardiac lesion. Septal defects also occur as well as peripheral branch pulmonary artery stenosis. 3) Pulmonary stenosis. H. Noonan's syndrome Note: In Noonan's syndrome pulmonary valve stenosis due to a dysplastic or thick valve is seen often associated with left ventricular hypertrophy. Branch stenosis of the pulmonary artery also is found in Noonan's syndrome. Pediatric extended match questions Dr samed alsalmi
  • 9. Cardiology 5 C. Congenital Rubella syndrome D. Down syndrome E. Foetal alcohol syndrome F. Glycogen storage disease G. Marfan's syndrome H. Noonan's syndrome I. Turner's syndrome J. Williams syndrome Match each of the following cardiovascular abnormalities to the single most likely associated disorder. 1) Endocardial cushion defect. D. Down syndrome Note: In Down syndrome approximately 40% of children have a congenital heart disease. The most common being endocardial cushion defects although VSDs, ASDs and PDA also occur. 2) Coarctation of the aorta. I. Turner's syndrome Note: In Turner's syndrome cardiac defects are common. 30% include bicuspid aortic valves with the second most common heart defect being coarctation of the aorta. Aortic stenosis, mitral valve prolapse and hypertension are also found. 3) Septal defects. E. Foetal alcohol syndrome Note: In foetal alcohol syndrome individuals have poor growth, developmental delay and usually characteristic facial features including microcephaly and a short smooth philtrum. The most common cardiac lesion in these children are septal defects primarily ventricular septal defects. Theme : BREATHLESSNESS A. Asthma B. Hyperventilation C. Tuberculosis D. Cystic Fibrosis E. Pneumocystis carinii Pediatric extended match questions Dr samed alsalmi
  • 10. Cardiology 6 F. Ventricular septal defect G. Gastroesophageal reflux H. Atrial septal defect I. Bronchiolitis J. Mitral stenosis For each of these patients with breathlessness, select the most likely diagnosis 1) A thirteen-year-old girl who has intermittent episodes of breathlessness, which tend to occur in crowded shops. She feels the need to take deep breaths and then breathes very quickly, complaining of pins & needles around her mouth and in her hands. Her chest is clear and her blood gases show a normal pO2 and low pCO2. B. Hyperventilation Note: This description is one of anxiety. The blood gas picture is one of hyperventilation. 2) A three-year-old boy who presents with worsening cough and breathlessness of 3 weeks’ duration. His mother was an intravenous drug abuser. He has always been prone to infections. When he was 2 years old he had chicken pox for 4 weeks. On examination, he has an emaciated appearance, his weight is below the 0.4th centile, he has a temperature of 37.6°C and he has generalized crepitations on auscultation of his chest. A blood count shows severe lymphopenia. E. Pneumocystis carinii Note: This has resulted from congenitally acquired HIV. PCP has an insidious onset and often there are no chest signs in children. Lymphopenia is consistent. Treatment is with septrin or nebulised pentamidine as second line. 3) A 4-month-old baby has not gained much weight since birth and only takes small milk feeds, as he appears to become breathless on feeding. He is tachypnoeic, sweaty and has a tachycardia. His liver is enlarged and he has a harsh grade 2- pansystolic murmur at the left lower sternal edge. F. Ventricular septal defect Note: Poor feeding is a symptom of heart failure in babies. A VSD has a pan systolic murmur and if large may have a lower grade as there is less resistance to flow. Hepatomegaly is an early sign of heart failure in infants. Theme : CONGENITAL HEART DISEASE A. Ostium secundum atrial septal defect B. Ventricular septal defect C. Transposition of the great arteries D. Total anomalous pulmonary venous drainage E. Atrioventricular septal defect F. Patent ductus arteriosus G. Pulmonary valve stenosis Pediatric extended match questions Dr samed alsalmi
  • 11. Cardiology 7 H. Coarctation of the aorta I. Tetralogy of Fallot J. Hypoplastic left heart syndrome Which of the above is the most likely diagnosis in the following cases. 1) A 13-year-old girl is referred for evaluation of her short stature. She is prepubertal. On auscultation she has an ejection systolic murmur in the second and third left intercostals spaces radiating to the back, but is a symptomatic. G. Pulmonary valve stenosis Note: The murmur describes pulmonary stenosis, which could also be a left peripheral pulmonary stenosis. She is short and has delayed puberty and coupled with the cardiac findings would suggest Noonan’s syndrome. 2) A 7-week-old infant presents with breathlessness on feeding and failure to thrive. On examination his femoral pulses are difficult to feel but present. Chest radiograph shows cardiomegaly and increased vascular markings. H. Coarctation of the aorta Note: Absent or weak femoral pulses suggest coarctation. Remember association with Turner’s syndrome. 3) An infant is seen for his 6-week check and found to have a loud ejection systolic murmur in the third left intercostal space and a single second heart sound on examination. There is no obvious cyanosis but a suggestion of mild desaturation. On the chest X ray there is a concavity on the left heart border and decreased pulmonary vascular markings. I. Tetralogy of Fallot Note: Tetralogy of Fallot may present later than in the neonatal period. The ejection systolic murmur is from the infundibular stenosis. The desaturation results from the right to left shunt across the VSD. Theme : Congenital heart disease A. Atrioventricular septal defect B. Coarctation of the aorta C. Hypoplastic left heart syndrome D. Ostium secundum atrial septal defect E. Patent ductus arteriosus F. Pulmonary valve stenosis G. Tetralogy of Fallot H. Total anomalous pulmonary venous drainage I. Transposition of the great arteries Pediatric extended match questions Dr samed alsalmi
  • 12. Cardiology 8 J. Ventricular septal defect Which is the most likely diagnosis in the following cases? 1) An infant is found profoundly cyanosed and lethargic in his cot on day 2. On auscultation there is a soft systolic murmur heard inconsistently at the left sternal edge and a single second sound. The chest X ray shows a narrow upper mediastinum, hypertrophied right ventricle and increased pulmonary vascular markings. The ECG shows a normal neonatal pattern. I. Transposition of the great arteries Note: Cyanosis on the second day is suggestive of a duct-dependent lesion. The rest of the answer describes TGA 2) A 3-week-old premature infant born at 27 weeks gestation remains ventilated following surfactant deficient respiratory distress syndrome. On auscultation of his chest a systolic murmur is heard at the left sternal edge and pulses are very easy to feel. There is pulmonary plethora on chest X ray. E. Patent ductus arteriosus Note: PDA is a relatively common problem in premature babies. The left to right shunt results in excess blood flow through the lungs and frequently oxygen dependency and difficulty in weaning from the ventilator. A loud systolic murmur radiating to the back with easily palpable pulses are typical. 3) A 7-year-old boy is examined for a chest infection. An incidental finding of a short systolic murmur with fixed splitting of the second heart sound is detected. His blood pressure is normal and all pulses are normal. D. Ostium secundum atrial septal defect Note: Ostium primum defects are unlikely to present incidentally but rather with heart failure or pulmonary hypertension. Fixed splitting is typical of ostium secundum defects Pediatric extended match questions Dr samed alsalmi
  • 13. Endocrine Theme : Endocrine A. Addison's disease B. Congenital adrenal hyperplasia C. Cushing's syndrome D. Diabetes insipidus E. Diabetes mellitus F. Hyperthyroidism G. Hypoparathyroidism H. Hypothyroidism I. Primary aldosteronism J. Septo-optic dysplasia Select the most likely diagnosis from the list above that would explain the presentations of the following patients: 1) An 11 year old boy who was previously fit and well presents to the GP with obesity and purple abdominal striae. His blood pressure is 140/90. C. Cushing's syndrome 2) A breast-fed baby presents with poor feeding. He has a hoarse voice and coarse facial features. Examination of the skull reveals wide sutures and a large anterior fontanelle. J. Septo-optic dysplasia H. Hypothyroidism 3) A 7 year old girl with moderate learning difficulties presents with dry skin, alopecia and mucocutaneous candidiasis. H. Hypothyroidism G. Hypoparathyroidism Comments: Cushing's syndrome is a disorder due to high circulating cortisol levels either exogenous or endogenesis in origin. Clinical features include arrested growth, trunk obesity, hirsuitism, muscle wasting and mood changes. Purple striae are often seen. Laboratory investigations may reveal polycythaemia, abnormal glucose tolerance and high cortisol levels. 2-Congenital hypothyroidism is relatively common. Affected babies may have coarse facies, dry skin and a hoarse cry. Other features include hypotonia, umbilical hernia and constipation and prolonged jaundice. Blood tests reveal low T4 levels and a high TSH. In some infants brain development may be irreversibly damaged before birth however early detection may avoid permanent neurology sequelae. 3-Hypoparathyroidism – in the neonatal period transient hypoparathyroidism occurs resulting in hypocalcaemia, possibly convulsions or apnoeic episodes. Hypoparathyroidism is rare. Features include headaches, vomiting, photophobia, cataracts, poor dentition and chronic diarrhoea. Investigations will 1/ X X Pediatric extended match questions Dr samed alsalmi
  • 14. confirm low calcium and high phosphate levels. Theme : Endocrine A. Addison's disease B. Congenital adrenal hyperplasia C. Cushing's syndrome D. Diabetes insipidus E. Diabetes mellitus F. Hyperthyroidism G. Hypoparathyroidism H. Hypothyroidism I. Primary aldosteronism J. Septo-optic dysplasia Select the most likely diagnosis from the list above that would explain the presentations of the following patients: 1) A 14 year old girl presents with weight loss. She denies any dieting or use of laxatives. She also complains of polydipsia and has recently started wetting the bed. E. Diabetes mellitus 2) A 6 year old boy presents with fatigue and weight loss. He complains of abdominal pain and disturbed bowel habit. He is found to have a low blood sugar. A. Addison's disease 3) A 6 month old baby with histiocytosis presents with vomiting weight loss and signs of dehydration. D. Diabetes insipidus Comments: 1-Diabetes mellitus results from insulin deficiency. Symptoms are characteristic and early features being polyuria, nocturnal enuresis, thirst, lethargy, weight loss and anorexia. Late symptoms include vomiting, abdominal pain and shock. 2-Addison's disease or adrenocortical failure has an auto-immune basis and may occur in association with other endocrine problems such as diabetes, hypoparathyroidism and thyroiditis. Clinical features include weakness, weight loss and increased pigmentation. Children may also have hypotension and hypoglycaemia. 3-Diabetes insipidus may be idiopathic or as a result of tumours, histiocytosis etc. It is due to a lack of anti-diuretic hormone and results in polyuria, polydipsia hypothermia, weight loss, constipation and poor growth. Treatment requires DDAVP. Pediatric extended match questions Dr samed alsalmi
  • 15. Hematology 1 Hematology Theme : Haematological Disorders A. Aplastic anaemia B. Beta Thalassaemia Major C. Elliptocytosis D. Glucose 6 Phosphate dehydrogenase deficiency E. Haemochromatosis F. Haemophilia A G. Immune thrombocytopenia H. Polycythaemia I. Pyruvate Kinase deficiency J. Sickle Cell anaemia Select the most appropriate haematological diagnosis from the above list that best explains the following scenarios. 1) May be a consequence of parvovirus infection. A. Aplastic anaemia 2) May present with a characteristic skin pigmentation and is secondary to multiple transfusions. E. Haemochromatosis 3) Occurs as a result of a congenital defect of the red cell membrane. C. Elliptocytosis Comments: Aplastic anaemia or bone marrow aplasia results in anaemia, neutropenia and thrombocytopenia. It may be congenital for example Fanconi's anaemia or acquired for example as a result of infection with for example Parvovirus infectio or due to drugs such as Chloramphenical, Sulphonamides or Chemotherapy. Haemochromatosis occurs when the structure or function of organs is deranged because of excessive Iron storage. Patients may exhibit bronzed skin and may develop Hepatitis or diabetes. Treatment requires chelation. Elliptocytosis is similar to spherocytosis in that the red blood cells exhibit a defect in the membrane. In Elliptocytosis the red blood cells are oval and the condition is benign. Haemolysis occurs with some forms of the disease. Theme : Malignancy Pediatric extended match questions Dr samed alsalmi
  • 16. Hematology 2 A. Acute leukaemia B. Ewing's Osteosarcoma C. Lymphoma D. Melanoma E. Neuroblastoma F. Retinoblastoma G. Sarcoma H. Thyroid cancer I. Testicular seminoma J. Wilm's tumour Select the most appropriate chocie from thee above list for the following scenarios. 1) A condition which affects children under the age of 3, associated with a genetic locus on Chromosome 13. F. Retinoblastoma 2) A tumour known that is recognised to spontaneously regress. E. Neuroblastoma 3) A jaundiced baby on the postnatal ward is described as having blueberry muffin spots on the skin. A. Acute leukaemia Comments: Retinoblastoma is a tumour in the posterior part of the retina. It is associated with a deletion of the long arm of chromosome 13. The average age of diagnosis is 8 months for bilateral involvement ands 24 months for unilateral involvement. It may present with leucocoria, visual loss or a squint. A Neuroblastoma may arise anywhere where neural crest cells migrate. The median age of diagnosis is approximately 2. In babies less then 1 year of age or in the early stages of tumour development spontaneous regression has been known to occur. Acute lymphoblastic leukaemia is the commonest malignancy of childhood. Initial symptoms may be non-specific for example lethargy and irritability. Ultimately there is bone marrow failure which presents with pallor, thrombocytopenia and neutropenia. Diagnosis is made by seeing blasts on a peripheral smear There may be anaemia, thrombocytopenia and the white cell count may be raised or low. Babies born with congenital leukaemia have lesions over the skin which are described as being similar to blueberry muffins spots Theme : Haematological Disorders A. Aplastic anaemia Pediatric extended match questions Dr samed alsalmi
  • 17. Hematology 3 B. Beta Thalassaemia Major C. Elliptocytosis D. Glucose 6 Phosphate dehydrogenase deficiency E. Haemochromatosis F. Haemophilia A G. Immune thrombocytopenia H. Polycythaemia I. Pyruvate Kinase deficiency J. Sickle Cell anaemia Select the most appropriate haematological diagnosis from the above list that best explains the following scenarios. 1) May be exacerbated by exposure to anti-malarial therapy D. Glucose 6 Phosphate dehydrogenase deficiency 2) Characterised by the overgrowth of bones of the face. B. Beta Thalassaemia Major 3) May be an associated feature of congenital cyanotic heart disease. H. Polycythaemia Comments: Glucose-6-phosphate dehydrogenase deficiency is an episodic haemolytic anaemia which can be exacerbated by certain drugs, in particular anti-malarial therapy. There is a racial difference in prevalence. Laboratory findings include low levels of G6PD and Heinz bodies present in the red blood cells. Thalassaemia is a group of hypochromic anaemia's due to abnormalities of globin chain synthesis. Regular blood transfusions are necessary and clinical features include compensatory hypertrophy of erythropoietic tissue in medullary and extra medullary spaces. For example liver, spleen and the marrow in the face which results in characteristic facies. Polycythaemia exists when red blood cell count, Haemoglobin and haematocrit exceed the upper limits of normal and may co-exist with chronic oxygen desaturation states, for example cyanotic cardiovascular and pulmonary disease. Theme : Bleeding disorders A. Antithrombin III deficiency B. Congenital afibrinogenaemia Pediatric extended match questions Dr samed alsalmi
  • 18. Hematology 4 C. Haemophilia A D. Haemophilia B E. Haemophilia C F. Idiopathic thrombocytopenic purpura G. Protein C deficiency H. Protein S deficiency I. Vitamin K deficiency J. Von Willebrands disease Choose the most appropriate diagnosis from the above list that best describes the following coagulation abnormalities. 1) An autosomal dominant disorder, characterised by thrombotic events. A. Antithrombin III deficiency Note: Antithrombin 3 deficiency prolonged in occlusion of blood vessels with platelet plugs resulting in thrombotic events. 2) Prolonged partial thromboplastin time, normal Prothrombin time. factor 8 deficiency C. Haemophilia A Note: Haemophilia A results due to low factor 8 levels. The severity of the illness depends on the level of activity and patients often present with haemarthrosis. 3) An autosomal dominant disorder associated with a prolonged bleeding time. J. Von Willebrands disease Note: describes Von Willebrand's disease an autosomal dominant condition which results in prolonged bleeding due to low Von Willebrand factor, which is needed for platelet aggregation. There is a prolonged bleeding time and treatment with FFP is sometimes required. Theme : Bleeding disorders A. Antithrombin III deficiency B. Congenital afibrinogenaemia C. Haemophilia A D. Haemophilia B Pediatric extended match questions Dr samed alsalmi
  • 19. Hematology 5 E. Haemophilia C F. Idiopathic thrombocytopenic purpura G. Protein C deficiency H. Protein S deficiency I. Vitamin K deficiency J. Von Willebrands disease Choose the most appropriate diagnosis from the above list that best describes the following coagulation abnormalities. 1) Normal Prothrombin time. Elevated thromboplastin time, factor 9 deficiency. D. Haemophilia B Note: Haemophilia B also known as Christmas disease is an X linked recessive disorder. It is as a result of low Factor 9 levels. 2) Prolonged Prothrombin time and partial thromboplastin time, normal bleeding time. I. Vitamin K deficiency Note: Item 2 describes Vitamin K deficiency which is necessary for carboxylation of factors 2, 7 ,9 and 10. 3) Bruising and petechiae over lower extremities with low platelets. F. Idiopathic thrombocytopenic purpura Note: fits the diagnosis of idiopathic thrombocytopenic purpura. A condition associated with mucocutaneous bleeding and petechiae often secondary to viral infections and the platelet count is less than 20 x 10/9. Treatment options include steroids and gammaglobulin. Theme : Malignancy A. Acute leukaemia B. Ewing's Osteosarcoma C. Lymphoma D. Melanoma E. Neuroblastoma Pediatric extended match questions Dr samed alsalmi
  • 20. Hematology 6 F. Retinoblastoma G. Sarcoma H. Thyroid cancer I. Testicular seminoma J. Wilm's tumour Select the most appropriate choice from the above list for the following scenarios 1) A 3 year old with pallor, bruising and hepatosplenomegaly. A. Acute leukaemia 2) A 16 year old with shortness of breath and night sweats and a cervical lymphnode. C. Lymphoma 3) A 4 year old known to have a deletion of Chromosome 11 presents with an abdominal mass. J. Wilm's tumour Comments: Acute lymphoblastic leukaemia is the commonest malignancy of childhood. Initial symptoms may be non-specific for example lethargy and irritability. Ultimately there is bone marrow failure which presents with pallor, thrombocytopenia and neutropenia. Diagnosis is made by seeing blasts on a peripheral smear There may be anaemia, thrombocytopenia and the white cell count may be raised or low. Babies born with congenital leukaemia have lesions over the skin which are described as being similar to blueberry muffins spots. Lymphoma is divided in to Hodgkin's and Non-Hodgkin's lymphoma. It arises in lymph nodes and enlarged nodes may be firm and non-tender. Mediastinal involvement may cause a chronic cough or bronchial / tracheal compression. Symptoms include night sweats, persistent fever and weight loss . Wilm's tumour is a solitary growth in a part of the kidney. It is associated with deletions of chromosome 11 (the probable location of the tumour suppression gene). It may present with an abdominal mass or haematuria. It is also associated with genital urinary anomalies, aniridia, hemihypertrophy and Beckwith-Wiedemann syndrome. Theme : Diagnosis of neonatal jaundice. A. ABO incompatibilty B. Breast milk jaundice C. Cephalhematoma D. Drug allergy E. Extrahepatic biliary atresia F. Galactosaemia G. G6PD Deficiency H. Neonatal sepsis I. Physiological jaundice J. RH Incompatibility Pediatric extended match questions Dr samed alsalmi
  • 21. Hematology 7 For each of these jaundiced babies below choose from the list above the single most likely diagnosis. Each option may be chosen more than once or not at all. 1) A mother has blood group A Rhesus positive. Her baby is blood group B Rhesus negative. A. ABO incompatibilty Note: This mother would produce antibodies to the baby's B blood group and hence haemolysis with jaundice occurs. 2) A neonate is severely jaundiced with reducing substances noted on urine dipstick. F. Galactosaemia Note: A typical picture of galactossaemia. In the newborn period, infants present with an acute encephalopathy. In untreated patients, there is severe liver disease, mental retardation, epilepsy and choreoathetosis. 3) A 4 day old baby is well but has a tinge of jaundice. I. Physiological jaundice Note: Common in the first week of life. 4) A breast fed 3 week old baby has mild jaundice but is gaining weight satisfactorily. B. Breast milk jaundice Note: Common from the 2nd to 5th days of life. 5) A 12 day old baby is jaundiced with pale stools. E. Extrahepatic biliary atresia Note: Biliary atresia. is a progressive inflammatory process that begins very soon after birth. On average, there is one case of biliary atresia out of every 15,000 live births. Females are affected slightly more often than males. In the United States, approximately 300 new cases are diagnosed each year. Pediatric extended match questions Dr samed alsalmi
  • 22. Infectious Diseases 2) Contra-indications to Vaccines A. BCG B. Diphtheria Tetanus Polio C. Hepatitis B D. Haemophilus Influenzae B E. Influenza F. Measles Mumps Rubella G. Polio H. Rubella I. Tetanus J. Tuberculin Which of the vaccines listed above would be contra-indicated in the following scenarios: 1) DIARRHOEA A. Coeliac disease B. Crohns disease C. Ulcerative colitis D. Cows milk protein intolerance E. Toddlers diarrhoea F. Cystic fibrosis G. Lactose intolerance H. Irritable bowel syndrome I. Abdominal migraine J. Haemolytic uraemic syndrome For the following children presenting with diarrhoea which is the most likely diagnosis: 1) An 11 month infant presents with chronic diarrhoea and failure to thrive since breast feeding was discontinued at 9 months. He has been treated by his GP with antibiotics on two occasions for chest infections. Initial investigations reveal low serum potassium, chloride and metabolic alkalosis. F. Cystic fibrosis Note: Failure to thrive, chest infections and diarrhoea (the latter results from failure of pancreatic exocrine function) are indicative of CF. The abnormal biochemistry is caused by salt loss and may be described as pseudo-bartters syndrome 2) A 15 year old boy presents with delayed puberty and short stature. He gives a history of intermittent abdominal pain and diarrhoea. He recently presented to his family doctor with a painful red swelling on his shin which has now resolved. B. Crohns disease Note: Crohn’s disease may present in this way, the skin lesions are erythema nodosum which are associated with inflammatory bowel disease. 3) A 8 month old infant has recurrent diarrhoea. Mother dates the start of symptoms from the an episode of gastroenteritis. The diarrhoea is explosive in nature. G. Lactose intolerance Note: Secondary lactose intolerance is a known sequelae of viral gastroenteritis. It is not prevented by regrading milk following the episode of viral infection. Pediatric extended match questions Dr samed alsalmi
  • 23. 1) Contra-indication if known to have a hypersensitive reaction to egg. E. Influenza Note: relates to hypersensitivity to egg, which contra-indicates the Influenza vaccine as there is residual egg protein present. 2) Contra-indication if known to have an allergy to gelatin. F. Measles Mumps Rubella Note: refers to contra-indications to MMR, which include children with allergies to Gelatin, Neomycin or kanamycin as well as children with untreated malignant disease or altered immunity. Those receiving immuno-suppressive drugs or radiotherapy are also contraindicated. Children who have received another live vaccine by injection within 3 weeks should not be given MMR vaccine. MMR should also not be given within 3 months of receiving an Immunoglobulin injection. 3) Contra-indication if known to have an allergy to kanamycin. F. Measles Mumps Rubella Note: refers to contra-indications to MMR, which include children with allergies to Gelatin, Neomycin or kanamycin as well as children with untreated malignant disease or altered immunity. Those receiving immuno-suppressive drugs or radiotherapy are also contraindicated. Children who have received another live vaccine by injection within 3 weeks should not be given MMR vaccine. MMR should also not be given within 3 months of receiving an Immunoglobulin injection. 3) Childhood chest infections A. Chlamydia B. Cytomegalovirus C. Group B Streptococcus D. Haemophilus Influenza E. Mycoplasma pneumonia F. Parainfluenza G. Respiratory syncytial virus H. Rhinovirus I. Staphylococcus aureus J. Ureaplasma Select the most appropriate pathogen from the above list that would account for the following presentations: 1) The commonest cause of pneumonia in the 2 month - 6month age group. G. Respiratory syncytial virus 2) A 9 year old boy presents with a cough and fever. Chest x-ray demonstrates a reticular nodular pattern with high adenopathy and a small pleural effusion. E. Mycoplasma pneumonia 3) A 5 year old boy presents with painful red swellings of his shins. Examination of his throat reveals an exudative tonsillitis. Pediatric extended match questions Dr samed alsalmi
  • 24. C. Group B Streptococcus Comments: The commonest cause of pneumonia in the 2month – 6 month age group is Respiratory syncytial virus, which results in bronchiolitis and occurs in epidemics. Mycoplasma pneumonia mainly affects older children (between 5 and 15 years) resulting in community acquired pneumonias. Tonsillitis can occur at any age although is frequently seen in the 4-7 age group. The majority of episodes are caused by viruses although beta haemolytic streptococcal infection is also responsible and has systemic reactions such as erythema nodosum, rheumatic fever and glomerular nephritis. 4) Childhood Diarrhoea A. Adenovirus B. Cytomegalovirus C. Epstein-Barr virus D. E. Coli E. Giardia Lamblia F. Hepatitis A G. Norwalk virus H. Rotavirus I. Salmonella species J. Staphylococcus aureus Select one option from the list above that is most suitable for the following patients 1) Accepted as the most common cause of infectious diarrhoea in children in the developed society. H. Rotavirus Note: Rotavirus is the most common virus responsible for diarrhoea worldwide. It is a double stranded RNA virus, especially prevalent in the winter months. It causes fever and watery diarrhoea. 2) Can follow ingestion of dust containing dried faecal material. F. Hepatitis A Note: Hepatitis A is an RNA virus typically transmitted by the faecal-oral route. 3) Attaches the small intestinal border and releases extoxin. D. E. Coli Note: E-coli and enteropathogen is one of the bacterial causes of diarrhoea. Many of its effects are a consequence of an exotoxin. It should be suspected if there is a history of travel or a history of ingestion of poorly prepared food. Bacterial diarrhoea usually results in bloody diarrhoea. Pediatric extended match questions Dr samed alsalmi
  • 25. 5) Urinary Tract Disease A. Haemolytic uraemic syndrome B. Henoch Schonlein purpura C. Nephrotic syndrome D. Polycystic kidneys E. Pyelonephritis F. Renal Calculi G. Renal tubular acidosis H. Renal vein thrombosis I. Systemic lupus erythematosus J. Wilm's tumour Select the most appropriate diagnosis from the above list of options that would best explain the following cases: 1) A 2 year old boy is admitted with a history of bloody diarrhoea, abdominal pain and puffiness around his eyes. He is found to be slightly jaundiced. His urea is 12.6 and Creatinine 163 mmol / liter. A. Haemolytic uraemic syndrome Note: describes a child with haemolytic uraemic syndrome, which is of unknown aetiology. Various agents have been implicated including E.coli (0157) Salmonella, Shigella and viruses. It usually presents with a prodromal symptoms of an upper respiratory tract infection or gastroenteritis and later develops in to pallor, oliguria and hypertension. Laboratory investigations may reveal microangiopathic haemolytic anaemia, thrombocytopenia and urinalysis may reveal proteinuria. 2) A 9 day old baby is admitted with a 2 day history of vomiting and diarrhoea. During the admission haematuria is noted and he has had 2 convulsions. A mass is palpable in the left hypochondrium. H. Renal vein thrombosis Note: describes a child with renal vein thrombosis. In neonates it is associated with dehydration, asphyxia, shock or sepsis. Manifestations include sudden onset of haematuria and identification of an enlarged kidney. If both kidneys are involved it may result in acute renal failure. 3) A 5 year old girl presents with puffiness around the eyes. Urinalysis confirms proteinuria with a trace of blood. C. Nephrotic syndrome Note: relates to a child with Nephrotic syndrome. This occurs when there is proteinuria resulting in hypoalbuminaemia and oedema with an unknown aetiology. Peri-orbital or dependant oedema is usually noted first. There may also be a history of abdominal pain, vomiting and diarrhoea. The child needs to be monitored carefully as hypovolaemia and circulatory collapse. Steroids are the treatment of choice with careful management of fluids. 6) NEONATAL JAUNDICE A. Physiological jaundice B. Biliary atresia C. Hypothyroidism D. Rhesus incompatibility E. Congenital spherocytosis F. Congenital cytomegalo virus infection G. Galactosaemia H. Glucose-6-phosphate dehydrogenase deficiency I. Cystic fibrosis J. Fructose intolerance Pediatric extended match questions Dr samed alsalmi
  • 26. For each of the following jaundiced babies, select the most likely cause: 1) A 13 day old infant who was noted to have an umbilical hernia and has very dry skin presents with jaundice. She is a floppy baby. Her bilirubin is checked and is found to be elevated and mainly unconjugated. The community midwife has been unable to gain access to the home for the last week. C. Hypothyroidism Note: Umbilical hernia, dry skin, hypotonia and jaundice are features of congenital hypothyroidism. 2) A caucasian infant has required surgery at the regional neonatal unit for meconium ileus and has developed jaundice. The serum conjugated bilirubin is 65 micromol/L. The diagnosis is eventually confirmed from the neonatal screening tests. I. Cystic fibrosis Note: Meconium ileus and jaundice may be presenting features of CF in the neonatal period. The screening test is immunoreactive trypsin. 3) A 12 day old male baby has a conjugated and unconjugated hyperbilirubinaemia. He is breast fed and has become more disinterested in feeds. He was investigated for sepsis and blood cultures demonstrated E-coli septicaemia, urine cultures were clear. His clotting is deranged. G. Galactosaemia Note: Galactosaemia typically presents around 2 weeks of age with jaundice. E coli septicaemia is a feature. Treatment is by removing galactose from the diet. Cataracts are a later feature even if treatment instituted early. 7) Viral infections A. Measles B. Rubella C. Chicken Pox D. Herpes Simplex E. Mumps F. Glandular fever G. Pertussis H. Polio myelitis I. Hepatitis A For each description below choose the single most likely viral infection from the list of options. 1) May result in Giant cell pneumonia. A. Measles Note: Measles infection is uncommon due to world-wide immunisation. A child with measles usually has a fever, upper respiratory tract symptoms and a morbilliform rash. Serious complications include a giant cell pneumonia and encephalitis (SSPE). 2) May result in a Keratoconjuctivitis. D. Herpes Simplex Pediatric extended match questions Dr samed alsalmi
  • 27. Note: Herpes simplex infection may be transmitted verdantly to an infant from their mother's genital tract. It may cause isolated skin lesions, a Keratoconjunctivitis or a paronychia. More seriously it is also responsible for encephalitis. 3) Caused by an RNA virus with no known carrier state. I. Hepatitis A Note: Hepatitis A, which is caused by an RNA virus, is usually transmitted by the oral route. It has an incubation period of between 15-50 days and treatment is usually symptomatic only. 8) Urinary Tract Disease A. Haemolytic uraemic syndrome B. Henoch Schonlein purpura C. Nephrotic syndrome D. Polycystic kidneys E. Pyelonephritis F. Renal Calculi G. Renal tubular acidosis H. Renal vein thrombosis I. Systemic lupus erythematosus J. Wilm's tumour Select the most appropriate diagnosis from the above list of options that would best explain the following cases: 1) An 11 year old with a previous history of chronic glomerulonephritis presents with bruising and epistaxis. A full blood count confirms a pancytopenia. I. Systemic lupus erythematosus Note: describes a child with glomerulonephritis and bone marrow failure suggestive of a connective tissue disease such as SLE. 2) A 14 year old boy with a history of recurrent urinary tract infections present with severe abdominal pain radiating to his back. Dysuria and haematuria. F. Renal Calculi Note: relates to Urolithiasis, Renal Calculi. Children present with abdominal pain, voiding abnormalities, dysuria, haematuria may be present. Avoidance of dehydration is important. Treatment may require lithotripsy. 3) A 1 year old girl with a 3 month history of vomiting is investigated for failure to thrive. She is found to be mildly acidotic. G. Renal tubular acidosis Note: Renal tubular acidosis is the answer to Item 3. It is a clinical state of systemic hyperchloraemic acidosis resulting from impaired urinary acidification. Three types exist. Type 1 distal RTA, type 2 proximal RTA and type 4 mineralocorticoid deficiency. Type 3 is thought to be a variant of type 1. Types 1, 2 and 4 each have several causes. Children with isolated forms of proximal or distal commonly present with Pediatric extended match questions Dr samed alsalmi
  • 28. growth failure; gastrointestinal symptoms are also common. Nephro-calcinosis and hypercalciuria may complicate distal RTA and goals of treatment are to correct the acidosis and to maintain normal Bicarbonate and Potassium levels. 9) Mucocutaneous infections and infestations A. Enterobiasis vermicularis B. Chlamydia C. Candidiasis D. Tinea pedis E. Pityriasis rosea F. Scabies G. Ringworm H. Lichen sclerosis I. Napkin dermatitis J. Lichen planus For each of the following children who present with an itch, select the most appropriate diagnosis from the list of options: 1) A child complains of an itchy rash over his arms. His brother and sister have similar symptoms. F. Scabies Note: Scabies is caused by the mite Sarcoptes scabiei hominis. Transmission is through close body contact, the adult mites lay their eggs in burrows in the skin and it results in an eczematous rash with parotitis skin. 2) A baby has an excoriated perineal rash involving the flexures. C. Candidiasis Note: Candidiasis is caused by Candida albicans (yeast). In babies it presents as a perineal rash which usually affects the flexures. Satellite lesions may also be seen. 3) A boy has an itchy rash over the trunk with a solitary large oval lesion on the back. E. Pityriasis rosea Note: Pityriasis rosea is a benign condition of the skin resulting in oval pink / brown scaly lesions over the trunk, which are usually preceded by a herald patch (a solitary large lesion usually between 1 –10cm). No treatment is required. 10) Childhood respiratory diseases A. Asthma B. Bronchiolitis C. Croup D. Cystic fibrosis E. Diphtheria F. Epiglottis G. Pneumonia H. Influenza I. Retropharyngeal abscess J. Whooping cough For each patient below, choose the SINGLE most probable diagnosis from the above list of Pediatric extended match questions Dr samed alsalmi
  • 29. options. Each option may be used once, more than once or not at all. 1) A six month old baby presents with high fever, breathlessness, cough and feeding difficulties. Chest examination reveals dull percussion note over the right base posteriorly with bronchial breath sounds on auscultation. G. Pneumonia Note: Pneumonia may occur at any age. Patients present with fever, tachypnoea, feeding difficulties and cyanosis. Examination reveals bronchial breath sounds and crepitations. Chest X ray may show consolidation. Common organisms are pneumococcus, haemophilus, staphylococcus, mycoplasma, TB and viruses. 2) A 1 year old baby boy is wheezy, coughing, cyanosed and breathless with intercostal recession. B. Bronchiolitis Note: Acute bronchiolitis is very common in infancy. In winter epidemics of respiratory syncytial virus infection are the commonest cause. Wheeze, cough, fever and respiratory distress are common. Chest X ray shows hyperinflation. 3) A 4 year old non immunized boy presents with bouts of coughing ending in vomiting. He has an absolute lymphocytosis. J. Whooping cough Note: Whooping cough is caused by Bordetella pertussis infection. Bouts of coughing ending in vomiting, especially at night and after feeding suggest the diagnosis. The characteristic whoop, forced inspiration through a closed glottis may or may not be present. Absolute lymphocytosis is common. Complications include CNS haemorrhages, rectal prolapse and bronchiectasis. 4) A nine month old baby girl is upset and has stridor. Her voice is hoarse and has a barking cough. She has a low grade fever. C. Croup Note: Croup usually occurs in epidemics in autumn or spring. Causative viruses are Parainfluenza (types 1, 2, or 3), respiratory syncytial viruses and measles virus. Onset is over a few days; stridor is harsh and occurs only when child is upset. A barking cough, harsh voice and ability to swallow secretions are typical. 5) A 2 year old boy is very unwell. His temperature is 39°C and he is unable to swallow his secretions. F. Epiglottis Note: Acute epiglottis is due to Haemophilus influenza type B infection. It is characterised by sudden onset, high fever, continuous stridor and drooling of secretions. Intravenous antibiotics, anaesthetic support are usually indicated. Pediatric extended match questions Dr samed alsalmi
  • 30. 11) Treatment of infectious disease A. Acyclovir B. Acyclovir plus Cefotaxime C. Ampicillin plus Gentamicin D. Benzylpenicillin E. Ceftriaxone F. Ciprofloxacin G. Flucloxacillin H. Netilmicin I. Symptomatic treatment J. Teicoplanin For each case of infectious disease described below choose the single best treatment option from the list. 1) A 4 year old with a 3 day history of vomiting and diarrhoea. I. Symptomatic treatment Note: A child with a history of gastroenteritis is a very common childhood illness. Conservative treatment only is required. 2) A 2 year old child with an immune deficiency develops Chicken Pox. A. Acyclovir Note: A child who is immuno-deficient and therefore must be provided with Zoster Immune Globulin if exposed to Chicken pox through contact. If the Chicken pox develops, treatment with Acyclovir is required. 3) A 7 year old child with widespread impetigo. G. Flu-cloxacillin Note: Impetigo is a common staphylococcal skin manifestation, which is highly contagious. Most frequent sites being nostrils and the peri-oral area. Flucloxacillin is the treatment of choice. 12) Exanthem A. Staphylococcal scalded skin syndrome B. Rubella C. Measles D. Kawasaki’s disease E. Impetigo F. Scarlatina G. Infectious mononucleosis H. Henoch – Schonlein purpura I. Meningococcal infection J. Stills disease (systemic onset juvenile chronic arthritis) Match the following descriptions of rash with the illness for which they are the most typical exanthem 1) A 5 year old boy has a bright, red, punctate, erythematous rash which blanches on pressure, beginning in the axillae with some perioral pallor and relative facial sparing. The skin feels like “sandpaper”. The rash fades and desquamates on the hands and feet. A thick white exudate develops on the tongue which peels leaving a “strawberry tongue” with prominent papillae. F. Scarlatina(scarlet fever) Pediatric extended match questions Dr samed alsalmi
  • 31. Note: This description is typical of Scarlatina i.e. beta haemolytic streptococcal infection. The rash may be confused with that of Kawasaki disease. However, desquamation occurs in late in Kawasaki’s disease. 2) A maculopapular rash develops in a child with sore throat and fever who has been treated with ampicillin. G. Infectious mononucleosis Note: This is a known effect of giving ampicillin during EBV infection. 3) A 3 year old child presents with high fever for 7 days, conjunctival injection, fissuring of the lips and strawberry tongue, erythema followed by desquamation of the hands and feet, and a macular rash over the trunk with cervical lymphadenopathy. D. Kawasaki’s disease Note: There are major and minor features of Kawasaki disease. High fever and desquamation are typical. 13) Presentation of infectious disease in childhood A. Chicken Pox B. Herpes simplex C. Infectious mononucleosis D. Measles E. Mumps F. Mycoplasma G. Pertussis H. Rubella I. Tuberculosis For each presentation of infectious disease choose the single most likely diagnosis from the list of options. 1) May be complicated by cerebellar ataxia. A. Chicken Pox Note: Chicken pox is a common childhood illness caused by Varicella. Vesicles usually appear as crops over the trunk. Viral cerebellitis is a complication. 2) May present with apnoeas during infancy. G. Pertussis Note: Pertussis is caused by Bordetella pertussis. In infancy it can be dangerous often presenting with apnoeas. Signs of an upper respiratory tract infection are also common with a paroxysmal cough (whoop). Babies may also present with vomiting and cyanosis. A blood count will reveal a lymphocytosis. Complications include broncho-pneumonia and bronchiectasis as sequelae is also associated though uncommon. 3) Associated with a maculo-papular rash that typically starts on face and extends to rest of body H. Rubella Pediatric extended match questions Dr samed alsalmi
  • 32. Note: Rubella infection is uncommon now due to widespread MMR vaccination. Typically causes a mild illness which may go undetected in 25-50% of cases. Typical symptoms are transient macular papular rash that starts on the face and extends to body and a low grade fever. Complications include arthritis, encephalitis and thrombocytopenia. 14) RASHES A. Staphylococcal scalded skin syndrome B. Rubella C. Measles D. Kawasaki’s disease E. Impetigo F. Scarlatina G. Infectious mononucleosis H. Henoch – Schonlein purpura I. Meningococcal infection J. Stills disease (systemic onset juvenile chronic arthritis) Match the following descriptions of rash with the illness for which they are the most typical exanthem. 1) A salmon-coloured, reticulate macular rash develops mainly over the extensor surfaces of the limbs in a 5 year old boy with swinging temperature; hot, swollen, painful knees and left elbow and palpable spleen. The ESR is 95. The blood count, C-reactive protein and chest X-ray are normal. J. Stills disease (systemic onset juvenile chronic arthritis) Note: “Salmon-coloured” is the description used to describe the rash of Still’s disease. The distribution is not that of HSP which covers typically the buttocks and limbs and is a purpuric rash. 2) A 12 year old boy develops petechiae and papules, some of which become purpuric over his buttocks and legs, associated with painful swollen knees. There is microscopic haematuria on testing. The platelet count is normal. H. Henoch – Schonlein purpura Note: This is a description of HSP and the well recognised complication of HSP nephritis. A proportion of these patients will develop progressive nephritis and end stage renal failure. 3) A 5 day old girl has a high temperature and is irritable. She has areas of desquamation over her finger tips and in the axillae. Her carer notices that her skin blisters easily following minimal contact. A. Staphylococcal scalded skin syndrome Note: SSSS results from infection with staphylococci with the exofoliative toxin A and B. These exotoxins cause disruption to the epidermal layer by interfering with intercellular junctions. Mortality is up to 3% in children. A positive Nikolsky sign (slippage of the superficial layer of the epithelium on gentle pressure) The desquamation occurs concomitantly with the illness unlike Kawaskis disease and Kawasaki disease does not occur in this age group. There may be a history of minimal skin trauma which provides a port of entry for the organism. Pediatric extended match questions Dr samed alsalmi
  • 33. 15) DIARRHOEA A. Coeliac disease B. Crohns disease C. Ulcerative colitis D. Cows milk protein intolerance E. Toddlers diarrhoea F. Cystic fibrosis G. Lactose intolerance H. Irritable bowel syndrome I. Abdominal migraine J. Haemolytic uraemic syndrome For the following children presenting with diarrhoea which is the most likely diagnosis: 1) A 6 year old boy with Downs’s syndrome has between 3 and 4 loose stools a day. He is pale but otherwise the examination is unremarkable. When plotting his growth on a chart specific for Downs syndrome children it is clear that both height and weight have fallen across 2 centiles. A. Coeliac disease Note: Coeliac disease is more occurs more frequently in Downs’s syndrome. To screen, anti-TTG antibodies taken with serum IgA must be evaluated. 2) A 3 year old has intermittent diarrhoea with stools of varying consistency and sometimes undigested food particles. His growth is satisfactory. His mother had tried to reduce his intake of diary produce but dietary manipulation has had no impact on the symptoms. E. Toddlers diarrhoea Note: Toddler diarrhoea (thought to be a normal variant) occurs mainly in preschool children and is evidenced by undigested food observed in the faeces in a well child. Poor growth would indicate a likely pathological cause. 3) A 4 month baby girl has severe atopic eczema and is regularly reviewed in the paediatric clinic. She had now developed diarrhoea and her weight has been static in recent weeks. A full blood count shows a slight eosinophilia. D. Cows milk protein intolerance Note: CMPI is associated with eczema and eosinophilia. Cow’s milk should be excluded from the diet and reintroduced between 1-2 years if tolerated. An alternative milk source such as soy milk can be used until this is done. Pediatric extended match questions Dr samed alsalmi
  • 34. 16) Childhood infections A. Adenovirus B. Cytomegalovirus C. Epstein-Barr virus D. Escherichia coli E. Giardia lamblia F. Haemophilus influenzae type B G. Listeria monocytogenes H. Mycobacterium tuberculosis I. Mycoplasma pneumoniae J. Neisseria meningitidis K. Pseudomonas aeruginosa L. Pneumocystis carinii M. Respiratory syncytial virus N. Rotavirus O. Staphylococcus aureus P. Staphylococcus epIdermidis Q. Streptococcus agalactiae R. Streptococcus pneumoniae S. Streptococcus pyogenes T. Toxoplasma gondii All of the infectious diseases described below occur in children. For each one, select the most likely underlying causative agent from the list of options 1) A 3 year old boy is referred to hospital with a two day history of lethargy, irritability and poor feeding. On examination, he is pyrexial, drowsy and has a purpuric rash on his trunk and extremities. CSF obtained from a lumbar puncture is cloudy and contains 540 white cells/mm 3 (90% polymorphs) and 5 red blood cells/mm 3 . J. Neisseria meningitidis 2) A 6 year old girl presents with a one week history of febrile illness with sore throat and headache. One day prior to hospital admission, the patient awoke with pain and swelling in the right ankle. On examination, she has a warm swollen right ankle and a systolic heart murmur, consistent with mitral regurgitation. S. Streptococcus pyogenes 3) A new born infant is found to be lethargic and has a distended abdomen immediately after birth. On examination, the infant is jaundiced and has hepatosplenomegaly. A cranial CT scan reveals periventricular calcification. B. Cytomegalovirus 4) A 7 year old male child is referred to hospital by the general practitioner with acute renal failure. The child had bloody diarrhoea and a low grade fever a week ago; both resolved with rehydration. D. Escherichia coli 5) A 4 month old female infant is brought to the hospital with severe respiratory distress. Five days previously, she had a cough and rhinitis. On examination her temperature is 38.9 o C, pulse 180/min and the respiratory rate 80/min. She had subcostal retractions and nasal flaring. On auscultation, there are rhonchi and wheezes all over her chest. M. Respiratory syncytial virus Comments: 1. The diagnosis of Meningococcal meningitis is clear. 2. Rheumatic fever results from immune-mediated post Group A streptococcal infection, Streptococcus pyogenes being a common pathogen. 3. Congenital CMV infection is associated with petechiae, choroidoretinitis, hepatosplenomegaly, intracerebral calcification which may lead to CNS damage with long term sequaelae. 4. E. coli infection has resulted in Haemolytic Uraemic Syndrome with renal failure. 5. Respiratory Syncytial Virus is the commonest cause of lower respiratory tract infections in children worldwide, and is the leading cause of bronchiolitis and pneumonia in children. Pediatric extended match questions Dr samed alsalmi
  • 35. 17) Viral infections A. Measles B. Rubella C. Chicken Pox D. Herpes Simplex E. Mumps F. Glandular fever G. Pertussis H. Polio myelitis I. Hepatitis A For each description below choose the single most likely viral infection from the list of options 1) Caused by a gram negative pleomorphic bacillus G. Pertussis 2) Is caused by a paramyxovirus. E. Mumps 3) This infection is often followed by a transient immuno-deficiency. F. Glandular fever Comments: Whooping cough is not uncommon in infancy. It typically presents with apnoeic episodes or cyanotic episodes during infancy. In the older child upper respiratory tract infections and a paroxysmal cough with a whoop is characteristic. Mumps is caused by a paramyxovirus. Glandular fever virus infects the B lymphocytes which results in an immuno-deficiency which is usually self limiting. 18) Childhood Diarrhoea A. Adenovirus B. Cytomegalovirus C. Epstein-Barr virus D. E. Coli E. Giardia Lamblia F. Hepatitis A G. Norwalk virus H. Rotavirus I. Salmonella species J. Staphylococcus aureus Select one option from the list above that is most suitable for the following patients 1) Causes diarrhoea by invading the brush border of the small intestine and causes vacuolation. I. Salmonella species Note: Salmonella may contaminate foods improperly foods cooked or stored and invades the brush border of the small intestine. Symptoms include nausea, vomiting and diarrhoea approximately between 1-8 hours after ingestion. 2) Spread is due to personal contact as well as by contaminated water supply. E. Giardia Lamblia Note: Pediatric extended match questions Dr samed alsalmi
  • 36. Giardia Lamblia is a protozoa and may contaminate water or be transmitted by the faecal or oral route or person to person contact. Children are often asymptomatic however diarrhoea and cramps and weight loss may be features of the disease. 3) An organism which is not only associated with gastroenteritis but is also the most common cause of osteomyelitis and arthritis in children. J. Staphylococcus aureus Note: Staphylococcus aureus is a gram positive coccus and the most common cause of osteo-myelitis and arthritis in children. It may contaminate skin resulting in impetigo, cellulitis, folliculitis and furunculosis. It may cause pneumonias and may contaminate food resulting in enterotoxins being released in to the intestinal tract. 19) Contra-indications to Vaccines A. BCG B. Diphtheria Tetanus Pertussis C. Hepatitis B D. Haemophilus Influenzae B E. Influenza F. Measles Mumps Rubella G. Polio H. Rubella I. Tetanus J. Tuberculin Which of the vaccines listed above would be contra-indicated in the following scenarios: 1) Contra-indicated if known to be allergic to Neomycin. F. Measles Mumps Rubella Note: refers to contra-indications to MMR, which include children with allergies to Gelatin, Neomycin or kanamycin as well as children with untreated malignant disease or altered immunity. Those receiving immuno-suppressive drugs or radiotherapy. Children who have received another live vaccine by injection within 3 weeks and children should not be given MMR within 3 months of an Immunoglobulin injection. 2) Contra-indicated in HIV positive patients. A. BCG Note: relates to vaccines in HIV infection. The department of health has advised that HIV positive subjects with or with out symptoms should not receive BCG, yellow fever or typhoid vaccinations. 3) Contra-indicated in subjects with progressive neurological conditions. B. Diphtheria Tetanus Pertussis Note: relates to DTP (and the pertussis component) which should not be given to subjects with any progressive neurological disorder particularly epilepsy and immunisation should be delayed until the condition is stable. Pediatric extended match questions Dr samed alsalmi
  • 37. 20) CHILDHOOD RESPIRATORY DISEASES A. Asthma B. Bronchiolitis C. Croup D. Cystic fibrosis E. Diphtheria F. Epiglottis G. Pneumonia H. Influenza I. Retropharyngeal abscess J. Whooping cough For each patient below, choose the SINGLE most probable diagnosis from the above list of options. Each option may be used once, more than once or not at all. 1) A six month old baby presents with high fever, breathlessness, cough and feeding difficulties. Chest examination reveals dull percussion note over the right base posteriorly with bronchial breath sounds on auscultation. G. Pneumonia Note: Pneumonia may occur at any age. Patients present with fever, tachypnoea, feeding difficulties and cyanosis. Examination reveals bronchial breath sounds and crepitations. Chest X ray may show consolidation. Common organisms are pneumococcus, haemophilus, staphylococcus, mycoplasma, TB and viruses. 2) A 1 year old baby boy is wheezy, coughing, cyanosed and breathless with intercostal recession. B. Bronchiolitis Note: Acute bronchiolitis is very common in infancy. In winter epidemics of respiratory syncytial virus infection are the commonest cause. Wheeze, cough, fever and respiratory distress are common. Chest X ray shows hyperinflation. 3) A 4 year old non immunized boy presents with bouts of coughing ending in vomiting. He has an absolute lymphocytosis. J. Whooping cough Note: Whooping cough is caused by Bordetella pertussis infection. bouts of coughing ending in vomiting, especially at night and after feeding suggest the diagnosis. The characteristic whoop, forced inspiration through a closed glottis may or may not be present. Absolute lymphocytosis is common. Complications include CNS haemorrhages, rectal prolapse and bronchiectasis. 4) A nine month old baby girl is upset and has stridor. Her voice is hoarse and has a barking cough. She has a low grade fever. C. Croup Note: Croup usually occurs in epidemics in autumn or spring. Causative viruses are Parainfluenza (types 1, 2, or 3), respiratory syncytial viruses and measles virus. Onset is over a few days; stridor is harsh and occurs only when child is upset. A barking cough, harsh voice and ability to swallow secretions are typical. 5) A 2 year old boy is very unwell. His temperature is 39oC and he is unable to swallow his secretions. F. Epiglottis Note: Acute epiglottis is due to Haemophilus influenza type B infection. It is characterised by sudden onset, high fever, and continuous stridor and drooling of secretions. Intravenous antibiotics, anaesthetic support are usually indicated. Pediatric extended match questions Dr samed alsalmi
  • 38. 21) CONGENITAL AND NEONATAL DEFECTS DUE TO MATERNAL INFECTIONS A. AIDS B. Cytomegalovirus (CMV) C. Coxsackie group B D. Hepatitis B E. Herpes simplex F. Listeriosis G. Rubella H. Syphilis I. Toxoplasmosis J. Varicella For each patient below, choose the SINGLE most probable diagnosis from the above list of options. Each option may be used once, more than once or not at all. 1) A newborn baby presents with rudimentary digits, limb hypoplasia and convulsions. J. Varicella Note: Chicken pox infection within the first 20 weeks of pregnancy may result in the congenital varicella syndrome. This is characterised by cerebral cortical and cerebellar hypoplasia, microcephaly, convulsions, limb hypoplasia and rudimentary digits. Prevention is by administering varicella vaccine even before pregnancy. Varicella immunoglobulin is administered to pregnant women who are exposed to infection. Infection during pregnancy is treated with acylovir. 2) A six week old baby is confirmed to have cataracts, cardiac abnormalities, thrombocytopenia and cerebral calcification. G. Rubella Note: This baby has congenital rubella. It occurs in children of non immunised women. Symptoms are absent in 50% of mothers. The foetus is most vulnerable in the first 16 weeks of pregnancy. Cataracts are associated with infections in weeks 8-9, deafness at 5-7 weeks and cardiac lesions from 5-10 weeks. Diagnosis is based on rising antibody titres in blood taken 10 days apart and the presence of IGM antibodies at 4-5 weeks from incubation period. 3) A pre term neonate has multi-organ disease with granulomas on his skin. His mother had a special liking for soft cheese during her pregnancy. F. Listeriosis Note: Maternal listeriosis is usually a mild infection but transplacental infection and premature labour may occur in about 5% of cases. Avoidance of partially cooked meats, soft cheeses and unpasteurised milk should avoid this infection. Treatment is with ampicillin and gentamicin. Neonatal infection is usually multi-organ and granulomas may be found on the skin and the pharynx. 4) A two week old baby has microcephaly, seizures and chorioretinitis. I. Toxoplasmosis Note: Maternal and foetal toxoplasma infection may be avoided by advising pregnant women to wear gloves when gardening or handling cat litter and to thoroughly cook meat. Affected babies are treated with pyrimethamine, sulphadiazine and folic acid. 5) A new born baby is very unwell with jaundice, hepatosplenomegaly and microcephaly. B. Cytomegalovirus (CMV) Note: Maternal CMV infection is usually mild and asymptomatic. 5 in 1000 live births are affected, 5% will develop cytomegalic inclusion disease. The foetus is most at risk in early pregnancy. There is no effective prevention. Pediatric extended match questions Dr samed alsalmi
  • 39. 22) Childhood chest infections A. Chlamydia B. Cytomegalovirus C. Group B Streptococcus D. Haemophilus Influenza E. Mycoplasma pneumonia F. Parainfluenza G. Respiratory syncytial virus H. Rhinovirus I. Staphylococcus aureus J. Ureaplasma Select the most appropriate pathogen from the above list that would account for the following presentations: 1) A 4 year old boy presents with a rapid history of high fever and drooling. D. Haemophilus Influenza 2) A 4 year old girl presents with a 24 hour history of rhinitis, a barking cough and hoarseness. F. Parainfluenza 3) The commonest cause of pneumonia in the neonate. C. Group B Streptococcus Comments: Item 1 describes a child with acute epiglottitis. This is a rare infection mainly caused by Haemophilus influenza. Airway obstruction can develop rapidly due to oedema around the epiglottis. Item 2 describe a child with croup. Acute laryngotracheal bronchitis. It is almost exclusively viral in origin. Mainly Parainfluenza. Streptococcus pneumonia is a common pathogen in the lung and the commonest cause of pneumonia in the neonate. 23) Mucocutaneous infections and infestations A. Enterobiasis vermicularis B. Chlamydia C. Candidiasis D. Tinea pedis E. Pityriasis rosea F. Scabies G. Ringworm H. Lichen sclerosis I. Napkin dermatitis J. Lichen planus For each of the following children who present with an itch, select the most appropriate diagnosis from the list of options: 1) A child has itching in the vulva region mostly at night. A. Enterobiasis vermicularis Note: Enterobiasis vermicularis is also known as threadworms. It is a common infestation in children and present with nocturnal anal pruritus and a perianal irritation. 2) A child presents with annular regions over the trunk. G. Ringworm Pediatric extended match questions Dr samed alsalmi
  • 40. Note: Ringworm also known as tinea corporis presents with plaques of scaling eczema which are characteristically itchy. 3) A diabetic child presents with a vulva rash. C. Candidiasis Note: Candidiasis is caused by Candida albicans (yeast). In babies it presents as a perineal rash which usually affects the flexures. Satellite lesions may also be seen. 24) Presentation of infectious disease in childhood A. Chicken Pox B. Herpes simplex C. Infectious mononucleosis D. Measles E. Mumps F. Mycoplasma G. Pertussis H. Rubella I. Tuberculosis For each presentation of infectious disease choose the single most likely diagnosis from the list of options. 1) Commonly causes an acute gingivostomatitis. B. Herpes simplex Note: Herpes simplex - the majority of children have benign manifestations of primary infection with Herpes simplex, for example a gingival stomatitis. The virus is readily spread by direct contact especially to damaged skin e.g. eczema. 2) Causing an acute parotitis E. Mumps Note: Mumps infection is now uncommon due to the vaccination. It is caused by a paramyxovirus and usually causes minimal symptoms. The most common manifestation being an acute parotitis although severe infection with mumps may result in meningoencephalitis plus deafness. 3) Causing an exudative tonsillitis. C. Infectious mononucleosis Note: Glandular fever is also called infectious mononucleosis. It is caused by Epstein-Barr virus and usually presents with an exudative pharyngitis or tonsillitis and cervical lymphadenopathy. It may cause a transient impairment of cellular and humeral immunity, which is usually self limiting. Pediatric extended match questions Dr samed alsalmi
  • 41. 25) Childhood respiratory infections A. Allergic Bronchopulmonary aspergillosis B. Aspergilloma C. Bordetella pertussis D. Chlamydia E. Coxsackie B F. Klebsiella pneumonia G. Mycoplasma pneumonia H. Pneumocystis carinii I. Pseudomonas aeruginosa J. Tuberculosis From the list above, select the most appropriate diagnosis for the following presentations: 1) A 5 year old boy with cystic fibrosis undergoes a routine chest x-ray which reveals an apical round lesion on the left of his chest. B. Aspergilloma Note: Aspergilloma is a fungus which may complicate conditions such as asthma and cystic fibrosis. Children present with a cough and wheeze and there may be a positive skin test to aspergillus with an eosinophilia and an elevated IgE. Chest x-ray may reveal an apical round mass. 2) A 5 year old girl with a high temperature presents with painful blisters on the palms and soles of her feet E. Coxsackie B Note: describes a child with hand foot and mouth, which is typically caused by Coxsackie B. Characteristically individuals develop papules which then progress to vesicles. They are painful and pruritic and typically affect acral areas. It is often associated with a high temperature. 3) A 14 year old boy presents with fever, anorexia and loss of weight of 3 months duration. Scattered crepitations are heard over both lungs. Chest x-ray is abnormal with generalised mottling. J. Tuberculosis Note: TB- Mycobacterium Tuberculosis is a primary infection which may occur in the lung, gut or skin. The local infection spreads to the surrounding lymph nodes, which constitutes the primary complex in the lungs. Progression of the primary complex may result in bronchopneumonia or bronchial obstruction secondary to enlargement of the lymph nodes. Pleural effusions may also occur as well as cavitations. Primary TB may spread to the blood stream resulting in TB. Examination may reveal hepatosplenomegaly and fundoscopy may reveal choroid tubercles. 26) Treatment of infectious disease A. Acyclovir B. Acyclovir plus Cefotaxime C. Ampicillin plus Gentamicin D. Benzylpenicillin E. Ceftriaxone F. Ciprofloxacin G. Flucloxacillin H. Netilmicin I. Symptomatic treatment J. Teicoplanin Pediatric extended match questions Dr samed alsalmi
  • 42. For each case of infectious disease described below choose the single best treatment option from the list. 1) A 7 year old child develops an exudative tonsillitis and lethargy, monospot is positive I. Symptomatic treatment Note: Glandular fever where the treatment is symptomatic / conservative only. 2) A 1 year old child presents with neck stiffness and a purpuric rash. D. Benzylpenicillin Note: A child with meningococcaemia presented to the surgery and Benzylpenicillin needs to be given urgently prior to transfer to hospital. 3) A 4 year old child presents with encephalopathy. The EEG shows high amplitude, abnormal waveforms. The MRI scan is also abnormal. A diagnosis of encephalitis is made. B. Acyclovir plus Cefotaxime Note: A child with encephalitis. The majority of cases are secondary to viruses although toxic and metabolic causes need to be considered. Of the viruses Herpes simplex is the most common agent causing encephalitis. When encephalitis is suspected both antibiotics and Acyclovir must be commenced immediately. 27) Cutaneous manifestations of infectious disease A. Rheumatic fever B. TB C. Lyme's disease D. Chicken pox E. Histoplasmosis F. Cat scratch disease G. Measles H. Parvovirus I. Hepatitis B J. Herpes simplex For each cutaneous manifestation described below choose the single most likely associated infectious disease. 1) Erythema infectiosum. H. Parvovirus Note: Erythema infectiosum also known as Fifths disease caused by human parvovirus, often results in a low grade temperature, slapped cheek appearance to the face and a reticular lacy rash to the arms. 2) Gianotti-Crosti. I. Hepatitis B Note: Gianotti-Crosti is a syndrome of non-pruritic erythematous papules on the face, buttocks and extremities. Characteristically related to Hepatitis B infection although other viruses for example EBV may be associated. 3) Koplik spots. G. Measles Note: Koplik spots are white coarse granules found on the buccal mucosa opposite the back molars and is pathognomonic for measles. Pediatric extended match questions Dr samed alsalmi
  • 43. 28) Congenital and neonatal defects due to maternal infection A. AIDS B. Coxsackie group B C. Cytomegalovirus (CMV) D. Hepatitis B E. Rubella F. Herpes simplex G. Listeriosis H. Syphilis I. Varicella J. Toxoplasmosis For each patient below, choose the SINGLE most probable diagnosis from the above list of options. Each option may be used once, more than once or not at all. 1) A newborn baby presents with rudimentary digits, limb hypoplasia and convulsions. I. Varicella 2) A six week old baby is confirmed to have cataracts, cardiac abnormalities, thrombocytopenia and cerebral calcification. E. Rubella 3) A pre term neonate has multi-organ disease with granulomas on his skin. His mother had a special liking for soft cheese during her pregnancy. G. Listeriosis 4) A two week old baby has microcephaly, seizures and chorioretinitis. J. Toxoplasmosis 5) A new born baby is very unwell with jaundice, hepatosplenomegaly and microcephaly. C. Cytomegalovirus (CMV) Comments: Rudimentary digits and limb hypopkasia suggest congenital chicken pox infection. The presence of cataracts, cardiac abnormalities, thrombocytopaenia and cerebral calcification suggest a diagnosis of Congenital rubella. The presence of Choroido-retinitis strongly suggests a diagnosis of Toxoplasmosis. The baby born with granulomas and whose mother has a penchant for soft cheeses suggests Listeria Monocytogenes infection. Chorio-retinitis suggests The presence of Jaundice, HSM and microcephaly suggests CMV infection. 29) CHILDHOOD VIRAL INFECTIONS A. Adenovirus B. Coxsackie C. Cytomegalovirus D. Epstein barr E. Measles F. Molluscum contagiosum G. Mumps H. Rotavirus I. Rubella J. Varicella For each patient with the group of symptoms listed below, choose the SINGLE most probable causative agent from the above list of options. Each option may be used once, more than once or not at all. Pediatric extended match questions Dr samed alsalmi
  • 44. 1) A two year old infant boy is admitted to hospital with vomiting, none bloody watery diarrhoea and is dehydrated. It emerges other children from his play group have developed a similar illness. H. Rotavirus Note: Rotavirus is the most common cause of severe viral gastroenteritis worldwide. Infection is via the faeco- oral route and often occurs in children aged between six months to six years. This RNA virus replicates in the intestinal mucosal cells damages transport mechanisms leading to salt and water depletion which results in diarrhorea and vomiting. Diagnosis is made from clinical features and culture of virus from stools and also by Polymerase chain reaction techniques. Treatment is mainly re-hydration and correction of any electrolyte imbalance. 2) A two year old boy is mildly unwell. His mother has noticed vesicles in his mouth, palms and soles of his feet. B. Coxsackie Note: Coxsackie A16 virus is the cause of hand, foot and mouth disease characterized by fever, sore throat and ulcerating vesicles in palms, orophaynx and on soles. Incubation period is 5-7 days and these heal without crusting. Treatment is symptomatic. 3) A Three-year baby girl presents with a macular confluent rash which appeared initially behind the ears and is spreading. Over the previous five days she has had a low grade fever, catarrh and conjunctivitis. Her mother is vague about her immunization history. E. Measles Note: Measles is caused by and RNA paramyxovirus and occurs worldwide. Outbreaks are common in areas with high numbers of non immunized children. Infection is transmitted via respiratory droplets and incubation period is 10-21 days. The prodromal stage fever conjunctivitis, runny nose and coughing lasts for five days. Koplik’s spots are bright red lesions with a central white dot which appear on the buccal mucosa. These are virtually diagnostic. The typical macular confluent rash appears on the face from day 3-5 and spreads to the rest of the body. Diagnosis is made from clinical features, viral culture from lesions and a grater than 4-fold rise in antibody titres. Otitis media, pneumonia, meningitis and very rarely several years after primary infection subacute sclerosing panencephalitis (SSPE). 4) A Ten year old girl develops an itchy rash on her body which is spreading to her body. She traveled to see her grandmother who was unwell with a painful rash three weeks ago. J. Varicella Note: Varicella (Chicken pox) is transmitted by respiratory droplets and contact with somebody with shingles. Incubation period is 14-21 days and following a brief period of malaise, an itchy papulovesicular rash appears appears on trunk and spreads to the head and the extremities. The rash evolves from papules to vesicles, pustules and finally crusts. Antiviral therapy is reserved for systemic disease in the immunocompromised. 5) The four month old baby daughter of an HIV positive mother is admitted to hospital with seizures. She has neonatal jaundice and microcephaly. C. Cytomegalovirus Note: Cytomegalovirus inclusion disease is the result of infection of the foetus. Many organs may be affected and congenital abnormalities result. Microcephaly, seizures, neonatal jaundice, hepatosplenomegaly, deafness and mental retardation are some of the features that may occur. Pediatric extended match questions Dr samed alsalmi
  • 45. 30) Cutaneous manifestations of infectious disease A. Rheumatic fever B. TB C. Lyme's disease D. Chicken pox E. Histoplasmosis F. Cat scratch disease G. Measles H. Parvovirus I. Hepatitis B J. Herpes simplex For each cutaneous manifestation described below choose the single most likely associated infectious disease. 1) Erythema nodosum in presence of abnormal chest x-ray. B. TB Note: Erythema nodosum may be associated with streptococcal reactions, rheumatic fever and Tuberculosis. In the presence of an abnormal chest X-ray TB is the most likely answer. 2) Erythema marginatum A. Rheumatic fever Note: Erythema marginatum is one of the five major criteria to make a diagnosis of rheumatic fever. It is a pink rash with pale centres and a serpiginous margin, found on the trunk and proximal limbs. 3) Erythema chronicum migrans. C. Lyme's disease Note: Erythema chronicum migrans found in Lyme's disease. A febrile illness caused by Borrelia burgdorferi transmitted by bites of animal tic. The rash is characterised by red margins and central clearing. Pediatric extended match questions Dr samed alsalmi
  • 46. Neurology Theme : Genetics - clinical abnormalities of limbs A. Bloom's syndrome B. Cockayne's syndrome C. Down's syndrome D. Ehlers Danlos syndrome E. Hunter's syndrome F. Prader-Willi syndrome G. Rubinstein-Taybi syndrome H. Russell Silver syndrome I. Sotos' syndrome J. Williams syndrome In a child with a suspected genetic disorder and the following clinical abnormalities choose the single most likely diagnosis from the list of options. 1) Asymmetry of limbs and clinodactyly H. Russell Silver syndrome Note: In Russell-Silver syndrome skeletal features of this condition include short stature of pre-natal onset with asymmetry, most commonly of the limbs. There is a short inward curving to the 5th finger (clinodactyly). 2) Sandal gap. C. Down's syndrome Note: In Down's syndrome 'Sandal gap' is found. This is wide gap between the first and second toes. Plantar creases between the first and second toes are usually deep and there is characteristic dermal ridge pattern. Simian creases are found in approximately 45% of babies with Down's syndrome. This is a single horizontal palmer crease. 85% of affected individuals have a distal positioning of the palmer axial tri-radius. 3) Lax ligaments and joint hypermobility. D. Ehlers Danlos syndrome Note: In Ehlers Danlos syndrome there is joint and skeletal hyper-extensibility with poor wound healing. Blood vessels are fragile and this often manifests itself with recurrent bruising. Affected individuals tend to have characteristic facial features including epicanthic folds, blue sclerae and a narrow mandible. Comments: Examination of the hand sand feet are important in the assessment of a dysmorphic infant. Theme : Neurological investigation Pediatric extended match questions Dr samed alsalmi
  • 47. A. Angiography B. CT head scan C. ECG D. EEG E. Lumbar puncture F. MRI scan G. No investigation required H. Skeletal survey I. Skull x-ray J. Tensilon test Select the most appropriate investigation for the following cases 1) A 6 month old baby with microcephaly is brought to A&E. His mother gives a history of the baby flexing forwards whilst sat in her highchair. D. EEG 2) A 6 year old boy with a previous history of convulsions and abnormal EEG's presents with tingling sensations in his right thumb which precedes rapid jerking of his right arm. B. CT head scan 3) A 2 year old boy with an otitis media is brought to casualty. His father reports that the child has had a fit. Temperature recorded is 38.8°C. G. No investigation required Comments: Item 1 relates to a baby with infantile spasms. Brief symmetrical contractions of the neck, trunk and extremities. It is related to pre-natal, peri-natal and post-natal causes for example hypoxic ischaemic encephalopathy, congenital infections and metabolic disease. The baby requires an EEG and this will show hypsarrhythmia, a chaotic high amplitude slow wave and spike abnormality. Item 2 describes a boy with focal seizures, previous EEG's have been abnormal. A CT scan is indicated to exclude underlying space occupying lesions. The treatment of choice would be Carbamazepine. Febrile convulsions are very common they manifest as generalised tonic clonic seizures and occur with rapidly rising temperatures. 50% of children having had one febrile convulsion are likely to have a second. No investigations are required. Theme : Neurological investigation A. Angiography B. CT head scan C. ECG D. EEG E. Lumbar puncture F. MRI scan G. No investigation required Pediatric extended match questions Dr samed alsalmi
  • 48. H. Skeletal survey I. Skull x-ray J. Tensilon test Select the most appropriate investigation for the following cases 1) A 5 year old boy presents with screaming at night usually between midnight and 2am. G. No investigation required 2) A 3 month old with a coryzal illness presents to surgery for his first immunisations. He has bruising to both cheeks and mother reports that it is due to rolling off the settee. H. Skeletal survey 3) A 3 year old boy is brought to hospital for investigations with a history that he is progressively getting more tired by the end of the day and an inability to swallow. J. Tensilon test Comments: Item 1 describes a child having night terrors. These are very common (approximately 3% of all children). Typically a child between the ages of 5 and 7 will awake between the age of 12am and 2am screaming. They appear frightened, they may have a tachycardia and dilated pupils. Children tend to fall back to sleep with total amnesia of the event the following morning. Item 2 describes a baby having sustained physical, non-accidental injury. This diagnosis must be suspected if medical findings are unexplained or the history offered is inconsistent. Also if the child suggests that the injury has been caused by an adult. Characteristic injuries include bruises as in Item 3 and further investigations including blood tests, skeletal survey, plus or minus a CT head scan must be considered. The last question describes a child with progressive weakness, hypotonia and an inability to suck and swallow. Myasthenia Gravis must be considered, this is a defect in the neuromuscular junction with acetyl choline receptor antibodies. A Tensilon test may show an increase in muscle strength. Theme : Side effects of Anti-Convulsants A. Carbamazepine B. Clonazepam C. Ethosuximide D. Gabapentin E. Lamotrigine F. Phenobarbitone G. Phenytoin H. Sodium Valproate I. Topiramate J. Vigabatrin Select one of the drugs from the list of options that corresponds with the listed side effect: Pediatric extended match questions Dr samed alsalmi
  • 49. 1) Increased appetite resulting in weight gain. H. Sodium Valproate 2) Visual field defects. J. Vigabatrin 3) Is associated with hyperammonaemia. H. Sodium Valproate Comments: Sodium Valproate has many side-effects including gastric irritation and nausea as well as Hyperammonaemia (interferes with the urea cycle, reducing Carnitine concentrations and has been associated with hyperammonaenic encephalopathy), increased appetite and weight gain and transient hair loss. Vigabatrin is used for treatment of partial epilepsy with or with our secondary generalisation, usually in combination with other anti-epileptic drugs although can be used as monotherapy in the management of infantile spasms. Side-effects include drowsiness and confusion and visual field defects. Theme : Side effects of Anti-Convulsants A. Carbamazepine B. Clonazepam C. Ethosuximide D. Gabapentin E. Lamotrigine F. Phenobarbitone G. Phenytoin H. Sodium Valproate I. Topiramate J. Vigabatrin Select one of the drugs from the list of options that corresponds with the listed side effect: 1) Needs to be withdrawn if a rash develops. E. Lamotrigine 2) Coarse facial features, hirsutism and gingival hypertrophy. G. Phenytoin 3) Anorexia leading to weight loss. Pediatric extended match questions Dr samed alsalmi
  • 50. I. Topiramate Comments: Lamotrigine is an anti-convulsant used for treatment of partial seizures as well as generalised seizures. It is also used for myoclonic seizures, atypical absences and Lennox-Gastaut syndrome. Lamotrigine may cause a serious skin rash in children and consideration must be given to withdrawal of the drug should the rash develop. Phenytoin is effective in the treatment of tonic-clonic seizures and partial seizures. It may cause facies, acne, hirsutism and gingival hyperplasia. Topiramate can be given as adjunctive treatment for partial seizures with or with secondary generalisation. One of the main side-effects is anorexia leading to weight loss. Pediatric extended match questions Dr samed alsalmi
  • 51. Pediatrics 1 Pediatrics 1) Theme: ABDOMINAL PAIN A. Peritonitis B. Appendicitis C. Constipation D. Urinary tract infection E. Mesenteric lymphadenitis F. Migraine G. Intussusception H. Henoch-Schonlein vasculitis I. Psychosomatic J. Shigella dysentry For the following patients with abdominal pain, choose the most likely diagnosis. 1) A previously well 10 month old baby presenting with drawing up of her legs with crying and some bloody runny stools. G. Intussusception Note: This is the right age group for intussception. It is often preceded by an URTI and it is thought that mesenteric lymphadenopathy may precipitate intussception. A red currant jelly-like stool is a late sign. In the older child, the cause of intussception may be HSP or lymphoma. 2) A 4-year-old child, with poor eating habits, iron deficiency anaemia and intermittent colicky abdominal pain. C. Constipation Note: Constipation is common in children and should be managed aggressively to establish regular bowel habit quickly. 3) A 2-year-old girl with nephrotic syndrome who has developed a fever and abdominal pain. A. Peritonitis Note: The causes of abdominal pain in nephrotic syndrome are peritonitis (usually pneumococcal), renal vein thrombosis and hypovolaemia. 2) Theme: Skin lesions A. Café au lait spots B. Capillary haemangioma C. Port-wine stain D. Shagreen patch E. Adenoma sebaceum F. Cutis Marmorata G. Herald's patch H. Peri-oral pigmentation I. Hypo pigmentation J. Tuber Select one option from the list above that is most suitable for the following patients 1) An 11 year old boy with learning difficulties has an acne type rash over the nose E. Adenoma sebaceum Note: relates to the condition known as Tuberous sclerosis. It is a condition where by hamartomatous lesions become evident in early childhood. Seizures and learning difficulties are common and skin manifestations include fibrous angiomatous lesions in the naso labial folds known as adenoma sebaceum. White macules are seen over the trunk known as Ash leaf macules and Shagreen patches resemble goose like flesh. Subungual fibromata are also common. 2) A 4 year old boy with a history of focal seizures and a birth mark C. Port-wine stain Note: relates to Sturge-Weber syndrome. The association and localization of aberrant vasculature in the facial skin, eyes and meninges. Most commonly in the trigeminal nerve distribution. 3) An infant with Down's syndrome has mottled appearance to the skin. F. Cutis Marmorata Note: relates to Cutis Marmorata, a mottled, marbled type appearance to the skin. Common in Down's syndrome as well as other Trisomies, hypothyroidism and Cornelia-de-Lange syndrome. Pediatric extended match questions Dr samed alsalmi
  • 52. Pediatrics 2 3) Theme: Poisoning A. Ant psychotics B. Benzodiazepines C. Ethylene glycol D. Methanol E. Opiates F. Organophosphates G. Salicylates H. Sympathomimetic I. Tricyclic antidepressants J. Volatile solvents Select the most appropriate agent from the above list that would elicit the following toxic effects: 1) Coma, pinpoint pupils, hyperventilation E. Opiates 2) Hyper salivation, broncho-rrhoea, broncho- spasms, perspiration, neuromuscular paralysis F. Organophosphates 3) Tachypnoea, metabolic acidosis, Haematemesis, Reyes Syndrome. G. Salicylates Comments: A pinpoint pupil suggests : opiates (Heroin – Morphine – codeine). The hyper salivation, tachypnoea and sweating suggest :Organophosphate poisoning. This drug inhibits ant-cholinesterase activity promoting Ach effects. Treatment consists of anticholinergics < (Atropine dose i.v 0.05 mg/kg repeated every 5-10 min as needed adverse effect tachycardia, dry mouth, blurred vision, and urinary retention). Finally, the use of salicylates in children is well-recognized cause of Reyes syndrome, a condition associated with fulminant hepatic failure. Suspected poisoning in children result in thousands of attendances at A&E department each year. Various drugs cause specific signs when taken in overdose. Careful examination of the child should follow A, B, C protocol. 4) Theme: Chemotherapy Select the most appropriate chemotherapeutic agent from the above list that corresponds with the following actions and side effects: 1) A drug disrupting microtubule leading to loss of deep tendon reflexes. J. Vincristine 2) A drug inhibiting initiation of DNA synthesis resulting in conjunctivitis and cerebellar toxicity. G. Cytosine arabinoside 3) A drug resulting in dose related lung damage. D. Bleomycin Comments: All cytotoxic agents have adverse effects. General side-effects include nausea, vomiting and bone marrow suppression, alopecia and stomatitis. More specific side- effects are listed in the questions above. Alkylating agents such as Cyclophosphamide may cause a haemorrhagic cystitis. Asparaginase results in a dose related Pancreatitis. The anthracyclines e.g. Doxorubicin and Daunorubicin are cardiotoxic, which is often very difficult to detect. Monitoring with echocardiograms is advised. Vincristine is an alkaloid agent and results in sensory motor neuropathy with long-term use. It also has an affect on the autonomic system resulting in severe constipation and paralytic ileus. It may also result in sensory changes with parathesis progressing to loss of tendon reflexes. Bleomycin is the main drug resulting in lung damage and occurs in up to 10% of patients. The damage is dose related. Pediatric extended match questions Dr samed alsalmi